You are on page 1of 365

Honors: General Chemistry

CHEM 1422
Section 1

Spring, 2010
9:10 - 10:30 T-Th
E137 Howe Russell

Prof. George G. Stanley


614 Choppin Hall
578-3471
E-mail: gstanley@lsu.edu

Chapter 15
(not much on E)

Thermodynamics:
Enthalpy, Entropy
& Gibbs Free
Energy

Thermo 2

Thermodynamics:

thermo = heat (energy)


dynamics = movement, motion

Some thermodynamic terms chemists use:


System: the portion of the universe that we are
considering
open system:
energy & matter can transfer
closed system:
energy transfers only
isolated system: no transfers
Surroundings: everything else besides the system
Isothermal: a system that is kept at a constant
temperature by adding or subtracting heat from the
surroundings.
Heat Capacity: the amount of heat energy required to
raise the temperature of a certain amount of material by
1C (or 1 K).
Specific Heat Capacity: 1 g by 1C
Molar Heat Capacity: 1 mole by 1C

Thermo 3

Calorie: the amount of heat required to raise the


temperature of 1g of water by 1C.
specific heat of water = 1 cal/g C

1 calorie = 4.18 joules


Specific Heats and Molar Heat Capacities
Substance

Specific Heat (J/Cg)

Molar Heat (J/Cmol)

Al
Cu
Fe
CaCO3

0.90
0.38
0.45
0.84

24.3
24.4
25.1
83.8

Ethanol

2.43

112.0

Water

4.18

75.3

Air

1.00

~ 29

important to:

engineers

chemists

EXAMPLE: How many joules of energy are needed to raise


the temperature of an iron nail (7.0 g) from 25C to 125C?
The specific heat of iron is 0.45 J/Cg.
Heat energy = (specific heat)(mass)(T)
Heat energy = (0.45 J/Cg)(7.0 g)(100C) = 315 J
Note that T can be C or K, but NOT F. When just T is being used
in a scientific formula it will usually be kelvin (K).

Thermo 4

Problem: How much energy does it take to raise


the body temperature 2.5C (a fever of just over
103F) for someone who weighs 110 pounds (50 kg).
Assume an average body specific heat capacity of
3 J/C.g.

Problem: What would be more effective at melting


a frozen pipe hot water or a hair dryer (hot air
gun). Why?

Thermo 5

State Functions
System properties, such as pressure (P), volume (V),
and temperature (T) are called state functions.
The value of a state function depends only on the state
of the system and not on the way in which the system
came to be in that state.
A change in a state function describes a difference
between the two states. It is independent of the
process or pathway by which the change occurs.
For example, if we heat a block of iron from room
temperature to 100C, it is not important exactly how
we did it. Just on the initial state and the final state
conditions. For example, we could heat it to 150C,
then cool it to 100C. The path we take is unimportant,
so long as the final temperature is 100C.
Miles per gallon for a car, is NOT a state function. It
depends highly on the path: acceleration, speed, wind,
tire inflation, hills, etc.
Most of the thermodynamic values we will discuss in
this chapter are state functions.

Thermo 6

Energy: "The capacity to do work


and/or transfer heat"
Forms of Energy:
Kinetic (Ekinetic = mv 2)
Heat
Light (& Electromagnetic)
Electricity
Sound
Potential
Gravitational
Chemical
Nuclear - Matter (E = mc 2)

WORK

Thermo 7

First Law of Thermodynamics:


The total amount of energy (and
mass) in the universe is constant.
In any process energy can be
changed from one form to
another; but it can never be
created nor destroyed.

You can't get something for


nothing

Thermo 8

Enthalpy (Heats) of Reaction


The amount of heat released or absorbed by a
chemical reaction at constant pressure (as one would
do in a laboratory) is called the enthalpy or heat or
reaction. We use the symbol H to indicate
enthalpy.
Sign notation (EXTREMELY IMPORTANT!!):

+H indicates that heat is being absorbed in the


reaction (it gets cold)

endothermic

H indicates that heat is being given off in the


exothermic
reaction (it gets hot)
Standard Enthalpy = H ( is called a not)

Occurring under Standard Conditions:


Pressure
Concentration

1 atm (760 torr)


1.0 M

Temperature is not defined or part of Standard


Conditions, but is often measured at 298 K (25C).

Thermo 9

Standard Enthalpy of Formation --

H f

The amount of heat absorbed (endothermic) or


released (exothermic) in a reaction in which one
mole of a substance is formed from its elements in
their standard states, usually at 298 K (25C).
Also called heat of formation.

H f = 0 for any element in its standard state (the


natural elemental form at 1 atm or 1 M) at 298 K.
EXAMPLES:
C(graphite, s) + O 2 (g)
Hrxn = 0 kJ/mol

CO2 (g)

0 kJ/mol

elements in their
standard states
negative sign
heat released -- exothermic rxn

393.5 kJ/mol

product
(one mole)

Hf (CO2 ) =

393.5 kJ/mol

Thermo 10

2H2 (g) + O 2 (g)


Hrxn = 0 kJ/mol

0 kJ/mol

2H2 O (g)
483.6 kJ/ 2 mol

elements in their
standard states

product
(two moles)
divide by 2 to put
on per mole basis!!

negative sign
heat released -- exothermic rxn

Hf (H2 O) =

241.8 kJ/mol

Note that I usually will not have you calculate Hf on homeworks or tests
so you generally dont have to worry about normalizing your answer to a
per mole basis.

Hess's Law -- Adding Reactions


The overall heat of reaction (Hrxn) is equal to the
sum of the Hf (products) minus the sum of the Hf
(reactants):
Hrxn =

(# eqiv) H (products) (# eqiv) H (reactants)


f

Therefore, by knowing Hf of the reactants and


products, we can determine the Hrxn for any
reaction that involves these reactants and products.

Thermo 11

EXAMPLE: CO2 is used in certain kinds of fire extinguishers


to put out simple fires. It works by smothering the fire with
"heavier" CO2 that replaces oxygen needed to maintain a fire.
CO2 is not good, however, for more exotic electrical and
chemical fires.
2Mg(s)
Hf = 0 kJ/mol

CO2 (g)

2MgO(s)

- 393 kJ/mol

- 602 kJ/mol

REACTANTS

Hrxn =
Hrxn =

C(s)
0 kJ/mol

PRODUCTS

(# eqiv) H (products) (# eqiv) H (reactants)


f

(2 eqiv)(-602 kJ/mol) + (1 eqiv)(0 kJ/mol)


(2 eqiv)(0 kJ/mol) + (1 eqiv)(-393 kJ/mol)

Hrxn = (-1204 kJ/mol)


Hrxn = (-1204 kJ/mol)
Hrxn =

811 kJ/mol

(-393 kJ/mol)
+ 393 kJ/mol

highly exothermic rxn !!

Therefore, Mg will "burn" CO2 !

Thermo 12

You can also add two reactions together to get the Hrxn for
another new reaction:
Calculate Hrxn for the following reaction:
C2 H4 (g) + H 2O(l)

Hrxn = ??

C 2 H 5OH(l)

Given these two reactions and thermodynamic data:


a) C 2 H 5OH(l) + 3O 2(g)

2CO2 (g) + 3H 2 O(l)

Hrxn = -1367 kJ/mol

b)

2CO2 (g) + 2H 2 O(l)

Hrxn = -1411 kJ/mol

C2 H4 (g) + 3O 2(g)

How to solve:
1) C 2 H 5OH is on the product side of the first reaction -- so we want to
switch equation a) around to get C 2 H 5OH also on the product side:
2CO2 (g) + 3H 2 O(l)

C 2 H 5OH(l) + 3O 2(g) Hrxn = +1367 kJ/mol

note that when we


reverse the reaction, Hrxn
changes sign!!!

2) Now we can add the two reactions together to give us the desired
net reaction:
1
2CO2 (g) + 3H 2 O(l)
C 2 H 5OH(l) + 3O 2(g) Hrxn = +1367 kJ/mol

C2 H 4 (g) + 3O 2(g)

2CO2 (g) + 2H 2 O(l)

C2 H 4 (g) + H 2O(l)

C 2 H 5OH(l)

Hrxn = -1411 kJ/mol


Hrxn = -44 kJ/mol

If we have to multiply one (or more) of the reactions by some


constant to get them to add correctly, then we also would have
to multiply Hrxn for that reaction by the same amount.

Thermo 13

Chemists use bomb calorimeters to measure


Enthalpies of formation or reaction.

Bomb Calorimeter
Stirrer

Electrical
contacts
to initate
sample
combustion

Thermocouple
to measure
temperature

Sample
placed
inside
inner
container

Water
Highly insulated outside container

Thick-walled inner container


(bomb) to contain combustion
of sample (pressurized with O2)

In order to use this effectively one must know the


heat capacity of the bomb (inner part) and water
bath. By measuring the temperature increase of the
water one can calculate the amount of heat given off
during the combustion process.

Thermo 14

Problem: Calculate Hrxn for the following


reactions given the following H f values:
H f (SO2, g) = 297 kJ/mol

H f (SO3, g) = 396 kJ/mol

H f (H2SO4, l) = 814 kJ/mol

H f (H2SO4, aq) = 908 kJ/mol

H f (H2O, l) = 286 kJ/mol

H f (H2S, g) = 20 kJ/mol

a) S(s) + O2(g)

b) 2SO2(g) + O2(g)

SO2(g)

2SO3(g)

c) SO3(g) + H2O(l)

H2SO4(l)

d) 2H2S(g) + 3O2(g)

2SO2(g) + 2H2O(l)

Thermo 15

Internal Energy -- E
The internal energy, E, represents all the energy
contained within a material. It includes kinetic
energy (heat), intra- and intermolecular forces
(bond energies, electrostatic forces, van der Waals),
and any other forms of energy present. As with
enthalpy, H, the absolute value cant (or is extremely
difficult) to define.
What we can track is the change in E:
E = Efinal Einital = Eproducts Ereactants
A key relationship is:
E = q + w
Where q = heat and w = work performed on or by
the system. Sign notations:
q = positive = heat added to system (adds energy)
q = negative = heat removed (removes energy)
w = positive = work done on system (adds energy)
w = negative = work done by system (removes energy)

Thermo 16

The most common type of work involves


pressure/volume changes: e.g., explosion of gasoline
vapors in an internal combustion engine. The
explosion creates a dramatic pressure and volume
increase that pushes the piston and creates work.
If one has a constant volume situation, then no
pressure/volume work will be done and w = 0.
So under constant volume conditions:
E = q
The change in internal energy is, therefore, equal to
the amount of heat added or removed from the
material (system).

Thermo 17

Relationship Between E & H


E = q (at constant volume and temperature)
H = q (at constant pressure and temperature)
The difference is that volume changes occur for H and
that typically involves work of some type. Remember
that significant volume changes only occur when gasses
are involved. So we only need to be concerned about
volume work when there is a change in the amount of gas
produced in a chemical reaction.

The relationship between H and E is defined,


therefore, as:
H = E + (n)RT
Where R = gas constant; T = temperature in kelvin,
and:
n = equivalents (moles) of product gas
equivalents (moles) of reactant gas
If n = 0, then H = E.
But even when n 0, the PV work component is usually
small. See example in textbook (pages 574-575).

Thermo 18

Entropy
The final state of a system is more energetically
favorable if:
1. Energy can be dispersed over a greater number
and variety of molecules.
2. The particles of the system can be more
dispersed (more disordered).
The dispersal of energy and matter is described by
the thermodynamic state function entropy, S.
The greater the dispersal of energy or matter in a
system, the higher is its entropy. The greater the
disorder (dispersal of energy and matter, both in
space and in variety) the higher the entropy.
Adding heat to a material increases the disorder.
Ice - well ordered structure

water vapor - most disordered

water - more disordered

Thermo 19

Unlike H, entropy can be defined exactly because


of the Third Law of Thermodynamics:

Third Law of Thermodynamics: Any


pure crystalline substance at a
temperature of absolute zero (0.0 K) has
an entropy of zero (S = 0.0 J/K mol).

Sign notation (EXTREMELY IMPORTANT!!):

+S indicates that entropy is increasing in the


reaction or transformation (it's getting more
disordered -- mother nature likes)
S indicates that entropy is decreasing in the
reaction or transformation (it's getting less
disordered {more ordered} -- mother nature doesn't
like, but it does happen)

Thermo 20

Qualitative "Rules" About Entropy:


1) Entropy increases as one goes from a solid to a
liquid, or more dramatically, a liquid to a gas.
250

Entropy (J/mol)

Solid
200

Gas
Liquid

150
phase transitions

100
50
0

Temperature (K)

2) Entropy increases if a solid or liquid is dissolved


in a solvent.
3) Entropy increases as the number of particles
(molecules) in a system increases:
N2O4(g)
S = 304 J/K (1 mole)

2NO2(g)
S = 480 J/K (2 moles)

These first 3 above are most important for


evaluating Srxn.

Thermo 21

The rules below are mainly for comparing the


entropy of individual molecules or materials.
4) The Entropy of any material increases with
increasing temperature
5) Entropy increases as the mass of a molecule
increases
S(Cl2(g)) > S(F2(g))
S = 165 J/Kmol

S = 158 J/Kmol

6) Entropy is higher for weakly bonded compounds


than for compounds with very strong covalent
bonds
S(graphite) > S(diamond)
S = 5.7 J/Kmol

S = 2.4 J/Kmol

Note that for individual molecules (materials) the


higher the entropy, the more likely the molecule will
want to fall apart to produce a number of smaller
molecules.

Thermo 22

7) Entropy increases as the complexity (# of atoms,


# of heavier atoms, etc.) of a molecule increases
Entropy of a Series of Gaseous Hydrocarbons
H
H

Methane

S = 186 J/Kmol

S = 201 J/Kmol

Acetylene
H

H
C

Ethylene

C
H

S = 220 J/Kmol

S = 230 J/Kmol

H
Ethane

H
Propane

S = 270 J/Kmol

Thermo 23

What are the biggest factors for evaluating Srxn for


a chemical rxn?
1) phase change

2) change in # of molecules

Problem: For the following reactions, is the entropy

of the reaction increasing or decreasing?


a) Ag+(aq) + Cl-(aq)

AgCl(s)

b) H2CO3(aq)

H2O + CO2(g)

c) Ni(s) + 4CO(g)
d) H2O(s)

Ni(CO)4(l)
H2O(l)

e) graphite

diamond
2Na+(aq) + 2OH-(aq) + H2(g)

f) 2Na(s) + 2H2O
g) H2S(g) + O2(g)
h) 2H2O(l)

H2O(l) + SO(g)
2H2(g) + O2(g)

i) CO2(g) + CaO(s)

CaCO3(s)

j) CaCl2(s) + 6H2O(l)
k) 2NO2(g)

N2O4(g)

CaCl26H2O(s)

Thermo 24

Just as with enthalpies, one can calculate entropies


of reaction.
S rxn =

(# eqiv) S(products) (# eqiv) S(reactants)

EXAMPLE:
2Mg(s)

CO2 (g)

2MgO(s)

Sf = 32 J/Kmol 215 J/Kmol

27 J/Kmol

REACTANTS

S rxn =
S rxn =

C(s)
6 J/Kmol

PRODUCTS

(# eqiv) S(products) (# eqiv) S(reactants)

(2 eqiv)(27 J/Kmol) + (1 eqiv)(6 J/Kmol)


(2 eqiv)(32 J/Kmol) + (1 eqiv)( 214 J/Kmol)

S rxn = (60 J/K.mol)

S rxn =

218 J/K.mol

(278 J/K.mol)

entropy is decreasing
(reaction is becoming more ordered)

Thermo 25

Spontaneous Processes
A process that takes place without the net input of
energy from an external source is said to be
spontaneous (not instantaneous).
1) Rxn of sodium metal with water:
2Na+(aq) + 2OH-(aq) + H2(g)

2Na(s) + 2H2O

2) Combustion rxns:
C3H8(g) + 5O2(g)

3CO2(g) + 4H2O(g)

2H2(g) + O2(g)

2H2O(l)

3) Expansion of a gas into a vacuum


xCO2(g)

yCO2(s) + zCO2(g) (x = y + z)

4) A salt dissolving into solution:


NH4NO3(s) + H2O(l)

NH4+(aq) + NO3-(aq)

Thermo 26

Second Law of Thermodynamics: In


any spontaneous process the entropy
of the universe increases
Suniverse = Ssystem + Ssurroundings

Second Law (variant): in trying to do


work, you always lose energy to the
surroundings.

You can't even break even!


Neither entropy (S) or enthalpy (H)
alone can tell us whether a chemical
reaction will be spontaneous or not.
An obvious (?) conclusion is that one
needs to use some combination of the two.

Thermo 27

Gibbs Free Energy


The combination of entropy, temperature
and enthalpy explains whether a reaction is
going to be spontaneous or not. The symbol
G is used to define the Free Energy of a
system. Since this was discovered by J.
Willard Gibbs it is also called the Gibbs
Free Energy. "Free" energy refers to the
amount of energy available to do work once
you have paid your price to entropy. Note
that this is not given simply by H, the heat
energy released in a reaction.

G = H TS
When G is negative, it indicates that a
reaction or process is spontaneous. A
positive G indicates a non-spontaneous
reaction.

Thermo 28

G = H TS
S
+
G = negative

G = ??

spontaneous
at all temperatures

spontaneous
at high temperatures

G = ??

G = positive

spontaneous
at low temperatures

non-spontaneous
at all temperatures

Spontaneous = exoergic (energy releasing)


Non-spontaneous = endoergic (energy releasing)

Thermo 29

Remember that entropies are


given in units of J/Kmol while
enthalpies and free energies are in
kJ/mol.
DON'T forget to convert all units to
kJ or J when using both S and H
in the same equation!!

Thermo 30

G vs. G: Standard vs. Non-Standard Conditions


Remember that the (not) on G indicates that the
numerical value of G is based on the reaction at
standard conditions (1 M solution concentration, 1 atm
gas pressure). Temperature is NOT part of standard
conditions!
As soon as one has a concentration different than 1 M or
1 atm pressure, the not goes away and one has G.
Consider the reaction:

Initial:

1 atm
1 atm
2SO2(g) + O2(g)

1 atm
2SO3(g)

Grxn = 142 kJ/mol

The Grxn of 142 kJ/mol is for when each gas is present


with a concentration of 1 atm. This indicates that the
reaction under these conditions will proceed to make
products (spontaneous).
As the reactants start reacting, however, their
concentrations decrease (SO2 twice as fast as O2) and
G turns into G and becomes less negative.
When G = 0 the reaction has reached equilibrium.
Although for this rxn, SO2 is probably the limiting
reagent (not enough present to complete the rxn).

Thermo 31

Example: Calculate Gf for CO2 at 298 K. Hf


(CO2) = 393 KJ/mol, S (O2) = 205 J/molK, S
(C) = 6 J/molK, S (CO2) = 213 J/molK

C(graphite) + O2(g)
CO2(g)

Gf = Hf TSf
Note change in
Sf = Sprod Sreact

units J to KJ

Sf = (213 J/molK) (205 + 6 J/molK)


Sf = 2 J/molK)
Gf = (393 KJ/mol) (298 K)(0.002 KJ/molK)
Gf = (393 KJ/mol) (1 KJ/mol)
Gf = 394 KJ/mol

DANGER!!
Common
mistake!!

Problem: Calculate Gf for CO at 298 K. Hf

(CO) = 110 KJ/mol, S(O2) = 205 J/molK,


S(C) = 6 J/molK, S(CO) = 198 J/molK
2C(graphite) + O2(g)

2CO(g)

Thermo 32

Just as with enthalpies and entropies, one can


calculate free energies of reaction.
Grxn =

(# eqiv) G (products) (# eqiv) G (reactants)


f

EXAMPLE:
2Mg(s)
Gf = 0 kJ/mol

CO2 (g)

2MgO(s)

- 394 kJ/mol

- 570 kJ/mol

REACTANTS

Grxn =
Grxn =

C(s)
0 kJ/mol

PRODUCTS

(# mol) G (products) (# mol) G (reactants)


f

(2 mol)(-570 kJ/mol) + (1 mol)(0 kJ/mol)


(2 mol)(0 kJ/mol) + (1 mol)(-394 kJ/mol)

Grxn = (-1140 kJ)

(-394 kJ)

Grxn = (-1140 kJ) + 394 kJ


Grxn =

746 kJ

SPONTANEOUS rxn!
highly exothermic rxn !!

Compare to H rxn which was -811 kJ for the same rxn.


The "missing" 65 kJ of energy went to ENTROPY!

Thermo 33

Example: To make iron, a steel mill takes Fe2O3 (rust


or iron ore) and reacts it with coke (a complex, impure
form of carbon) to make iron and CO2. Based on the
data below, this is a non-spontaneous reaction at room
temperature, but it becomes spontaneous at higher
temperatures. Assuming that H and S do not
change much with temperature, calculate the temperature above which the reaction becomes spontaneous
(i.e., Grxn = 0).
Hrxn = +465 kJ/mol
Srxn = +552 J/molK (or 0.552 kJ/molK)
Grxn = +301 kJ/mol (at 298 K)

Grxn = Hrxn TSrxn


as we raise the temperature, G will eventually reach 0 and
then go negative & spontaneous, so let G = 0 and solve for
T, the temperature at which this will happen:

0 = Hrxn TSrxn
rearranging to solve for T gives:

T = (Hrxn) / (Srxn)
T = (465 kJ/mol) / (0.552 kJ/molK)
T = 842 K
(above this temperature Grxn will be negative we will have
a spontaneous reaction)

Thermo 34

Problem: Calculate Grxn for the following.


Gf (SO2, g) = 300 kJ/mol

Gf (SO3, g) = 371 kJ/mol

Gf (H2SO4, l) = 690 kJ/mol

Gf (H2SO4, aq) = 742 kJ/mol

Gf (H2O, l) = 237 kJ/mol

Gf (H2S, g) = 34 kJ/mol

a) S(s) + O2(g)

b) 2SO2(g) + O2(g)

SO2(g)

2SO3(g)

c) SO3(g) + H2O(l)

H2SO4(l)

d) 2H2S(g) + 3O2(g)

2SO2(g) + 2H2O(l)

Thermo 35

Comparisons of Hrxn and Grxn


S(s) + O2(g)

SO2(g)

Hrxn = 297 kJ/mol


Grxn = 300 kJ/mol
2SO2(g) + O2(g)

Srxn = 11 J/mol K
2SO3(g)

Hrxn = 198 kJ/mol


Grxn = 142 kJ/mol
SO3(g) + H2O(l)

H2SO4(l)

Hrxn = 132 kJ/mol


Grxn = 82 kJ/mol
2H2S(g) + 3O2(g)
Hrxn = 1126 kJ/mol
Grxn = 1006 kJ/mol

2SO2(g) + 2H2O(l)

Thermo 36

Entropy of Fusion and Vaporization


While the entropy of a substance increases steadily
with increasing temperature, there is a considerable
jump in the entropy at a phase transition:
250

Entropy (J/mol)

Solid
200

Gas
Liquid

150
100

phase transitions

50
0

Temperature (K)

This jump at the melting point is called the entropy


of fusion, Sfusion, and as you might expect, it is
related to the enthalpy (or heat) of fusion, Hfusion:

Hfusion
S fusion
Tm

Thermo 37

The jump in entropy at the boiling point is called the


entropy of vaporization, Svaporization, and it is
related to the enthalpy of vaporization, Hvaporization:

Hvaporization
S vaporization
Tb
EXAMPLE: What is the boiling point of bromine
(Br2)? Svapor = 93.2 J/Kmol and Hvapor = 30.9
kJ/Kmol.
Hvaporization
S vaporization
Tb
but we want to solve for Tb, the boiling point
temperature, so we need to rearrange the formula:

Hvaporization
Tb
Svaporization
30.9 103 J / mol
Tb
93.2 J / K mol
Tb 3315
. K 58.3 C

Note how we have to


convert kJ/mol to J/mol in
order for the units on H
to be the same as the units
on S!! To convert kJ to
J we need to multiply the
kJ value by 1000.

Thermo 38

Problem: Calculate the boiling point for ethanol

(CH3CH2OH) from the data in the thermodynamic


tables.

(literature value = 78.5C)

Thermo 39

Thermo 40

th

From General Chemistry, 7 Ed, by Whitten, Davis, Peck & Stanley. Thomson Brooks/Cole Publisher.

Kinetics 1

Chapter 16
(no derivation of kinetic rate laws)

Kinetics 2

Chemical Kinetics is the area of chemistry that is


concerned with finding out how fast a chemical rxn
will go and what is the mechanism by which it
proceeds? The mechanism of a rxn is how the
reactants react in a step-wise fashion to form the
product molecules.
Thermodynamics only tells us if the rxn is
energetically possible (exothermic, endothermic,
spontaneous) nothing about the rate of the rxn or
how it will proceed (i.e., the mechanism of the rxn).
Kinetics provides us with the techniques for
measuring the rate of a reaction and gaining
information about how the reaction is proceeding.
This, in turn, can give us valuable information
about the mechanism of the rxn.

Kinetics 3

Consider the very spontaneous and exothermic rxn


of H2 + O2 to produce H2O:

Note: most text books use E


(potential energy) for the vertical
axis. I will generally use G

The activation energy or activation barrier, when


present and large enough, is what prevents a
spontaneous reaction from reacting instantly!

Kinetics 4

The activation energy arises due to kinetic effects,


i.e., the rates at which molecules react together and
the factors that affect those rates of reactions.
Consider the rearrangement of methyl isocyanide
into the more stable acetonitrile molecule:
H3C

methyl isocyanide

CH3

acetonitrile

This reaction involves the migration (movement) of


the methyl group (H3C or CH3) from the nitrogen
atom over to the terminal carbon atom.

Kinetics 5

The initial movement of the methyl group involves


partial breaking of chemical bonds between the
methyl group and nitrogen atom and the C-N
triple bond, both of which take energy. The
topmost part of the activation energy barrier is
called the transition state, which represents the
highest energy part of the reaction pathway from
reactants to products.
The structure of the molecule at the transition state
is something that one can never directly observe or
isolate!! We can only speculate (or calculate from
fancy quantum mechanical programs) what it may
look like.
But once we hit the transition state and the methyl
group starts moving towards the terminal carbon
atom to form a new and stronger bond, the same
amount of energy used to move up to the top of the
activation barrier from the reactant side is released.
So there is NO NET CONSUMPTION OF
ENERGY from the activation barrier the total
amount of energy released in the reaction is still
just G.

Kinetics 6

But if the activation barrier is high enough, we


may need to add some (or a lot of) heat (energy) to
the reactants to give them enough energy to make
it over the barrier. But we get that energy back
when they go down the other side of the activation
barrier to the products!!
On the other hand, if a reaction has a very small
activation barrier, the reactants will react
extremely quickly (sometimes instantaneously) to
make products!
For endoergic reactions, the activation energy is
defined as the entire height of the barrier:

Kinetics 7

If the potential energy curve/activation barrier has


a dip in it, the reaction is proceeding through an
intermediate. An intermediate is a molecule that is
stable enough to exist for some period of time.
Unlike the transition state, which only exists for an
instant in time.

The deeper the dip or well, the more stable the


intermediate species (molecule) will be. Sometimes
the intermediate will be stable enough to isolate
and study.

Kinetics 8

Problem: Consider these potential energy diagrams:


a)

b)

c)

R
G

G
R

Rxn Coordinate

e)
P

Rxn Coordinate

G
P

Rxn Coordinate

f)

g)

G R

R
Rxn Coordinate

d)

P
Rxn Coordinate

Rxn Coordinate

h)
G R

R
P
Rxn Coordinate

P
Rxn Coordinate

Which is the:
the most endoergic reaction?
the spontaneous reaction that will go the slowest?
he reaction that will go the fastest?
the reaction that will have the most stable intermediate species?
the lowest temperature dependence?
the highest temperature dependence?
the highest activation energy (a little tricky)
the most exoergic reaction?
the reaction that is going through the most reaction steps?

Kinetics 9

A factor that helps create the activation barrier


is that when two molecules collide to react
together, they usually must:
1) collide with enough energy to react
2) collide with the proper orientation to react
3) react!
Consider the following two molecules that are
trying to react.
CO

CO
OC Mo CO

OC

no reaction!

CO
CO

OC Mo CO

OC

no reaction!

CO

CO
CO

CO

OC Mo CO

OC

no reaction!

CO
CO

OC Mo CO

OC

no reaction!

CO

CO

CO

OC Mo CO

OC

CO

OC Mo CO

OC

CO

CO

Kinetics 10

Elementary or Fundamental Reaction Steps


These are the simplest reactions that can occur and
they pretty much fall into two categories:
unimolecular and bimolecular reaction steps.
A unimolecular reaction is one in which a single
molecule does something by itself and converts into
another molecule. It is not dependent on a collision
between two molecules. For example:
CO

CO

OC Mo CO

OC Mo CO

OC

CO

OC

CO

+ CO

CO

A bimolecular reaction is one in which two


molecules collide in the proper orientation to react
together to form a new product(s).
CO

CO
OC Mo CO

OC

CO

+ PMe3

OC Mo CO

OC

CO

PMe3

Kinetics 11

Putting these two fundamental steps together


allows us to propose a mechanism for the
substitution of a carbonyl ligand on the Mo(CO)6
with a PMe3 ligand.
Overall rxn:

Proposed mechanism consistent with experimental


data & kinetic measurements:

Of these two fundamental rxn steps, #1 is the slow


or rate determining step.

Kinetics 12

Factors that affect the kinetics (rate) of a rxn:


1) The nature of the reactants and products
(thermodynamics, G)
2) Concentration rxns usually go faster when
the reactants are more concentrated
3) Temperature rxns almost always go faster at
higher temperatures
4) The presence of an effective Catalyst will speed
up a chemical reaction
5) The surface area of a solid reactant or solid
catalyst the higher the surface area the faster
the reaction (dissolving, catalysis) will occur
For chemical reactions rates are usually expressed
as:

moles liter1 sec1


or

M sec1

Kinetics 13

The rate or speed of a reaction is defined as the


change in concentration of reactants (which are
consumed) or products (which are formed) over
some period of time.
Consider the reaction of A B. Lets start with a
1.0 M solution of A, with no B initially present.
The following is a plot of the conversion of A into B
versus time.

Note that we can either monitor the production of


product or the loss of reactant in this reaction.

Kinetics 14

The rate for most reactions constantly changes


(except at the end when it is zero) during the
course of the reaction. The rate at any point in
time is calculated by taking the slope of the tangent
to the production or consumption curves.

At very early times (for example, the first 10


seconds in the rxn above), the rate of formation is
essentially linear and gives one the fastest rate of
the reaction. This is called the initial rate of
reaction.

Kinetics 15

Once the rate of product formation starts to curve


downwards, one needs to measure the rate
graphically by the tangent method or via kinetic
rate expressions (math formulas) that we will
discuss below.
If we look at the data that we plotted more
carefully we can see a trend from which we can
construct a mathematical relationship.
A (M)

rate (M sec-1)

1.00

14.0 x 10-3

0.50

7.00 x 10-3

0.25

3.50 x 10-3

0.125

1.75 x 10-3

We find that as the concentration of A doubles, the


rate of reaction doubles (or if it is cut in half, the
rate is cut in half). This leads us to the following
mathematical relationship:

rate of reaction = k[A]


This is called a kinetic rate expression.

Kinetics 16

k is a constant (it does change with temperature!)


that we call the rate constant. Using the first line
of data from the table when A = 1 M, we can easily
solve for the value of k = 0.014 sec-1. From this we
can calculate the rate of the reaction at any
concentration of A. The bigger the rate constant,
the faster the reaction (so long as there are some
reactants present).
Note that as the concentration of the reactant gets
lower, the reaction goes slower. When it gets to 0,
the reaction stops and is over.
Since there is only one reactant (raised to the first
power) in the kinetic rate expression, we refer to
this reaction as a First Order Reaction.
The units on the rate constant are set so as to
ensure that the rate of reaction is always moles
liter-1 sec-1 (or M sec-1).

Kinetics 17

First Order Reactions are fairly common for


example, all radioactive atoms decay via first order
kinetics. If we carefully examine the plot of first
order data again we can see where the term HalfLife comes from and that it applies to a reaction
with first order kinetic behavior.

A half-life (often abbreviated t ) is the amount of


time it takes for of a reactant to be converted
into product.

Kinetics 18

For a First Order rxn, the half-life is independent


of the reactant (or product) concentration and is
defined as:

t =

0.693
k

Where k is the rate constant. The unit on t is


typically seconds, the reciprocal of that on k (sec1)
While the kinetic rate expression for a first order
reaction looks quite simple, it takes calculus to
convert it into an equation that allows you to
calculate the concentration of the reactant or
product over time (as shown in the plots on the
previous pages). One also needs to determine the
rate constant k from experimental measurements.

[ A t ] = [ A0 ]e - k t
[At] is the concentration of reactant A at time t.
[A0] is the initial or starting concentration of A at
the beginning of the reaction. k is the rate
constant, and t is the time in the same units as the
rate constant.

Kinetics 19

Bimolecular reactions (such as A + B C) have


Second Order Kinetics, for which we can write the
following rate equation:

rate = k[A][B]
The total order of a reaction is defined as the sum
of the powers on the reactant concentrations in the
rate expression (equation). In this case [A] and [B]
are both raised to the 1st power, so the total order =
1 + 1 = 2. So this is a Second Order Rxn.
It is important to note that the order on each
reactant concentration (the power to which it is
raised) can be 0, fractional, or even negative!!
THE ORDER IS NOT RELATED TO THE
COEFFICENTS ON THE BALENCED
CHEMICAL EQUATION, as with equilibrium
constants!!!

IT MUST BE DETERMINED
EXPERIMENTALLY!!!
Although a bimolecular rxn is second order and a
unimolecular rxn is first order, the opposite is not
necessarily true!!

Kinetics 20

The orders of fundamental rxns, however, are


based on their coefficients. The trick is knowing
which fundamental rxn defines the rate
determining step, and then to express the overall
rate law in terms of reactants.
Problem: Consider the following reaction:

3H2 + N2

2NH3

Which of the following correctly describes the kinetic


rate expression for this reaction?
3

a) rate = k[H2][N2]
3

b) rate = k[H2] [N2]


2

c) rate = k[H2] [N2]


d) rate = k[H2][N2]
e) not enough data to determine

Problem: Consider the following bimolecular reaction:

Ni(CO)3 + CO Ni(CO)4
Which of the following correctly describes the kinetic
rate expression for this reaction?
a) rate = k[Ni(CO)3][CO]
b) rate = k[Ni(CO)3]3[CO]
c) rate = k[Ni(CO)3]

d) rate = k[CO]

e) not enough data to determine

Kinetics 21

Second order rate equations:

1
1

=kt
[ A t ] [ A0 ]
or, rearranging to solve for [A]:

[At ] =

1
k t + [A ]

The half-life for a second order reaction is defined


as:

1
t 1 /2 =
k[A 0]
Plotted below is a comparison of the first order
and second order kinetic plots, both with rate
constants set to 0.014.

Kinetics 22

Problem: Qualitatively explain why the


production of product (or consumption of
reactants is significantly slower for the second
order rxn relative to the first order rxn in the
graph above?

Kinetics 23

Example: Given the following experimental

kinetic data, determine the kinetic rate expression


and overall order of reaction.
2HgCl2 + C2O42

Hg2Cl2 + 2CO2 + 2Cl

Exp #

[HgCl2]

[C2O42-]

Initial Rate

0.10 M

0.15 M

2 x 10-5

0.10 M

0.30 M

8 x 10-5

0.05 M

0.30 M

4 x 10-5

Our tentative rate expression for this reaction is:


rate = k[HgCl2]x[C2O42]y
We need to determine what the exponents (orders)
x and y are on the reactant concentrations. Then
we can solve for the rate constant k.
Step #1: find two experiments where the
concentration of one of the reactants stays the
same. Since the rate constant k is a constant and
the concentration of one of the components is not
changing (also a constant for this comparison), we
only have to worry about the one reactant that is
changing.

Kinetics 24

We can then set up the following proportionality:


y
[Aexp#2] y
[Aexp#2]
Rate(exp #2)
=
y = [A
[Aexp#1]
Rate(exp #1)
exp#1]
8 x 10

-5

2 x 10 -5

0.30

0.15

y
(2)
4 =
If you cant solve this by inspection, then
take the log of each side of the equation:
log(4) = y log(2)
Rearrange and solve for y:
log(4)
y=
log(2)

0.602
0.301

= 2

Kinetics 25

Step #2: Repeat for the other reactant


concentration terms in the rate equation:
[Aexp#2] x
Rate(exp #2)
=
Rate(exp #3)
[Aexp#3]

8 x 10 -5
4 x 10 -5

x
2 = (2)

0.10

0.05
x=1

Now that we have solved for the orders of the


kinetic rate expression, we can write out the rate
equation:
rate = k[HgCl2][C2O42]2
The overall order of the rate expression is the sum
of the individual orders (x + y): 1 + 2 = 3. So this
is called a third order rxn or rate law.
Note that the orders DO NOT correspond to the
coefficients on the chemical equation. Many times
they do, but just as many times they wont. There
is NO FORMAL CONNECTION (except for
fundamental rxns)!!
Common Mistake !

Kinetics 26

Step # 3: Calculate the rate constant (if asked for).


We now just plug in the experimental data from
any one of the experiments and rearrange and
solve for k, the rate constant. For this example we
will use the data from experiment # 1.
rate = k[HgCl2 ] [C2 O42- ] 2
k=

k=

k=

rate
[HgCl2 ] [C2 O4 2- ] 2
2 x 10-5 M sec -1
(0.10 M) (0.15 M ) 2
2 x 10-5 M sec -1
2.25 x 10-3 M 3

-3
k = 8.88 x 10 M -2 sec -1

Kinetics 27

Example: Given the following experimental

kinetic data, determine the kinetic rate expression


and overall order of reaction.
Mo(CO)6 + PMe3

Mo(PMe3)(CO)5 + CO

Exp #

[Mo(CO)6]

[PMe3]

Initial Rate

0.10 M

0.10 M

0.3

0.10 M

0.30 M

0.3

0.30 M

0.10 M

0.9

Our tentative rate expression for this reaction is:


rate = k[Mo(CO)6]x[PMe3]y
First, lets solve for [PMe3]y:
Rate(exp #2)
Rate(exp #1)
0.3
0.3

1 = (3)

0.30

[Bexp#2] y
[Bexp#1]

0.10
y

y=0

So the order on PMe3 is zero! This means that it


does NOT appear in the kinetic rate expression!

Kinetics 28

Now, lets solve for [Mo(CO)6]x:

Rate(exp #3)

Rate(exp #1)
0.9
0.3

x
3 = (3)

[Aexp#3] x

0.30

[Aexp#1]
x

0.10
x=1

So the final kinetic rate law (expression, equation)


is:
rate = k[Mo(CO)6]
This, therefore, is a First Order Rxn.

Kinetics 29

Problem: What is the kinetic rate law for the

following reaction (figure out w, x, y and z below)


H
catalyst = OC Co
CO
H2C CH2 + C O + H2
ethylene

Exp H2C=CH2

CO

CO

O
H

C
H2

CH3

adehyde

CO

H2

Catalyst

Initial Rate

2M

0.04 M

0.06 M

0.001 M

4 104

1M

0.04 M

0.06 M

0.001 M

2 104

1M

0.02 M

0.06 M

0.001 M

4 104

1M

0.02 M

0.03 M

0.001 M

2 104

1M

0.02 M

0.03 M

0.0005 M

1 104

rate = k[H2C=CH2]w[CO]x[H2]y[Catalyst]z

Kinetics 30

Problem: Consider the following reaction and

kinetic data. What is the kinetic rate expression?


Ni(CO)4 + CNCH3

Ni(CO)3(CNCH3) + CO

Exp #

[Ni(CO)4]

[CNCH3]

Initial Rate (Msec1)

1
2
3
4

0.04
0.04
0.08
0.08

0.06
0.12
0.06
0.12

0.2 105
0.2 105
0.4 105
0.4 105

Problem: What is the rate constant k for the

following reaction:
O
C
C

O C

C
O

O
O
C

C
O

Mn

Mn
C

O
O

C O

+ PMe3

O C

O
O
C

Mn

Mn
C
O

PMe3

C
O

C O

C
O

Exp

[Mn2]

[PMe3]

Initial Rate

0.2 M

0.5 M

1 M/sec

0.4 M

0.5 M

1.4 M/sec

0.4 M

1.0 M

2.8 M/sec

+ CO

Kinetics 31

Problem: Consider the following reaction and

kinetic data. What is the rate law and rate


constant?
CO(g) + H2O(g)

CO2(g) + H2(g)

Exp #

[CO]

[H2O]

Initial Rate (M sec1)

1
2
3
4
5

1.5
1.5
3.0
1.5
3.0

1.5
4.5
4.5
0.75
3.0

0.2
0.6
1.2
0.1
0.8

Kinetics 32

Temperature, Rates & the Arrhenius Equation


One can calculate the rate constant for a reaction
using the Arrhenius equation:

k = Ae

Ea / RT

A is a constant representing the fraction of collisions


between molecules having the correct orientation to
react when the reactants have a concentration of 1 M.
It is generally not known.
But A can be factored out if one ratios the equation at
two temperatures. One typically uses this modified
Arrhenius equation to either calculate Ea values from
rate constants derived from experiments at different
temperatures, or a rate constant at a different
temperature if one has already determined Ea.

1
k 2 Ea 1

ln =

k
R
T
T
1
1
2
k2
R ln
k1
Ea =
1
1
T T
1
2

Kinetics 33

Example: Chemists have a rule of thumb that


many reactions will double their rate when the
temperature increases by 10C (or K). Calculate
the Ea that fits this doubling of reaction rate.
First step: rate and rate constant are different, but
if one assumes that one is keeping concentrations
of the reactants the same, the rate is directly
related to the rate constant. So we want to use the
Arrhenius equation with a k2/k1 ratio = 2. We will
also assume a room temperature reaction with
temperatures of 300K = T1 and 310K = T2.
k2
R ln
k1
8.314 J/molK ln ( 2 )
5.76
Ea =
=
=
1
1
1.07 10 4
1
1
T T
300 310
1

2
Ea = 53, 832 J = 53.8 kJ
Warning: watch your units!!

Kinetics 34

Problem: How much will the rate increase if a rxn


has a Ea = 70 kJ and the temperature increases

from 300K to 400K?

Problem: A reaction has k = 1.6 105 s1 at

600 K. When the temperature is increased to


700 K the new measured k = 6.36 103 s1.
Calculate the Ea value for this reaction.

Kinetics 35

Catalysis
[catalyst]

A + B

A catalyst is a substance that increases the rate of


rxn without itself being consumed in the reaction.
After the rxn is finished you should be able to
recover the catalyst from the rxn mixture
unchanged. A catalyst speeds up the rate at which
a chemical reaction reaches equilibrium. The
overall thermodynamics of the rxn is NOT changed
by the catalyst. Therefore, very endothermic (nonspontaneous) reactions are usually NOT suitable
for catalytic applications.
A catalyst provides a lower energy pathway for the
production of products from reactants, thus
allowing the rxn to proceed faster. It lowers the
activation energy for a rxn (kinetics) it does NOT
change the thermodynamics of a rxn.

Kinetics 36

Catalyzed rxn
proceeding through
an intermediate

Ea
Ea
catalyzed

G
Reactants

G
Products

Reaction Coordinate

A catalyst provides an alternate mechanism (or


pathway) for the reactants to be transformed into
products. The catalyzed mechanism has an
activation energy that is lower than the original
uncatalyzed rxn. An excellent catalyst will lower
the activation energy the most.

Kinetics 37

Problem: Which of the following represents a correct

catalyzed potential (free) energy diagram based on this


uncatalyzed energy diagram. Why?

Kinetics 38

There are two broad classes of catalysts:


Heterogeneous Catalysis: Technically speaking
this is when the catalyst is in a different phase than
the reactants and products. Practically, it is
usually when the catalyst is a solid and the
reactants and products are liquids or gases. On a
solid catalyst, only the surface of the catalyst is
where the reaction will occur so the more surface
area available the more catalysis can occur.
Homogeneous Catalysis: This is when the catalyst
is in the same phase as the reactants & products.
Practically, this is usually in the liquid or solution
phase. Homogeneous catalysts are usually
molecules dissolved in solution.

Kinetics 39

Advantages/Disadvantages of Homogeneous
Catalysts Relative to Heterogeneous Catalysts
Good homogeneous catalysts are:
good generally more selective for a single product
far more active
more easily studied
far more easily optimized
bad
far more sensitive to deactivation
for separating product & catalyst

Heterogeneous catalysts dominate chemical and


petrochemical industry ~ 95% of all chemical
processes use heterogeneous catalysts.
Homogenous catalysts are used when selectivity is
critical and product-catalyst separation problems
can be solved.

Kinetics 40

Catalysis Terminology

Turnover (TO) one loop through the catalyst


cycle. Typically one equivalent of reactant is
converted to one equivalent of product (per
equivalent of catalyst).
Turnover Frequency (TOF) or Turnover Rate the
number of passes through the catalytic cycle per
unit time (typically sec, min or hrs). This number
is usually determined by taking the # of moles of
product produced, dividing that by the # of moles
of catalyst used in the rxn, then dividing that by
the time to produce the given amount of product.
The units, therefore, are usually just time1. Note
that the rate of a batch catalytic reaction is fastest
at the very beginning when the reactant
concentration is the highest and generally slows
down as the reaction proceeds stopping when all
the reactant is used up. Note the graph below for
the production of aldehyde product from the
homogeneously catalyzed reaction of vinyl acetate,
H2, and CO.

Kinetics 41

Vinyl Acetate Hydroformylation

sampling from
autoclave causes
pressure glitches

0.3mM catalyst -- 85C/90 psi H2 /CO

2,000
1,800

4.5

Uptake curve
1,600

4
1,400

Equiv
Aldehyde

1,200

kobs = 0.0076 min

1,000

3.5

-1

Ln plot

Prod
800

Ln ( P)

2.5

600

Initial TOF
8 TO/min
476 TO/hr

400

2
1.5

200

0
0

10

15

20

Time (hours)

The TOF, therefore, will vary throughout the


course of a batch reaction. The Initial TOF is
defined as the initial part of a catalytic reaction
where the rate is the fastest and essentially linear.
A far better measure of rate, however, is the
observed rate constant kobs, which allows one to
reproduce the entire product production curve
given a set of reactant & catalyst concentrations.
In the above graph, the reaction is pseudo-first
order in excess reactant alkene (vinyl acetate
concentration ~ 0.6 M, catalyst 0.3 mM) and kobs
is determined from a ln plot of the change in
H2/CO pressure (the reactant concentration)
versus time for this rxn.

Kinetics 42

Turnover Number (TON) the absolute number of


passes through the catalytic cycle before the
catalyst becomes deactivated. Academic chemists
sometimes report only the turnover number when
the catalyst is very slow (they dont want to be
embarrassed by reporting a very low TOF), or
decomposes quite rapidly. Industrial chemists are
interested in both TON and TOF. A large TON
(e.g., 106 - 1010) indicates a stable, very long-lived
catalyst.

Kinetics 43

Heterogeneous Catalyst Examples


Heterogeneous catalysts are often very small
particles of inert support (alumina, silica, MgO)
coated with a very thin layer of catalytically active
metal or alloy (mixture of metals). Remember that
the catalysis only occurs on the surface of a
heterogeneous catalyst.
Hydrogenation of CO to produce CH3OH:
Cu/ZnO
CO(g) + H2(g)

CH3OH(g, l)

This is one of the most selective heterogeneous


catalytic reactions known since methanol is almost
the only product formed. Methanol has many
important uses in the chemical industry. It can
also be used as a very clean fuel.

Kinetics 44

Fischer-Tropsch Catalysis:
Fe
xCO(g) + yH2(g)
CH4(g) + CH3(CH2)zCH3(g,l) + H2O(g)
z = 0 to 20+

GASOLINE (z = 6-10)!!
This very important catalytic process was
developed in Germany during the 1930's and made
WWII possible by allowing the Germans to
convert coal into gasoline. New Zealand and South
Africa both have Fisher-Tropsch plants to turn
coal into hydrocarbon mixtures.
Zeolites
Currently there is a great deal of interest in a very
important class of naturally occurring aluminosilicates called Zeolites, which have the general
formula:
Mx/nn+[AlxSiyO2x+2y]-x

Kinetics 45

Zeolites have porous structures that have tunnels


and chambers in which catalytic reactions can
occur:

Red = O
Yellow = Si
Blue = Al
Purple = Na+

ZSM-5

Kinetics 46

Zeolites are very popular because they do a very


good job at catalysis. Consider the following rxn:
ZSM-5
CH3OH(l)

"Gasoline" + H2O(l)

Unlike the Fisher-Tropsch catalysis discussed


earlier which gave a wide variety of products
which had to be separated apart (expensive!), the
zeolite ZSM-5 developed by Mobil produces a
gasoline blend of hydrocarbons quite selectively
and under mild conditions (sorta cheap).
Why?? The catalytic rxns are going on inside the
chambers in the zeolite which limits the size of the
products that can be produced!

Kinetics 47

Metal-Based Homogeneous Catalysis

Hydroformylation (also called the oxo rxn) is a


catalytic rxn that takes organic compounds with
C=C double bonds (alkenes or olefins), reacts them
with H2 and CO and turns them into new, more
useful organic compounds called aldehydes:
O

O
H2 C=CHR

H 2 CO
Rh or Co
catalyst

H-C-CH2 CH2R

linear

H 3 C-CHR
branched

Aldehydes

* Largest homogeneous catalytic process


* > 12 billion pounds of aldehydes (alcohols) per year
* Selectivity to linear (normal) or branched (iso)
products is important

Otto Roelen discovered hydroformylation in 1938.


He was actually studying the heterogeneously
catalyzed Fischer-Tropsch rxn using cobalt metal
and found that the cobalt metal was dissolving into
solution to produce a homogeneous catalyst that
did hydroformylation.

Kinetics 48

Roelen and co-workers figured out that HCo(CO)4


was the active catalytic species, but did not understand how it worked. In 1960 and 1961 Heck and
Breslow proposed what is now accepted as the
general mechanism for hydroformylation:
Scheme 1. Heck/Breslow Hydroformylation Mechanism
O
C
+ H2

OC

CO

Co

Co2 (CO) 8

C
O

+ CO

- CO

+ alkene
+ CO
R

Co2 (CO) 7

H
OC

Co

OC

+ H2

HCo(CO) 4

Co
R

C
O

- CO

CO

OC

C
O
rate
determining
step

+ CO

anti-Markovnikov
alkene addition
to M-H bond

- CO
proposed bimetallic
pathway -- since shown
to be of no importance
under normal catalytic
conditions

O
C

O
C
+ CO

CO

OC

Co
C
O

CO

OC

Co
O

2.5 atm -- 1.6:1 linear to branched


90 atm -- 4.4:1 linear to branched

C
O

increasing CO partial pressure keeps back rxns from


occuring -- this limits alkene isomerization and the
corresponding opportunity for making branched aldehyde

Kinetics 49

The kinetic rate expression for this rxn is:

d ( aldehyde)
= k [ alkene][ Co][ H2][ CO] 1
dt
The first hydroformylation plant in the USA was
build in 1947 in Baton Rouge by what is now called
Exxon Chemicals. It is located just north of the
state capital and still uses the HCo(CO)4
technology.
In the late 1960s it was discovered that Rh was a
far more active catalyst for hydroformylation and
when combined with PPh3 (triphenylphosphine) a
very selective catalyst for making linear aldehyde
product was produced. The first Rh/PPh3 hydroformylation plant was build in the early 1970s and
currently about 70-75% of all hydroformylation
plants use the Rh/PPh3 technology (or closely
related variant).

Kinetics 50

Problem: Consider the reaction: F + G

and the following experimental data:


Exp #

[F]

[G]

Rate (Msec1)

1.0

1.0

1.0

2.0

2.0

2.0

What is the kinetic rate expression for this


reaction?
a) rate = k[F]
b) rate = k[G]
c) rate = k[F][G]
d) rate = k[F]2[G]
e) rate = k[G]2
f) not enough information to determine

Chapters 17 & 20-1 to 20-3

Chemical
Equilibria

Equilibrium 2

Chemical Equilibrium: It is the condition of a


chemical reaction in which the rate of formation of
products (from reactants) equals the rate of
formation of the reactants (from products).
rate1
A + B

C + D
rate-1

Equilibrium occurs when rate1 = rate-1.


Although chemists usually want reactions to go
completely to products (and ideally only to a single
product), many do not. Theoretically all reactions
are in equilibrium.
A reaction will not generally reach equilibrium if:
1) The rxn is very exothermic (exoergic)
2) One (or more) of the products (or reactants)
is removed from the rxn
3) One of the products (or reactants) is
insoluble

Equilibrium 3

Consider the very important industrial reaction (the


Haber process) of nitrogen and hydrogen to produce
ammonia, which is used as a fertilizer:
N2(g) + 3H2(g)

2NH3(g)

This is a very difficult reaction (large activation barrier) that


requires high temperatures, pressures and a catalyst. At the
high temperatures required to make the reaction proceed at a
reasonable rate, the thermodynamics favors the N2 + H2
reactants producing the following behavior:

H2
P
NH3
N2

Time

Note that the rxn does not go to completion, rather


the forward and backward rxns reach a state of
chemical equilibrium.

Equilibrium 4

Equilibrium is a dynamic process. This means that when a


reaction has reached a state of equilibrium, the forward and
backward reactions making up the overall reaction have not
stopped!! The equilibrium definition states that equilibrium is
reached when the forward and backward reaction rates
become equal!
For example consider a saturated solution of NaCl (no
additional salt will dissolve):

NaCl(s)

Na+(aq) + Cl(aq)

If we add 5 g more NaCl(s) to this solution, the amount of solid


NaCl in the container will increase by 5 g (that is, no additional
solid NaCl will dissolve into solution). This does not mean,
however that some of the new NaCl that we just added won't
dissolve at all. Some of it will dissolve, while some Na+(aq) +
Cl(aq) in solution elsewhere will precipitate out! We could
follow this by adding radioactive 24Na38Cl to the container:
radioactive salt

TIME

Equilibrium 5

Demonstration:

Equilibrium 6

Law of Mass Action


One can set up a general mathematical expression to
describe the following chemical equilibria:

wA + x B

yC + z D

y
[C] [D]z
Keq =
[A]w [B]x

products
reactants

Keq is called the equilibrium constant


The equilibrium expression for the Haber process
reaction would be written as:

N2 (g) + 3H2 (g)

Keq

[NH3 ] 2

2NH 3 (g)
products

[N2 ] [H2 ]3 reactants

Equilibrium 7

Consider, for example, the equilibrium between


N2O4(g) and NO2(g):
N2O4(g)

2NO2(g)

[NO2]2
Keq =
[N2O4 ]
Listed below is experimental data giving initial
concentrations for N2O4(g) and NO2(g). After some
time the reaction reaches equilibrium and the
concentrations listed.
Initial

@ Equilibrium

N2O4

NO2

N2O4

NO2

Keq

0.00

0.02

0.0014

0.017

0.21

0.00

0.03

0.0028

0.024

0.21

0.00

0.04

0.0045

0.031

0.21

0.02

0.00

0.0045

0.031

0.21

Note how Keq is the same regardless of the initial


concentrations. This is why it is called the
equilibrium constant.

Equilibrium 8

Some Features of Equilibrium Constants


9 Keq usually depends on temperature
9 If one reverses the way a reaction is written the
new Keq is the inverse of the original value:
H2 (g) + O2 (g)
K eq =

H2O(g)

[H2 O]

= 10 (@ high temp)

[H2] [O2 ]

Reversing the above rxn we now write:


H2O(g)

*
K eq

H2(g) + O2 (g)
[H2] [O2 ]
[H2 O]

1
= 0.1 =

K eq

9 Multiplying a reaction by a constant factor


results in raising Keq to that power:
2H2 (g) + O2 (g)
#
K eq =

[H2O] 2
[H2 ]2[O2 ]

2H2O(g)
= 100 (@ high temp)

Equilibrium 9

Equilibrium constants have a number of very


important functions:
1) whether a rxn will be spontaneous under a given
set of conditions (equilibrium constants are
directly related to G Gibbs Free Energy, see
end of this chapter)
2) in which direction a reaction is going to proceed
to reach equilibrium
3) allow us to calculate the concentrations of
products and reactants at equilibrium
Qualitatively, the magnitude of Keq should
immediately tell you in what direction a reaction is
going to proceed and how far it will go before
reaching equilibrium. For example:
Keq >> 1 reaction will go mainly to products
Keq ~ 1

reaction will produce roughly equal


amounts of product and reactant

Keq << 1 reaction will go mainly to reactants

Equilibrium 10

Problem: Will the following equilibria proceed


mainly to products, reactants or produce an
approximately equal amount of both?
a) N2(g) + 3H2(g)
b) 2SO3(g)

2NH3(g)
2SO2(g) + O2(g)

c) 2HBr(g) + Cl2(g)

Keq = 0.001
Keq = 2

2HCl(g) + Br2(g)
Keq = 104

d) 2H2O(g)

2H2(g) + O2(g)

Keq = 10-28

e) N2O4(g)

2NO2(g)

Keq = 1

f) H2(g) + Cl2(g)
g) PCl3(sol) + Cl2(sol)
h) 2HCl(g) + Br2(g)

2HCl(g)

Keq = 1044

PCl5(sol)
Keq = 0.1
2HBr(g) + Cl2(g)
Keq = 10-4

Equilibrium 11

Units on Keq
There are typically no units on equilibrium
constants. This is because one is formally supposed
to use the activities of compounds instead of their
concentrations. The activity of a compound in an
ideal mixture is the ratio of its concentration (or
partial pressure) to a standard concentration (or
pressure). Since the activity is defined as a ratio, the
units cancel out.
Despite the fact that we should use dimensionless activities,
most chemists still refer to equilibrium concentrations in terms
of M or pressure units (atm).
One does need to watch out when one is working with gases.
An equilibrium constant calculated with gas pressures (atm is
the standard unit for gases) will not have the same numerical
value as one calculated using molarity values if there are
different # of gas molecules on each side of the equilibrium.
This is because of the relationship between pressure and
molarity as defined by the ideal gas law.
Chemists use various subscripts on the equilibrium constant K
to indicate different types of equilibria: Kp = gases (pressure),
Kc = solutions (molarity), Ka = acids, Kb = bases, Ksp =
solubility product (slightly soluble solids).

Equilibrium 12

Reaction Quotient
If a reaction is at equilibrium, then the equilibrium
relationship will hold true:

wA + x B

yC + z D

y
z
[C] [D]
Keq =
[A] w [B]x
But, what if one is NOT at equilibrium? Then we
find that the equilibrium expression is redefined as
the reaction quotient, Q:
[C] y [D]z
=
Keq =
w
x
[A] [B]
this will not be
equal to Keq if the
reaction is not at
equilibrium

Q
when this occurs, the
equilibrium expression
is defined as being equal
to Q, the Reaction Quotient

Equilibrium 13

By comparing Q to Keq, one can tell in which


direction a reaction will go to reach a state of
equilibrium:

wA + x B

Keq ?
=
Q > Keq

yC + z D

More products?
y
[C] [D]z
=
Q
[A] w [B]x
More reactants?
reverse reaction will be
spontaneous

Q = Keq

reaction @ equilibrium

Q < Keq

forward reaction will be


spontaneous

Equilibrium 14

EXAMPLES:
CO2(g) + H2(g)

CO(g) + H2O(g)

[CO][H2O]
= Keq = 4.4
[CO2][H2]
If the initial concentrations of all species are 1 M,
which way will the reaction proceed to reach
equilibrium?

[CO] [H2O]
[CO2 ] [H2 ]

(1) (1)
=

(1) (1)

= 1 = Q

K eq = 4.4
Q = 1
Q < Keq

therefore, the rxn will


go FORWARD to reach
equilibrium

Equilibrium 15

What if we increase the [CO] concentration to


10 M?

[CO] [H2O]
[CO2 ] [H2 ]

(10) (1)
=

(1) (1)

= 10 = Q

K eq = 4.4
Q = 10
Q > Keq

} therefore, the rxn will


go BACKWARD to reach
equilibrium

If you are ever given a problem where the product


and reactant concentrations are all non-zero, you
MUST calculate Q and compare it to Keq in order to
figure out which way the reaction has to go to reach
equilibrium.
DANGER!!
Common
mistake!!

Equilibrium 16

Problem: Are the following rxns @ equilibrium?


If not, which way must they proceed to reach a state
of equilibrium?
a) CO(g) + 2H2(g)
5M
1M

b) CO2(g) + H2(g)
1M
2M

c)

Br2(g) + I2(g)
0.2 M
0.2 M

d) N2(g) + O2(g)
2M
0.0001 M

CH3OH(g)
5M

Keq = 1

H2O(g) + CO(g)
2M
3M

Keq = 4

2BrI(g)
0.1 M

Keq = 1 x 104

2NO(g)
2M

Keq = 9

Equilibrium 17

Numerical Problems -- Using Keq


EXAMPLE: Consider the following rxn. The initial
concentrations are [I2] = [H2] = 2M, [HI] = 0M, and Keq = 16.
What will be the various concentrations when the reaction
reaches equilibrium?

I2(g) + H2(g)

2HI(g)

Step 1: Write out your initial and @ equilibrium conditions:


Initial cond:

[I2] = [H2] = 2M
[HI] = 0M

note that since [HI] = 0M, the reaction must proceed to the
right to make more product. Thus, for this example, we will
lose reactants and gain product. What we don't know is how
much. Therefore, we will setup an algebraic expression to
solve for x, the amount of product being produced and the
amounts of reactant that we are losing. It is CRITICAL to
remember to multiply x by the appropriate coefficient from
the balanced chemical equation.
@ Equilibrium:
[I2] = [H2] = (initial conc.) (coefficent)(x)
= 2x
[HI] = (initial conc.) + (coefficent)(x) DANGER!!
Common
= 0 + 2x
= 2x
mistake!!

Equilibrium 18

Step 2: Write out your equilibrium expression:

[HI] 2
Keq =

= 16
[H2] [I2]

now substitute in the @equilibrium conditions:


[HI] 2
[H2] [I2]

(2x)2
= 16

=
(2-x) (2-x)

now solve for x:


(2x)2

(2-x) (2-x)

(2x)2
(2-x) 2

= 16
DANGER!!
Common mistake!!

make sure that you don't miss


common algebraic simplifications!!

take the square root of each side:


(2x)2
(2-x) 2

(2x)
=

16
(2-x)

2x = (4) (2-x)
6x = 8

= 4

2x = 8 - 4x
x = 8/6

x = 1.33

Equilibrium 19

Step 3: Substitute the value for x that you solved back into the
equilibrium conditions that you wrote out in Step 1
above:
@ Equilbrium: [I2] = [H2] = 2 - x = 2 - 1.33 = 0.66 M
[HI] = 2x = 2.66 M
[I2] = [H2] = 0.66 M
[HI] = 2.66 M
Step 4: Substitute the equilibrium concentrations you just found
back into the equilibrium expression to see if you
calculate the correct value for Keq:

[HI] 2

Keq =

[H2] [I2]

= 16

substitute in the calculated equilibrium


concentrations and see if you get K eq
(2.66) 2

[HI] 2

= 16

=
[H2] [I2]

(0.66) (0.66)

Step 5: Carefully read the question and make


sure that you are picking the right
answer. Note that what you solve for x
may not be the answer (make sure you
do Step 3!)!!

DANGER!!
Common
mistake!!

Equilibrium 20

Problem:
Starting with [CO2] = 2 M, [H2] = 2 M, [CO] = 0 M and [H2O]
= 0 M, what will be the various concentrations @ equilibrium.
Keq = 9.

CO2(g) + H2(g)

CO(g) + H2O(g)

Equilibrium 21

More Difficult EXAMPLE: Calculate Keq for the following


reaction. Initial concentrations are: [SO2] = 4 M, [O2] = 4 M,
[SO3] = 6 M. At equilibrium [SO2] = 3 M.

2SO2(g) + O2(g)

2SO3(g)

Solution:
This is really just a stoichiometry problem. Given the initial
concentrations and a single equilibrium concentration (along
with some algebra) one can solve for the other equilibrium
concentrations. Once you have obtained all the equilibrium
concentrations, one can put them into the equilibrium
expression to solve for Keq.
First one must figure out which way the reaction is going to
go in order to reach equilibrium. We can't use the Reaction
Quotient, Q, because we don't know Keq. We do, however, have
our initial and one final equilibrium condition to tell us which
way the reaction will shift: initial [SO2] = 4 M, @ equilib
[SO2] = 3 M. So we are losing [SO2], therefore, the reaction
will go to make more product and to lose reactant.
Now we can setup and solve for the other equilib values:

Equilibrium 22

initial:

2SO2 (g) + O2 (g)


4-2x

@ equilib:

2SO3(g)

4-x

6+2x

Normally we would substitute our x values into the


equilibrium expression and solve for x. Here, however,
we actually know one of the equilibrium values:

@ equilib:
(given)

So we can let 4-2x = 3 and solve for x:

4-2x = 3

2x = 1

x = 0.5

Now, substitute x into our @ equilibrium formulas:

@ equilib:

4-2x

4-x

6+2x

3M

3.5 M

7M

Finally, substitute the final values into the equilibrium


expression to solve for Keq :

2SO2 (g) + O2 (g)


[SO3 ] 2
Keq =

[SO2 ]2[O2 ]

2SO3(g)
(7) 2
=

(3)2(3.5)

= 1.6

Equilibrium 23

Problem:
Consider the following reaction:

CO(g) + 2H2(g)

CH3OH(g)

Initially we start with [CO] = 10 M and [H2] = 11 M. When the


reaction reaches equilibrium there is 5 M [CH3OH]. Calculate
Keq for this reaction.

Equilibrium 24

As Tough as We Get EXAMPLE: Calculate equilibrium

concentrations for the following reaction. Initial values are:


[CO2] = 1 M, [H2] = 2 M, [CO] = 6 M, and [H2O] = 6 M.
Keq = 2.

CO2(g) + H2(g)

CO(g) + H2O(g)

Solution:
First we need to determine in which direction the reaction will
shift to reach equilibrium because none of the concentrations
are zero. To do this we use the Reaction Quotient, Q, and the
initial concentrations:
[CO] [H2O]
(6) (6)
=
= 18 = Q
[CO2] [H2]
(1) (2)
K eq = 2
Q = 18
Q > Keq

therefore, the rxn will


go BACKWARDS to reach
equilibrium

Now we can write out our initial and (most importantly) @


equilibrium values using x's:
Initial cond:

[CO2] = 1 M
[CO] = 6 M

@ Equilibrium: [CO2] = 1 + x
[CO] = 6 - x

[H2] = 2 M
[H2O] = 6 M
[H2] = 2 + x
[H2O] = 6 - x

Equilibrium 25

K eq =

(6-x) (6-x)

[CO] [H2O]

[CO2] [H2]

(1+x) (2+x)

= 2

(x2 - 12x + 36)


= 2

(x2 + 3x + 2)

x 2 - 12x + 36 = 2x 2 + 6x + 4
x 2 + 18x - 32 = 0

} this is a quadratic equation


solve by using quadratic formula:

ax 2 bx c = 0
x=

x=

x=

- 18

(18) 2 - 4(-32)
2

- 18
2

452

b 2 - 4ac
2a

-b

- 18

- 18 21.3
2
x = 1.7

Note that x is NOT OUR ANSWER!!!!

(324) + (128)
2

= 1.7 or -19.7
physically
impossible

DANGER!!
Common
mistake!!

Equilibrium 26

@ Equilibrium: [CO2] = 1 + x
[CO] = 6 - x

[H2] = 2 + x
[H2O] = 6 - x

Substituting in x = 1.7 we can get the equilibrium values:


@ Equilibrium: [CO2] = 2.7 M [H2] = 3.7 M
[CO] = 4.3 M
[H2O] = 4.3 M
Double-check that these numbers are correct by recalculating
Keq and comparing to the value given to you in the problem:

[CO][H2O] (4.3)(4.3)
=
= 1.85 2
[CO2][H2] (2.7)(3.7)
You dont get exactly 2.0 due to round-off error (I only carried
one decimal point in my calculation)

Equilibrium 27

A Simple EXAMPLE (but looks really hard if you don't


think): If 2 moles of H2O are placed into a 5L container, what
will be the equilibrium concentration of H2, O2 and H2O?
2H2O(g)

2H2(g) + O2(g)

Kc = 6.0 10-28

[H2]2[O2]
28
=
6.0

10
Kc =
[H2O]2
initial cond:

[H2O] = 2.0 moles/5 L = 0.40 M


[H2] = [O2] = 0 M

[H2] = 2x
[O2] = x
[H2O] = 0.40 - 2x
substituting into our equilibrium expression we get:
@ equilibrium:

[2 x ]2[ x ]
28
6
.
0
10
=

[0.4 - 2 x ]2
4 x 3 24.0 10 28 x 2 + 9.6 10 28 x 0.96 10 28 = 0
But this is a cubic equation!!! Almost impossible for
you to solve!!!
OH MY GOD, WHAT DO I DO NOW!!
What is this idiot Professor doing to me!!

Equilibrium 28

HOWEVER, consider the physical reality of the situation.


Kc = 6.0 10-28 is extremely small, this means that very little
H2O will decompose to form H2 or O2!! That means that the
amount of H2 or O2 forming will be very, very small. That
means that x will be very, very small. Small enough that we can
ignore it in the [H2O] = 0.40 2x expression. This
considerably simplifies the math:

[2 x ]2[ x ]
28
6.0
10
=

[0.4]2
4x 3
= 6.0 10 28
0.16
x 3 = 0.24 10 28
x = 2.9 10 10 M
Since x is indeed much, much smaller than 0.40, the
approximation was a very good one. So our concentrations at
equilibrium are:
@ equilibrium:
[H2] = 5.8 10-10 M
[O2] = 2.9 10-10 M
[H2O] = 0.40 M
Actually, one didn't have to do any calculations for this problem!
Because Kc is so very small, you should know that virtually no
products will be produced. Therefore:
@ equilibrium:
[H2] = [O2] = 0 M
[H2O] = 0.40 M
For this course and this kind of problem, there isn't much
difference between 5.8 10-10 M and 0 M!!

Equilibrium 29

Heterogeneous Equilibria
So far all the equilibrium examples we have used
have involved gases or solutions. What happens if
we have other states of matter present -- such as
solids or pure liquids?? How do they affect the
equilibrium??
It turns out that as long as some solid or liquid is
present, the equilibrium will be independent of the
amount of that solid or liquid that is present!
EXAMPLE:
Br2(l)
Br2(g)
Keq =

[Br2(g)]
[Br2(l)]

What is the concentration of [Br2(l)]?


M=

moles of Br2(l)
volume

The density of Br2(l) is 3.12 g/mL, so the # of moles is:


# moles = (3.12 g/mL) / (159.8 g/mol)
# moles = 0.0195

Equilibrium 30

The M can now be calculated for liquid bromine:


M=

moles of Br2(l)
volume

0.0195 moles of Br2(l)


0.001 L

M = 19.5 mol/L
Now we can look at our equilibrium expression:
Keq =

[Br2(g)]
[Br2(l)]

[Br2(g)]
19.5 M

note that this is a constant


concentration -- it is
independent of the amount
of liquid bromine present,
as long as some is there!!

Because the [Br2(l)] concentration is a constant value


we can multiply the equilibrium expression by that amount
and incorporate it into Keq :
(Keq ) (19.5 M) = [Br2(g)]

*
K eq

= [Br2(g)]

DANGER!!
VERY Common
mistake!!

Therefore, one should NOT include solids or


pure liquids in equilibrium expressions!!
A more technical, but simpler explaination is that we are
actually using activities instead of concentrations (see section
on units), and the activity of a solid or pure liquid is defined as
being = 1. Thus it factors out of the equilibrium expression.

Equilibrium 31

Problem: Write out equilibrium expressions for


the following reactions:
a) CaO(s) + CO2(g)

CaCO3(s)

b) Ag+(aq) + Cl(aq)

AgCl(s)

c) Br2(l) + Ni(CO)4(l)

d) H2CO3(aq)

e) HCl(g) + H2O(l)

NiBr2(s) + 4CO(g)

H2O(l) + CO2(g)

H+(aq) + Cl(aq)

Equilibrium 32

Numerical Example: What is the equilibrium

concentration for [Ag+] in the following reaction:


Ag2(SO4) (s)

2Ag+(aq) + SO42(aq)

Ksp = 4 109

Answer: Ksp refers to equilibria involving solubility products, that is,

solids that are slightly soluble in water (or other solvents). Note that
the reactant in this problem is a solid and, as such, will NOT appear
in the final equilibrium expression. We also usually do NOT give the
amount of the solid and assume that there is excess present, since only
a little bit will dissolve in solution. Otherwise, set it up and solve just
like a regular equilibrium problem:
Initial:

excess solid
Ag2(SO4) (s)

@Equilib:

0M

0M

2Ag+(aq) + SO42(aq)

less excess solid

2x

The equilibrium expression for this rxn is:


Ksp = [Ag+]2[SO42]
(the solid Ag2SO4 doesnt appear in the equilibrium expression
because it is a solid!). Plug in the @equilb values and solve for x:
[2x]2[x] = 4 10 9
4x3 = 4 109
x3 = 1 109
x = 1 103
DANGER!!

BUT, watch out, x is NOT our answer!!


[Ag+] = 2x, so [Ag+] = 2 103 M.

Common
mistake!!

Equilibrium 33

Problems:
a) What is the equilibrium concentration for OH
in the following reaction:
Ca(OH)2(s)

Ca2+(aq) + 2OH(aq)

Ksp = 4 106

b) What is the equilibrium concentration for Ag+ for


the following system:
Ag3(AsO4)(s)

3Ag+(aq) + AsO43(aq) Ksp = 1 1022

Equilibrium 34

Kp Kc Relationship
When the number of equivalents of gas phase
reactants and products is not equal the following
relationship relates Kc (concentration in M) and Kp
(concentration in pressures - atm). This is true even
if we technically use dimensionless activities due to
the relationship between molarity and pressure
(even when units are factored out).
Kp = Kc(RT)n
Kc = Kp(RT)n

Kp
Kc =
-or(RT)n

n = (ngas prod) (ngas react)


No gas molecules? Then n = 0.
Equal number of gas molecules on reactant &
product side? Then n = 0.
Pressures must be expressed in atmospheres (atm).

Equilibrium 35

Le Chatelier's Principle
When a system in a state of equilibrium is acted
upon by some outside stress, the system will, if
possible, shift to a new equilibrium position to
oppose the effect of the stress.
What do we mean by "stress"? Stress means that
we are disturbing the reaction by: adding or
removing reactants or products; increasing or
decreasing the temperature; and increasing or
decreasing the pressure (if gases are involved).
Once we do one of these things, the reaction will
(usually) no longer be in equilibrium and will have
to shift to make more reactants or products to
reattain a state of chemical equilibrium.

A + B

original rxn in
equilibrium

C + D
+ C but now we have added

the reaction has to shift


backwards to consume
some of the products and
make more reactants!

more product C and there


is too much product

Equilibrium 36

Concept Demonstration:
Reactants

Keq > 1
G rxn < 0

Products

Energy

reaction coordinate

Chemical Demonstration:
2

CoCl4 (sol) + xH2 O

2+

Co(H 2 O) 6 (sol) + 4Cl (sol)

H2O
H 2O

Co

H 2O

O2H

OH2
H2O

Equilibrium 37

Listed below are how various "disturbances" affect


equilibria:
1) Adding products (unless one of the products is a
solid!) to a reaction will cause the equilibrium to
shift back to produce more reactants.
2) Adding reactants (unless one of the reactants is a
solid!) to a reaction will cause the equilibrium to
shift forward to produce more products.
3) Removing reactants (unless one of the reactants
is a solid and as long as there is some left) will
cause the equilibrium to shift back to produce
more reactants.
4) Removing products (unless one of the products is
a solid and as long as there is some left) will
cause the equilibrium to shift forward to
produce more products.

Equilibrium 38

5) The effect of temperature on a reaction is


dependent on whether the reaction is exothermic
(Hrxn = negative) or endothermic (Hrxn =
positive):
Exothermic rxn:
A + B

C + heat

heat is one of
the products

a) increasing the temperature (adding heat)


will cause the equilibrium to shift back
to make more reactants
b) decreasing the temperature (removing heat)
will cause the equilibrium to shift forward
to make more products
Endothermic rxn:
heat is one of
the reactants
A + B + heat

a) increasing the temperature (adding heat)


will cause the equilibrium to shift forward
to make more products
b) decreasing the temperature (removing heat)
will cause the equilibrium to shift back to
make more reactants

Equilibrium 39

6) The effect of changing pressure depends on the


number of gasous reactants and products
present:
a) if there are NO gas phase species present
then pressure will have NO effect on the
equilibrium (actually, there is an effect at very
high pressures -- but we won't worry about this).
b) if there are gas phase species present, but
there are the same number of gaseous molecules
on each side of the reaction, pressure will have
NO effect on the equilibrium.
c) if there are different numbers of gas phase
species present on the reactant and product sides
of the equilibrium, then:
i) increasing the pressure will favor the side
of the equilibrium with the smaller number
of gas phase molecules.
ii) decreasing the pressure will favor the side
of the equilibrium with the larger number of
gas phase molecules.

Equilibrium 40

Qualitative EXAMPLE:

Consider the following equilibrium:


N2O4(g)

original
pressure

Original
pressure

Twice
original
pressure

2NO2(g)

Note how the N2 O4 molecules have


dissociated in order to produce more
NO2 molecules. These more efficiently
fill up the "empty" space present in
this system.

= N2O4
= NO 2

Note how the NO2 molecules have


combined in order to produce fewer
N2 O4 molecules. These more efficently
save space, which is in short supply in
this compressed system.

Equilibrium 41

Mathematical EXAMPLE: Effect of doubling the pressure

(halving the volume) on N2O4(g)

original
equilib

N2 O4

2NO2(g)

2NO2

Keq = 0.21

0.031 M

0.0045 M

compress
N2 O4
0.009 M

2NO2
0.062 M

is this in
equilibrium?

Calculate Q the reaction quotient to determine the direction of


the reaction to reach equilibrium:

[NO 2]2 (0.062)2


Q=
=
= 0.427
[N2O4] (0.009)
since Q > Keq the rxn has to go backwards to reach
equilibrium. That means that some of the NO2 has to
disappear.
@ equilibrium: [N2O4] = 0.009 + x } we are gaining N2O4
[NO2] = 0.062 2x } we are losing NO2
substituting [N2O4] = 0.009 + x and [NO2] = 0.062 2x into
our equilibrium expression we can solve for x:

(0.062 2 x )2
= 0.21
0.009 + x
4 x 2 0.458 x + 0.0019 = 0

Equilibrium 42

[solve using quadratic equation]


x = 0.004, 0.106
x = 0.106 M is physically unreasonable (that would give us a
negative concentration, which is impossible), we can forget it.
So x = 0.004 M. Substituting this back into our equilibrium
conditions we can find the final equilibrium concentrations:
[N2O4] = 0.009 + x = 0.013 M
[NO2] = 0.062 - 2x = 0.054 M

So the N2O4 concentration has increased and the


NO2 concentration has decreased: exactly what one
would qualitatively predict from Le Chatilier's
principle!!

Equilibrium 43

Problem: What are the various things that one can


do to the following reactions to shift the equilibria to
a) favor the reactants; b) favor the products? (you
can add or remove products and reactants; change
the temperature; change the pressure)
a) CaO(s) + CO2(g)

CaCO3(s)
Hrxn = - 179 kJ/mol

b) N2(g) + 3H2(g)

2NH3(g)

c) Ca+2(aq) + 2Cl(aq)

CaCl2(s)
Hrxn = + 95 kJ/mol

d) H2(g) + I2(g)
e) Br2(l) + Ni(CO)4(l)
f) H2CO3(aq)
g) Ag+(aq) + Cl(aq)

2HI(g)
NiBr2(s) + 4CO(g)
H2O(l) + CO2(g)
AgCl(s)

Equilibrium 44

Le Chatelier's Principle II: Common Ion Effect


The common ion effect is Le Chateliers principle
just under a different name. You will see another
variant of this at the end of the Acid/Base chapter
when we discuss Buffer solutions.
Consider the following equilibrium:
AgCl(s)

Ag+(aq) + Cl(aq)
Ksp = 1.8 1010

What happens to the Ag+(aq) concentration if we


add enough NaCl to raise the Cl(aq) concentration
to 0.1 M?
Qualitatively, of course, from Le Chateliers
principle, adding product (Cl) to the solution will
push the equilibrium backwards to produce more
reactant (AgCl(s)). This will decrease the free
Ag+(aq) concentration in solution. The Na+ cations
will not have any effect, so we can pretty much
ignore them (spectator ions).
In this case the Cl anion is the Common Ion
between the NaCl and AgCl.

Equilibrium 45

Lets set this up and mathematically solve for the


concentration of Ag+ after adding 0.1 M Cl.
Initial:

solid

AgCl(s)
@ equilib:

0.1

Ag+(aq) + Cl(aq)

less solid (-x)

0.1 + x

Ksp = [Ag+][Cl] = 1.8 1010


(x)(x + 0.1) = 1.8 1010
x2 + 0.1x 1.8 1010 = 0
solve via the quadratic equation to get:
x = 1.8 x 109 -- or -- 0.1000000003 Physically
x = [Ag+] = 1.8 109 M

impossible!!

Equilibrium 46

But, there is a simple approximation we can use to


make our math a lot easier. Before we add any extra
Cl, let's show that the concentrations of [Ag+] and
[Cl] present are very small:
Initial:

solid

AgCl(s)
@ equilib:

less solid (-x)

Ag+(aq) + Cl(aq)
x

Ksp = [Ag+][Cl] = 1.8 1010


(x)(x) = 1.8 1010
x 2 = 1.8 1010
x = [Ag+] = [Cl] = 1.3 x 105 M

Concentrations
with no extra
Cl- anion
added to the
solution!

So the Ag+ and Cl concentrations in solution from


AgCl(s) are 1.3 x 105 M. Le Chateliers principle
tells us that adding more Cl will decrease the Ag+
and Cl (x values) from the AgCl(s) dissociation
even further. Small enough that we can make the
approximation that 0.1 + x in the original problem is
essentially 0.1. This will reduce our quadratic
expression down to a very simple algebra problem:

Equilibrium 47

Initial:

solid
AgCl(s)

@ equilib:

Ag+(aq) + Cl(aq)

less solid (- x)

Ksp = [Ag+][Cl] = 1.8 1010


(x)(0.1) = 1.8 1010
divide through by 0.1

0.1

Approximation!

0.1 + x
Simplify to
just 0.1
because x is
very small

x = [Ag+] = 1.8 109 M


Note that this is the same as what we calculated
from the quadratic equation ([Ag+] = 1.8 109 M).
And it is a LOT quicker and easier to calculate!
So using this approximation, when appropriate, will
save you a lot of time. Typically it is OK to drop x
in a (# + x) or (# x) algebraic expression when x is
going to be more than an order of magnitude
smaller than the # it is being added or subtracted to
AND it will simplify the algebra.
You will see similar approximations a lot in Acids &
Bases for weak acid and base equilibrium
calculations.

Equilibrium 48

Problem: What is the concentration of Ag+ in a


0.01 M K2SO4 solution to which excess Ag2(SO4) is
added. Ksp (Ag2(SO4)) = 4 109
Initial:
Ag2(SO4) (s)
@Equilib:

2Ag+(aq) + SO42(aq)

Equilibrium 49

Problem: What is the concentration of H+ in a


0.1 M acetic acid (HOAc) solution to which 0.1 M
Na+OAc is added. Keq (HOAc) = 2 105
Initial:

0.1
HOAc(aq)

@Equilib:

0.1 x

0.1

H+(aq) + OAc(aq)
x

0.1 + x

Note: This is called a Buffer Solution (see Acids/Bases)

Equilibrium 50

G & the Equilibrium Constant


As Ive mentioned during the first part of this
chapter, the equilibrium constant is directly related
to the Gibbs Free Energy, G.
G = negative

Keq > 1 (spontaneous)

G = zero

Keq = 1

G = positive

Keq < 1 (non-spontaneous)

(rare)

The mathematical relationship for calculating G,


given Keq is:
G = R T l n K eq
R = 8.314 J/K (gas constant) T = Temp in K

Given the value of G, we can rearrange the above


equation to solve for Keq:
Keq = e(G/RT)
Thus, given G (or H and S) we can calculate
Keq at a given temperature. Similarly, given Keq,
we can calculate G.

Equilibrium 51

G vs. G: Standard vs. Non-Standard Conditions


Remember that the (not) on G indicates that the
numerical value of G is based on the reaction at
standard conditions (1 M solution concentration, 1 atm
gas pressure). Temperature is NOT part of standard
conditions!
As soon as one has a concentration different than 1 M or
1 atm pressure, the not goes away and one has G.
Consider the reaction:

Initial: 1 atm
1 atm
2SO2(g) + O2(g)
Grxn = 142 kJ/mol

1 atm
2SO3(g)

The Grxn of 142 kJ/mol is for when each gas is present


with a concentration of 1 atm. This indicates that the
reaction under these conditions will proceed to make
products (spontaneous).
As the reactants start reacting, however, their
concentrations decrease (SO2 twice as fast as O2) and
G turns into G and becomes less negative.
When G = 0 the reaction has reached equilibrium.

Equilibrium 52

Example: A reaction has a G value of 40


KJ/mol at 25C. What is the Keq for this reaction?
First convert the temperature from C to K:
Temp (K) = 25C + 273 = 298 K
Now we can use the formula for calculating Keq:
Keq =

e(G / RT)

Dont forget to carry along the


sign on G

Keq = e((40000 J/mol) / (8.314 J/mol K)(298K))


Important Note: R the gas constant has
units of J/molK, while we usually express
G in KJ/mol. The units must match!!!
The easiest thing is to multiply the G
value in KJ/mol by 1000 to give J/mol.
Keq = e (16.14)
Keq = 1.02 107

DANGER!!
Common
mistake!!

The negative G represents a spontaneous reaction.


See how this converts over to a large positive Keq
value, indicating that the reaction goes mainly to
products. Note also how there are no units when
you calculate Keq this way.

Equilibrium 53

Example: A reaction has a Keqvalue of 0.01 at


25C. What is G for this reaction?
First convert the temperature from C to K:
Temp (K) = 25 + 273 = 298 K
Now we can use the formula for calculating G:
G = R T l n K eq
G = (8.314 J/mol K)(298K) ln(0.01)
G = (2477 J/mol)(4.6)
G = +11,394 J/mol -- or -- +11.4 KJ/mol
Dont forget to convert J/mol to KJ/mol for the G
value!!
DANGER!!
Common
mistake!!

Equilibrium 54

Problem: A reaction has a Keq value of 10 at 25C.


What is G for this reaction?

Equilibrium 55

Catalysts
Catalyst: a material that speeds up the
RATE of a reaction without being
consumed in the reaction.
A catalyst will NOT change an
equilibrium, only the speed (rate) at
which equilibrium is reached!
Remember that equilibrium is directly
related to thermodynamics. Catalysts
never affect the thermodynamics of a
reaction. They only lower the energy of
activation (kinetics) of a reaction.

Equilibrium 56

Chemical Demonstration:

Oscillating Iodine Reaction


IO3-

H2O2

Mn2+ to Mn3+

HOI

I2

HO

H2O2
O

I3

HO

OH

I
+ H2O

Mn3+ to Mn2+

H2O2

I-

2CO2
HCOOH

OH

Chapters 18 & 19

Acids & Bases


H2O + HF(aq)

H3O+(aq) + F(aq)

Acids & Bases 2

Acid and Base Definitions

Arrhenius
Acid increases H + concentration
Base increases OH- concentration
Brnsted-Lowry (1923)
Acid donates a H +
Base combines with or accepts H +
Lewis
Acid electron acceptor
Base electron donor

Acids & Bases 3

Water itself has some ionic character:


H2O(l) + H2O(l)
H3O+(aq) + OH(aq)

Shorthand:
H2O(l)

H+(aq) + OH(aq)

This is called self-ionization. Although most chemists simply


write H+, it is important to realize that H+ by itself represents
a naked proton. In water the H+ is hydrated, that is, it forms
ionic-diople interactions with other water molecules. A
common way of more accurately representing the fact that it
is hydrated (interacting with waters) is to write H3O+.

Acids & Bases 4

H2O(l)

H+(aq) + OH(aq)

[H + ] [OH ]
Keq =
[H2O]
But H2O is a pure liquid and its concentration is
constant ([H2O] = 55.6 M, activity = 1), so it is not
included in the equilibrium expression:

K w = [H+ ][OH]
K w = 1.0 10 14 @ 20 DC
[H+] [OH] = 1 1014
[H+] = [OH] = 1 107 M
So the [H+] and [OH] concentration in pure water
is 1 107 M
We use Kw to indicate the water self- or autoionization. This is still a Keq or Kc. Chemists use
many subscripts on the equilibrium constant K to
indicate specific types of equilibria:
Kb = base equilibria
Ka = acid equilibria
Ksp = slightly soluble (solubility product) equilibria
Because of the often very small nature of H+
concentrations, chemists (and others) have devised

Acids & Bases 5

a logarithmic scale to simplify expressing these


values:
pH = log [H+]
The negative sign in front of the log makes sure
that most small concentrations of acid are given by
positive values.
[H+] = 1 107 M
pH = log(1 107) = 7.0
The greater the [H+] concentration, the LOWER
the pH value!!!

pH < 7.0
pH = 7.0
pH > 7.0

Acidic
Neutral
Basic (Alkaline)

Acids & Bases 6

Substance

pH

10 M HCl

1.0

1 M HCl

0.0

Stomach Acid (HCl)

1.4

Lemon Juice

2.1

Orange Juice

2.8

Wine

3.5

Black Coffee

5.0

Urine

6.0

Pure Water

7.0

Blood

7.4

Baking Soda Solution

8.5

Ammonia Solution

11.9

1 M NaOH

14.0

10 M NaOH

15.0

Acids & Bases 7

We can also define:

pOH = log

[OH ]

Although most chemists mainly use pH, pOH can


be useful in base equilibrium numerical problems
that we will run into later.
Another definition we use is:

pKw = log Kw = 14
So, for a given water solution:

pOH + pH = pKw = 14
or:

pOH = 14 pH
pH = 14 pOH

Acids & Bases 8

Example: The pH of wine is 3.5. What is the


[H+]? What is the [OH]? What is the pOH?
pH = log[H+] = 3.5
[H+] = antilog(3.5) = 103.5 = 3.16 104 M
14
1

10
K
w
11
=
3.1

10
[OH ] = + =
[H ] 3.16 10 4

pOH = log[OH] = log(3.1 1011) = 10.5


---- or ---pOH = pKw pH = 14 3.5 = 10.5

Problem: The pH in your stomach is around 1.


What is the [H+]? What is the [OH]? What is the
pOH?

Acids & Bases 9

Dissociation Equilibrium Constants


An acid is a compound that will ionize in solution
(usually water) to form a H+(aq) and a counteranion. This can be writen in a general fashion as:
HA(aq)
H+(aq) + A(aq)
The equilibrium expression for this reaction is:
[H+ ][ A ]

Keq =

[HA]

The equilibrium constant, Keq, is often given a


special name for acids: Ka or the acid dissociation
constant. The larger the Ka value the more H+ is
being produced (the lower the corresponding pH),
therefore, the stronger the acid!!!!
Just as with pH we can also define pKa as:

pKa = log Ka
The smaller (and more negative) the pKa the
stronger the acid!!!! This can be confusing, but is
very important: pKas are commonly used in
biology & chemistry!

Acids & Bases 10

Acid Dissociation Constants and pKa Values


Acid

HA

A-

Ka

pKa

hydroiodic

HI

1010

10

hydrobromic

HBr

Br

1010

10

perchoric

HClO4

ClO4

108

hydrochoric

HCl

Cl

108

sulfuric

H2SO4

HSO4

108

nitric

HNO3

NO3

108

trichloroacetic

Cl3COOH

Cl3COO

2 102

0.7

oxalic

HOOCCOOH

HOOCCOO

5.9 102

1.2

sulfurous

SO32

1.5 102

1.8

SO42

1.2 102

1.9

phosphoric

H2SO3
HSO4H3PO4

H2PO4

7.5 103

2.1

nitrous

HNO2

4.6 104

3.3

hydrofluoric

HF

NO2
F

3.5 104

3.5

formic

HCOOH

HCOO

1.8 104

3.8

benzoic

C6H5COOH

C6H5COO

6.5 105

4.2

oxalic (2nd H+)

HOOCCH2COO OOCCH2COO

6.4 105

4.2

acetic

CH3COOH

carbonic
hydrogen sulfide

sulfuric (2nd H+)

1.7 105

4.7

H2CO3

CH3COO
HCO3

4.3 107

6.4

H2S

HS-

9.1 108

7.1

HPO42

6.2 108

7.2

ammonium ion

H2PO4
NH4+

5.6 1010

9.2

hydrocyanic

HCN

NH3
CN-

4.9 1010

9.3

phosphoric (2nd)

Acids & Bases 11

Strong Acids
These are acids that are essentially completely
dissociated in solution (usually water).
H+(aq) + Cl(aq)
HCl(aq)
[H+ ][Cl ]
10 8
K =

[HCl]

In general, Ka > 1 for a strong acid (although there


is no firm dividing line!)
The common strong acids that you are expected to
know are:

HCl, HBr, HI (the hydrohalic acids)


H2SO4 (sulfuric acid) HNO3 (nitric acid)
HClO4 (perchloric acid)
HCl, H2SO4, and HNO3 are often referred to as mineral acids.

A strong acid completely dissociates the first H+, so


the H+ concentration is the same as the given acid
concentration. the pH of a strong acid is just
log of the acid concentration (= log[H+]).

Acids & Bases 12

Problem: What are the pH's of the following


solutions?
a) 0.001 M HCl
b) 0.1 M HNO3
c) 1 105 M H2SO4
d) 10 M HBr
e) 1 M HI
f) 0.1 M HF
g) 0.01 M HCl
h) 1 104 M HNO3
i) 1 M acetic acid
j) 1 1014 M HCl

Acids & Bases 13

Explanation for j) on the last page:


The actual proton concentration for any acid
dissolved in water is more precisely defined as:
[H+]total = [H+]acid + [H+]water
Normally, [H+]water is 1 107 M and is much less
than [H+]acid, so that we usually ignore it.
BUT, in this example, [H+]acid turns out to be
only 1 1014 M, which is much, much less than
the [H+]water.
So in this example we can actually ignore the tiny
amount of H+ contributed from the strong acid and
only consider the H+ naturally present in water:

This will work for acid concentrations of 1 108 M and


lower. It gets complicated mathematically right around
1 107 M (not dealt with in this course).

Acids & Bases 14

Lewis Dot Structures: Wheres the Proton?

When we write the formula for HNO3, it does NOT


mean that the proton (H+) is attached to the
nitrogen atom. The proton always binds to, or is
associated with a lone pair of electrons, usually on
one of the outer negatively charged atoms of a
polyatomic anion. Consider the Lewis Dot
structures for NO3, SO42, and ClO4:
when N has 4 bonds,
it is f or mally cationic

S can hav e
mor e than 8 ve-

O
O

N
O

Cl can hav e
mor e than 8 v e-

O
O

Cl

nitrate anion

sulfate dianion

perchlorate anion

Resonance, of course, will spread out the negative


charges and bonding over all the O atoms. H+ in
each of these cases binds to the oxygen atom(s) that
is (are) negatively charged:
H

N
O

O
O

O H

Cl

O H

O
O H

nitric acid

sulfuric acid

O
perchloric acid

Acids & Bases 15

Problem: Draw the Lewis Dot structure that best


minimizes the formal charges for the following
acids (some strong, some weak). Cleary show
where the H+ is coordinated.
a) HBF4 (fluoroboric acid, strong)

b) H2CO3 (carbonic acid, weak)

c) H3PO4 (phosphoric acid, weak)

d) CF3SO3H (triflic acid, strong)

e) HCO2H (formic acid, weak)

Acids & Bases 16

Strong & Weak vs. pH


A common mistake is to confuse pH with strong or
weak acids/bases. For example, if I tell you a
solution has a pH of 5.0, most of you would
incorrectly assume that it is a weak acid. Maybe,
maybe not.
A pH of 5.0 tells you that it is a weakly acidic
solution. This solution might have been made
from a medium to considerable amount of a weak
acid, or a very small amount of a strong acid.
Without knowing the concentration (or amount
and nature) of the acid used (not just the resulting
H+ concentration or pH), you cant tell whether it is
composed of a strong or weak acid.
It is true that solutions with very low pHs (for
example, 1.0) can pretty much only be composed
of strong acids.
So you need to be very careful with your language
in dealing with strong & weak acids and solutions
that are strongly or weakly acidic.

Acids & Bases 17

Polyprotic Acids
Sulfuric Acid:

H2SO4(aq)

H+(aq) + HSO4 (aq)

HSO4 (aq)

H+(aq) + SO42 (aq)

Ka1 108
Ka2 10-2

Phosphoric Acid:
H3PO4(aq)

H2PO4 (aq)

HPO42 (aq)

H+(aq) + H2PO4 (aq) Ka1 10-3

H+(aq) + HPO42 (aq) Ka2 10-8

H+(aq) + PO43 (aq) Ka3 10-13

Carbonic Acid:

H2CO3(aq)

H+(aq) + HCO3 (aq)

HCO3 (aq)

H+(aq) + CO32 (aq)

Ka1 10-7
Ka2 10-11

Acids & Bases 18

Note that for polyprotic acids only the first


dissociation (the first H+) is likely to occur. The
second (or third) dissociation process is far less
likely, so the contribution of these subsequent
dissociations to the overall [H+]total .

So, generally we only have to worry about the


first proton and the first dissociation constant
unless one is doing a titration or other acidbase reaction!!

Acids & Bases 19

Strong Bases
These are usually (and most commonly) alkali
metal hydroxides that dissociate completely in
solution:
NaOH(aq)

Kb =

Na+(aq) + OH(aq)
[Na + ][OH ]
10 8

[NaOH]

The common strong bases that you are expected to


know are:

LiOH, NaOH, KOH, RbOH, & CsOH


The alkaline earth hydroxides Ca(OH)2, Sr(OH)2,
Ba(OH)2 are medium strong bases.
Be(OH)2 & Mg(OH)2 are considered to be weak
bases since they only partially dissociate in water.
The smaller ionic radius of Be2+ and Mg2+ cations
polarize coordinated H2O enough to promote
hydrolysis (that is, loss of H + from H2O).

Acids & Bases 20

It is very important to remember that


there are almost always two steps in
converting from [OH] to pH:
1) convert [OH] to [H+]:
14
K
1

10
w
[H+ ] =
=
[OH ]
[OH ]

2) then convert [H+] to pH.


---- or ---1) convert [OH] to pOH
2) then convert pOH to pH:

pH = 14 pOH

Acids & Bases 21

Problem: What are the pH's of the following


solutions? What are the pOH's?
a) 0.001 M NaOH
b) 0.1 M CsOH
c) 1 105 M KOH
d) 10 M RbOH
e) 1 M NaOH
f) 0.1 M Be(OH)2
g) 0.01 M LiOH
h) 1 104 M NaOH
i) 1 M ammonia
j) 1 1014 M KOH

Acids & Bases 22

Explaination for j) on the last page:


The actual hydroxide concentration for any base
dissolved in water is more precisely defined as:
[OH]total = [OH]base + [OH]water
Normally, [OH]water is 1 107 M and is much
less than [OH]base, thus we can usually ignore it.
BUT, in this example, [OH]base turns out to be
only 1 1014 M, much less than the [OH]water.
So in this example we can actually ignore the tiny
amount of OH contributed from the base and only
consider the OH naturally present in water:

This will work for base concentrations of 1 108 M and


lower. It gets complicated mathematically right around
1 107 M (not dealt with in this course).

Acids & Bases 23

Conjugate Acid-Base Pairs


Bronsted-Lowry Definition of a Base: a substance
that combines or accepts a H+. Consider the
dissociation of an acid:
H2O + HF(aq)
H3O+(aq) + F(aq)

Because this is an equilibrium, F(aq) is back reacting with


H+(aq) to produce undissociated acid HF(aq). Therefore,
F(aq) is acting like a base! Since it was originally part of the
acid (HF), theres a special name for it: conjugate base.

Strong acids have weak conjugate bases.


Weak acids have stronger conjugate bases
(but usually not as strong as OH).
To indicate that an equilibrium favors one side of a
rxn chemists sometimes use the double equilibrium
arrows where one arrow is shorter. For a strong
acid (weak conjugate base) one could write:
HCl(aq)
H+(aq) + Cl(aq)

Acids & Bases 24

Bronsted-Lowry Definition of a Acid: a substance


that donates a H+.
Consider the reaction of a weak base with water:
NH3(aq) + H2O

NH4+(aq) + OH(aq)

Because this is an equilibrium, NH4+(aq) donates a H+(aq)


that reacts with OH(aq) to produce the original NH3 (and
water). Therefore, NH4+(aq) is acting like an acid! Since it
started as a weak base (NH3), we call NH4+ a conjugate acid.

Strong bases have weak conjugate acids.


Weak bases have stronger conjugate acids.
In the two reactions shown on this page and the previous,
water is acting as either an acid or a base. Any chemical that
can act as either an acid or base is called amphoteric.

In the following reaction, Na+ is more accurately called


a conjugate Lewis Acid (not a proton donor).

NaOH(aq)

Na+(aq) + OH(aq)

Acids & Bases 25

Weak Acids
These are acids that only dissociate to produce a
small amount of H+ in solution:

HF(aq)

H+(aq) + F(aq)

[H+ ][F ]
Ka =
= 3.5 10 4
[HF]

Most of a weak acid is dissolved in solution in its


undissociated, neutral form. An acid is weak
because its conjugate base (counter-anion) is a
good base and likes to bind to H+.
A weak acid typically has Ka < 1 103
Most weak acids are organic acids based on the
carboxylic group:

Some of the more common weak acids that we run


into on a daily basis include:

Acids & Bases 26

Acetic Acid

Carbonic Acid

Phosphoric Acid

H3C

OH

HO

active ingredient
in vinegar

HO

OH

OH
OH

used in soda
(Coke, Pepsi)

formed when CO2


dissolves in water

Acetylsalicylic Acid

Ascorbic Acid

O
HO
H3C

O
C

O
C

C
C

HO

CH2

OH

HO

OH

H
Aspirin

Vitamin C

Note that the OH groups in all these examples are not


hydroxides!! The oxygen atom of the OH is strongly bonded
to the atom they are attached to and will not fall off as OH .
Instead they dissociate H+ due to the Os electronegativity
and ability to stabilize negative charge(s).

Acids & Bases 27

Example: What is the pH of a 0.1 M solution of

acetic acid? Ka 1 105


Abbreviation for acetic acid: HOAc
Initial Cond: 0.1 M
HOAc(aq)
@ Equilib:

0.1 - x

0M
0M
H+(aq) + OAc(aq)
x

Substitute our x values into the equilibrium


expression and solve for x:
[H+ ][ OAc ]
K =
= 1 10 5

[HOAc]
( x )( x )
= 1 10 5
(0.1 x )

But, this will be a quadratic expression and we'll


have to use the quadratic formula to solve for x
(ugh!).
There is, however, a very good approximation we
can make to dramatically simplify the algebra.

Acids & Bases 28

Because Ka is fairly small (105) we know that


acetic acid is a weak acid and that we are only
going to make small quantities of H+. That means
that x should be a rather small number as well.
This in turn means that 0.1 x should be 0.1.
Our assumption, therefore, will be that x will be
much less than 0.1 M (initial concentration of
acid). This really simplifies the algebra:

( x )( x )
= 1 10 5
(0.1 x )
assume that x << 0.1 and drop from deonominator

( x )( x )
= 1 10 5
(0.1)

Important Approximation!!

x 2 = 1 10 6 } take square root of each side


x = [H+] = [OAc] = 1 103 M
pH = log(1 103) = 3.0

Acids & Bases 29

When can I drop x??


When is it a good approximation??
When Keq is 1 103 or smaller and x works out to
be at least an order of magnitude smaller than the
initial concentration that one is subtracting x from.
This also works when one is adding x to the initial
concentration (e.g., common ion problems).
Problem: What is the pH of a 10 M solution of
hydrogen sulfide? Ka 1 107
H2S(aq)

H+(aq) + HS(aq)

Acids & Bases 30

Problem: Prof. Stanley makes a new organic acid.


He prepares a 0.01 M solution and finds that the
pH is 4.0. What is the Ka for this new acid?
HA(aq)

H+(aq) + A(aq)

Problem: What is the pH of a 0.01 M solution of


HCN? Ka 4 1010

Acids & Bases 31

Effect of Structure on Acid/Base Behavior


Hydrohalic Acids
A combination of factors affects the acid strength
of hydrohalic acids:
1) Polarity of the H-X bond
Electrostatic
2) Strength of the H-X bond
attraction!!
3) Stability of the conjugate base, X
Thus, HF is a weak acid because the rather small
fluoride ion (F) has a concentrated negative
charge that very effectively and strongly attracts
the H+ cation, not allowing it to dissociate and
become a strong acid (like HCl, HBr, or HI).
HF
HCl
HBr
HI

Acids & Bases 32

Oxyacids
Oxyacids are those acids in which the central atom
(most commonly N, Cl, S or P) is bonded to at least
one, and usually more, oxygen atoms. The
resulting negative charge on this unit is balanced
by the proper # of H+ that associate with the
oxygen atoms (one per oxygen atom).
A list of common oxyacids and their names:
HNO2

HNO3

Nitrous

Nitric

HClO

HClO2

HClO3

HClO4

Hypochlorous

Chlorous

Chloric

Percloric

H2SO3

H2SO4

Sulfurous

Sulfuric

H3PO2

H3PO3

H3PO4

Hypophosphorous

Phosphorus

Phosphoric

Acids & Bases 33

The strength of oxyacids increases for a series of


compounds as follows:
1) given the same central atom, the more oxygens
present the stronger the acid
Reasoning: the more electronegative oxygen atoms
present, the more the negative charge on the anion is
spread out over a larger volume. This means that the
negative charge will be less concentrated on any single
oxygen. Thus, there will be a lower electrostatic attraction
to the H+ cations, allowing them to dissociate more easily.

2) for the same # of oxygens, the more electronegative the central atom, the stronger the acid
Reasoning: the more electronegative the central atom, the
more the negative charge on the anion will be pulled
towards the central atom and away from the outlying
oxygens. Thus, there will be a lower electrostatic
attraction to H+ cations, allowing easier dissociation.

3) For almost all acids, the higher the negative


charge on the anion (conjugate base), the lower
the acidity of the acid.
Reasoning: the higher the negative charge on the anion
(mono- or polyatomic) the stronger the electrostatic
attraction to the H+ cations. This makes it harder for the
H+ cation to dissociate.

Acids & Bases 34

4) For almost all acids, the more electronegative


atoms present (like O or F) the higher the
acidity of that acid.
Reasoning: the more electronegative atoms present, the
more the negative charge on the anion will be pulled
towards these atoms and away from the atom that the H+ is
associated with.

Example:
HClO

HClO2

HClO3

HClO4

Hypochlorous

Chlorous

Chloric

Percloric

Ka = 3 x 108

Ka = 1 x 102

Ka = 5 x 102

Ka 1 x 1010

Electrostatic charge potential (ECP) surface plots for ClO


through ClO4 anions (no H+). The red color (dark) indicates
more negative charge and a stronger electrostatic attraction
to the H+ cation (weaker acid). Positive charge is indicated
by the blue color (darker color on center atom).

Acids & Bases 35

Here are the ECP surface plots for the acids:


HClO
HClO2
HClO3

HClO4

Note that the blue area around the H atom indicating positive
charge is increasing as O atoms are added.

Problem: Which is the stronger acid of the pair?


a) H2SO4 or H3PO4
b) HNO2 or HNO3
c) HOBr or HOI
d) H2SeO4 or H2SeO3
e) HI or HF
f)

H2SO3 or H2SO4

g)

O
H
H

C
C

OH

or

F
F

C
C

OH

Acids & Bases 36

Weak Bases
These are bases that only react to a relatively small
extent with H+ in solution:
:NH3(aq) + H+(aq)
NH4+(aq)
The convention, however, for writing equilibria for
weak bases is to react the base with water, which
generates a small quantity of OH-:
NH4+(aq) + OH(aq)
:NH3(aq) + H2O
This is the equilibrium we use to define our base
equilibrium constant, Kb:
[NH4 + ][ OH ]
= 1.8 10 5
K =

[NH3]

In this equilibrium NH4+(aq) is called the


conjugate acid (it acts like a weak acid).

Acids & Bases 37

Some weak bases are shown in the table below:


Base

Formula

Structure

Kb

methylamine

NH2CH3

H N CH 3

4.4 x 10-4

carbonate ion

CO32-

2-

1.8 x 10-4

C
O

ammonia

NH3

H N H

1.8 x 10-5

hydrosulfide ion
nicotine

HS-

1.8 x 10-7

H S

C10H14N2

N
CH 3
N

hydroxylamine

NH2OH

H N OH

7 x 10-7
1.4 x 10-11
1.1 x 10-8

pyridine

C5H5N

1.9 x 10-9

Note that all bases have atoms with lone pairs of


electrons that can interact with a H+. Remember
that a H+ doesn't have any electrons and has a
positive charge. It will be attracted to atoms with
negative charges and/or lone pairs of electrons.

Acids & Bases 38

Example: What is the pH of a 0.1 M solution of

ammonia? Kb 1 105

Init:

0.1 M

0M

:NH3(aq) + H2O

0M

NH4+(aq) + OH(aq)

@ Eq: 0.1 - x

Substitute our x values into the equilibrium


expression and solve for x:
[NH4+ ][ OH ]
= 1 10 5
K =

[NH3]

( x )( x )
= 1 10 5
(0.1 x )
But, this will be a quadratic expression and the
quadratic formula is needed to exactly solve for x
(ugh!).
But lets use the very good approximation from
weak acid equilibria problem solving that will
dramatically simplify the algebra.

Acids & Bases 39

Because Kb is pretty small (105) we know that


ammonia is a weak base and that we are only going
to make small quantities of OH. That means that
x will be a pretty small number. This in turn
means that 0.1 - x will be 0.1.
Our assumption, therefore, will be that x will be
much less than 0.1 M (initial concentration of acid).
This now really simplifies our algebra:

( x )( x )
= 1 10 5
(0.1 x )
assume that x << 0.1 and drop from deonominator
Important Approximation!
( x )( x )

= 1 10 5
(0.1)
x 2 = 1 10 6 } take square root of each side

x = [OH] = [NH4+] = 1 103 M


pOH = log(1 103) = 3.0

pH = 14 pOH = 11

DANGER!!
VERY
Common
mistake!!

Acids & Bases 40

Problem: What is the pH of a 1 M solution of


sodium carbonate? Kb 1 104
CO32(aq) + H2O

HCO3(aq) + OH(aq)

Problem: What is the pH of a 0.01 M solution of


NH2OH? Kb 1 108

Acids & Bases 41

Lewis Acids & Bases


Acid
electron acceptor
electron donor
Base
Although the Lewis definition includes the
traditional Arrhenius and Brnsted-Lowry acids
& bases mentioned so far, it also encompasses
molecules that dont. Foremost are metal atoms
that form bonds to other molecules using empty
orbitals on the metal (Lewis acid) and filled lone
pairs on the donor atoms (Lewis bases).
Transition metal atoms typically form the
strongest bonds, followed by actinide and
lanthanides. Some examples are shown below:
PR3
Cl

Cl

Os

O
H

PhO

H3C

Cl

H2
N

R3P

Pd

PR3

N
PR3
Cl
O H2
Problem: Identify the Lewis Base and Acid parts of
each compound shown above. Extra: What is the
oxidation state of the metal in each compound?

Acids & Bases 42

Another class of important Lewis acids are


trivalent compounds of boron and aluminum that
have an empty orbital present. These are typically
very reactive to donor molecules including water.
BF3, for example, is considered a superacid.
F
Cl
B

Al

F
F
Cl
Cl
BCl3 will form a moderately strong bond to :NH3:
Cl
B

Cl

Cl

Cl
B

N
H

Cl

Cl

N
H

Problem: What is the hybridization of the B in BCl3?


What about the B in Cl3B:NH3?

Acids & Bases 43

Relationship Between Ka and Kb


Consider our ammonia equilibrium:
:NH3(aq) + H2O
NH4+(aq) + OH(aq)

[NH4+ ][ OH ]
Kb =
[NH3]
Kw
but remember that : [OH ] =
[H+ ]
substituting this in for [OH ] we now have :
[NH4+ ] Kw
Kb =
[NH3] [H+]

This expression now corresponds to the following


equilibrium multiplied times Kw:

:NH3(aq) +

H+(aq)

NH4+(aq)

The reverse of this reaction, however, is the "acid"


equilibrium:
NH4+(aq)

:NH3(aq) +

H+(aq)

Writing the reaction this way means that we can


now set-up a Ka equilibrium expression:

Acids & Bases 44

[NH3][H+ ]
Ka =
[ NH+4 ]
But note that the reciprocal of this expression is
already in the Kb relationship:

[NH3][H+ ]
Ka =
[ NH+4 ]

[NH4+ ] Kw
Kb =
[NH3] [H+]

Therefore, Ka and Kb are related via Kw.


Here are the very important Ka / Kb relationships
you need to know/understand:

Kw
Kb =
Ka

-or-

Kw
Ka =
Kb

Kw = Ka Kb

Acids & Bases 45

Knowing the mathematical relationship between


Ka and Kb is very important. Many references
only list Ka values for bases and not the Kb values
that you would expect.
When a Ka value is given for a base it is really the
value for the conjugate acid of that base. For
example:
Kb = 1.8 105
Base = NH3
Conjugate Acid = [NH4]+ Ka = 5.6 1010
(Kb)(Ka) = (1.8 105)(5.6 1010) = 1 1014
Kw
If a reference gives a Ka value for the base you are
looking up, you'll have to convert it to a Kb value
in order to calculate the [OH], and then the pH.
You must, therefore, pay close attention to what
data the problem is giving you and what you need
to use. READ CAREFULLY!!
DANGER!!
VERY Common mistake!!

Acids & Bases 46

Problem: What is the pH of a 0.1 M solution of the

weak base triethyl amine (NEt3)? Ka = 1 1011

Problem: What is the pH of a 0.01 M solution of

hydroxyamine (NH2OH)? Ka = 1 106

Acids & Bases 47

Salts of Weak Acids and Bases


When a weak acid (e.g., acetic acid) reacts with a
strong base (e.g., an alkali hydroxide like NaOH)
water and the salt of the weak acid is formed:

Note that the acetate anion formed in the reaction


is itself a weak base. Thus it can react with water
to produce a small amount of [OH].
Therefore, dissolving sodium acetate (the salt of
acetic acid) in water will make a basic solution.

Acids & Bases 48

Similarly, the reaction of a weak base with a strong


acid will produce the salt of a weak base, which will
act as a weak acid. Shown below we have the
reaction of ammonia with HCl to produce
ammonium chloride (the salt of a weak base):

Dissolving ammonium chloride in water, therefore,


produces an acidic solution (NH4+ is a weak acid).

Acids & Bases 49

The big question is How the heck do I tell whether


a salt will produce an acidic, basic, or neutral
solution??
The KEY is to remember the strong acids and
bases:
Strong Acids:
HCl(aq)
HBr(aq)
HI(aq)
HNO3(aq)
H2SO4(aq)
HClO4(aq)

H+(aq)
H+(aq)
H+(aq)
H+(aq)

+
+
+
+

Cl(aq)
Br(aq)
I(aq)
NO3(aq)

2H+(aq) + SO4(aq) *
H+(aq) + ClO4(aq)

These are
extremely
weak
conjugate
bases -Neutral
Anions!

Strong Bases:
LiOH(aq)
NaOH(aq)
KOH(aq)
RbOH(aq)
CsOH(aq)

Li+(aq)
Na+(aq)
K+(aq)
Rb+(aq)
Cs+(aq)

+
+
+
+
+

OH(aq)
OH(aq)
OH(aq)
OH(aq)
OH(aq)

These are
extremely
weak
conjugate
acids -Neutral
Cations!

Putting these Neutral (neutral here refers to their


acid-base properties, NOT their charges!!) anions
and cations together generates Neutral (not acidic
or basic) salts (Ca2+, Sr2+, Ba2+ salts not shown):

Acids & Bases 50

Neutral
Cation
Li+
Na+
K+
Rb+
Cs+

Neutral
Anion
Cl
Br
I
NO3
SO42
ClO4

Neutral Salts
LiCl, LiBr, LiI, LiNO3, Li2SO4,
NaCl, NaBr, NaI, NaNO3, Na2SO4,
KCl, KBr, KI, KNO3, K2SO4,
RbCl, RbBr, RbI, RbNO3, Rb2SO4,
CsCl, CsBr, CsI, CsNO3, Cs2SO4,
Perchlorate salts are explosive !!!

Solutions of these Neutral Salts are neither acidic


nor basic, but rather have a pH = 7 (Neutral!).
A simple set of guidelines, therefore, are:
CATIONS other than Li+, Na+, K+, Rb+, Cs+ (&
Ca2+, Sr2+, Ba2+) will generate ACIDIC solutions
(that is, the cation is a good conjugate acid)
ANIONS other than Cl, Br, I, NO3, SO42 will
generate a BASIC solution (that is, the anion is a
good conjugate base)

Acids & Bases 51

SALTS
Cation(+)Anion(-)
Cation comes from the base Anion comes from the acid
Identify the Cation
Is the cation from a strong base (Group 1A or Ca 2+, Sr 2+, Ba 2+)
YES

NO

Identify the anion


Is it from a strong acid?
YES
Salt Solution
will be
Neutral
(Salt of a
strong
acid & base)

Cation is from a weak base


(cation is a conjugate acid)

NO
Anion is from
a weak acid
(anion is a
moderate to
good conjugate
base)

Salt solution
will be basic
(salt of a
weak acid)

Identify the anion


Is the anion from a strong acid?
YES
Salt solution
will be acidic
(salt of a
weak base)

NO

Salt of a
weak acid &
a weak base!
Complicated!
Dont worry

Acids & Bases 52

Problem: Will the following salts make an acidic,


basic or neutral solution when dissolved in water?
a) NaF
Ka (HF) = 3.5 10-4
b) NaCl

c)

N H+ Cl-

Ka (HCl) = 1 108
Ka (pyridineH+) = 5 10-6

d) C6H5COONa Ka (C6H5COOH) = 6.5 10-5


e) RbBr

Ka (HBr) = 1 1010

f) CH3COOCs

Ka (CH3COOH) = 1.7 10-5

g) Cs2S

Ka (H2S) = 9.1 10-8

h) NH4NO3

Ka (NH4+) = 5.6 10-10

i) KCN

Ka (HCN) = 4.9 10-10

j) KNO3

Acids & Bases 53

Example: What is the pH of 1 M NaF?


Ka (HF) = 1 104. Neutral

Basic
Anion

Cation

Initial:

@ Eq:

1M

F(aq) + H2O

HF(aq) + OH(aq)

1-x

This is a basic equilibrium, so we need to convert


Ka into Kb:
14

Kw
1 10
10
Kb =
=
=
1

10
Ka
1 10 4

DANGER!!
VERY
Common
mistake!!

Our equilibrium expression, therefore, is:


[HF][OH ]
10
Kb =
=
1

10
[F ]
( x )( x )
= 1 10 10
(1 x )
assume that x << 1 and drop from deonominator

( x )( x )
= 1 10 10
(1)
x 2 = 1 10 10 } take square root of each side

x = [OH] = [HF] = 1 105 M

Acids & Bases 54

So the [OH] = 1 105 M. The pOH is:


pOH = -log(1 105) = 5
The pH, then, is given by:
pH = 14 - pOH = 14 - 5 = 9

DANGER!!
VERY
Common
mistake!!

Acids & Bases 55

Problem: What is the pH of a 0.1 M solution of


NH4Cl. Ka (NH4+) = 1 x 109.

Acids & Bases 56

Reactions of Acids & Bases


Strong Acids and Strong Bases:
The reaction of a strong acid and strong base is the
simplest type of acid-base reaction.
Consider the rxn of NaOH and HCl:
Na+(aq) + OH(aq) + H+(aq) + Cl(aq)
H2O + Na+(aq) + Cl(aq)
The rxn above shows all the aqueous species. Note that
NaCl(aq) is one of the products. When acids react with
bases, water and the salt of the acid/base are formed (the
base provides the cation and acid the anion).

The net ionic equation is shown below:


H2O
OH(aq) + H+(aq)
A key point to remember is that when we react
acids and bases we are usually reacting solutions of
acids and bases. When one mixes two solutions
together the overall volume increases and the
concentrations of all species will decrease!
Because of the change in volume due to the mixing
together of two solutions one has to take this into
account.
DANGER!! VERY Common mistake!!
We do this by dealing directly with MOLES and
not Molarity. So you have to:

Acids & Bases 57

convert molarity into moles,


do your calculations, then
convert back to molarity using the NEW total
(combined) solution volume
Example: What is the pH if we mix 100 mL of 0.1
M HCl with 50 mL of 0.1 M NaOH?
First, convert concentrations into moles:
(100 mL)(0.1 M) = 10 mmol HCl
(50 mL)(0.1 M) = 5 mmol NaOH
Now for the reaction: 5 mmol NaOH will react
with only 5 mmol HCl, leaving behind 5 mmol HCl.
But now we have 150 mL of solution, so [H+] is:

DANGER!! VERY Common mistake!!

5 mmol
= 0.033 M H+
150 mL

p H = 1.5

Acids & Bases 58

Problem: What will be the pH when 100 mL of


0.1 M HNO3 is mixed with 300 mL of 0.2 M KOH?

Problem: What will be the pH when 100 mL of


0.1 M HNO3 is mixed with 200 mL of 0.05 M
KOH?

Acids & Bases 59

Strong Acids and Weak Bases:


Strong Bases and Weak Acids:
Things get more complicated (well, not too bad)
when you react a strong acid with a weak base or a
strong base with a weak acid.
For this case, however, we will simplify things a
little by only using equivalent amounts of the acid
and base.
The key thing to remember when you react a weak
acid or base with a strong base or acid is that one
ends up with the SALT of that weak acid or base.
Remember that the SALT of a weak acid is actually
a weak base and gives a basic solution.
Similarly, the SALT of a weak base generates an
acidic solution.

DANGER!!
VERY Common mistake!!

Acids & Bases 60

Example: What is the pH if 150 mL of 0.5 M


acetic acid is mixed with 150 mL of 0.5 M NaOH?
Ka (HOAc) 1 105.
H2O + OAc(aq)
HOAc(aq) + OH(aq)
The OAc produced is a weak base and will
realizing this - DANGER!!
produce a weakly basic solution: NotVERY
Common mistake!!
H2O + OAc(aq)
HOAc(aq) + OH(aq)
Since this is a base equilibrium, we need to use Kb:

Kw
1 10 14
9
Kb =
=
=
1

10
Ka
1 10 5
Not realizing this - DANGER!!
Kb =

[HOAc][OH ]
[OAc ]

VERY Common mistake!!

= 1 10 9

Calculate the concentration of OAc :


(150 mL)(0.5 M) = 75 mmol HOAc
(150 mL)(0.5 M) = 75 mmol OH

Acids & Bases 61

Thus, we will produce 75 mmol of OAc. The


concentration of OAc, therefore, will be:
Volume Change DANGER!!
VERY Common
mistake!!

75 mmol
= 0.25M OAc
300 mL

Now we can set-up our initial and @eq conditions:


Initial:

0.25 M
H2O + OAc(aq)

@ Eq:

HOAc(aq) + OH(aq)

0.25 - x

[HOAc][OH ]
9
1
10
=

Kb =
[OAc ]
( x )( x )
= 1 10 9
(0.25 x )
assume that x << 0.25 and drop from deonominator
Approximation
( x )( x )
9
= 1 10
(0.25)
x 2 = 2.5 10 10 } take square root of each side
x = 1.6 10 5 = [OH ]

pOH = 4.6 } so the pH = 14 - 4.6 = 9.4


Forgetting to convert pOH to pH - DANGER!! VERY Common mistake!!

Acids & Bases 62

Problem: What is the pH of the reaction of


500 mL of 2 M NH3 and 500 mL of 2 M HCl?
Ka (NH4+) 1 1010.

Acids & Bases 63

Titrations
A titration is the careful measured addition of a
known concentration of one substance that will
react with another unknown material in order to
determine the concentration of the unknown
material.
Determining the concentrations of unknown materials is a
routine procedure in chemistry. Titrations are commonly
used to determine the concentrations of acids and bases in
solution, as well as many other chemicals. Titrations are also
often the simplest and least expensive way of determining
concentrations of unknown materials.

If we are titrating an unknown acid with a known


amount of base with a known concentration, we
can use the (M1)(V1) = (M2)(V2) relationship to
find the unknown's concentration at the
equivalence point (the point at which we've added
just enough base to react with all the acid).
In order to determine when we have reached the
equivalence point in an acid base reaction, we
generally use an indicator.

Acids & Bases 64

Indicators
An indicator is a weak organic acid or base that
has sharply different colors in its associated and
dissociated forms:
HIn(aq)
H+(aq) + In(aq)
red acidic
blue basic
Indicators usually have very intense colors so one
only has to use a very small amount (a few drops)
so it will not affect the titration of the solution.
Remember that the indicator is an acid or base so if
you add a lot it will affect the titration!!
OH

Phenophtalein

OH

H+

HO

HO
acidic form - colorless

anionic basic form - red

Methyl Red
O

O
OH
N N

acidic form - red

O
N

N N

anionic basic form - yellow

H+

Acids & Bases 65

Some Common Acid-Base Indicators


Name

pH color change
region

Acid color

Base color

Methyl violet

0-2

yellow

violet

methyl yellow

1.2 - 2.3

red

yellow

methyl orange

2.9 - 4.0

red

yellow

methyl red

4.2 - 6.3

red

yellow

bromthymol blue

6.0 - 7.6

yellow

blue

thymol blue

8.0 - 9.6

yellow

blue

phenolphthalein

8.3 - 10

colorless

pink

Alizarin yellow

10.1 - 12

yellow

red

HIn(aq)

H+(aq) + In(aq)

[H+][In ]
Ka =
[HIn]
At the color change [H+] = [In] = [HIn], so:

Acids & Bases 66

Titrating an unknown strong acid with a known


amount of strong base:

Acids & Bases 67

Titrating an unknown strong base with a known


amount of strong acid:

Acids & Bases 68

Titrating an unknown weak base with a known


amount of strong acid:

Acids & Bases 69

Titrating an unknown weak acid with a known


amount of strong base:

Acids & Bases 70

Buffer Solutions
Consider an acetic acid solution:

HOAc(aq)

H (aq) + OAc (aq)

If we add enough NaOAc (the salt of acetic acid) to


increase the OAc concentration roughly equal to
HOAc, we now form the following mixture:

HOAc(aq)

+
H (aq)

OAc(aq)

The added OAc, which is a weak base, will consume


some of the free H+ causing the pH to rise (become less
acidic).
Note that if we add H+ to this solution, it will react with
the large pool of weak base OAc to form HOAc. The
H+ concentration, therefore, will stay about the same.
Similarly, if we add some OH it will react with the H+
present. Since we have an equilibrium, however, some
HOAc will, in turn, dissociate to replace the missing H+,
keeping it about the same.
A solution of a weak acid and the salt (conjugate base)
of a weak acid or a weak base and the salt
(conjugate acid) of a weak base is called a BUFFER!

Acids & Bases 71

If the concentration of the two components is high


enough and you dont add too much acid or base, the
buffer solution will be very resistant to changes in the pH
due to added acid or base.
Buffer solutions are critically important to biological
systems (i.e., keeping us alive!).
For a solution of a weak acid or base and their salt, one
can write the following equations for calculating the
[H+] and [OH] of the buffer solution generated:

[OH ]

[Base]
= Kb

[
acidic
salt
]

[Acid]
= Ka

[
basic
sal
t]

These are called the Henderson-Hasselbalch


equations.
The log form of these equations:
[H+]

[basic salt]
pH = pK a + log

[
acid
]

[acidic salt]
pOH = pKb + log

[
base
]

Acids & Bases 72

If the acid/base concentration is the same as the


salt concentration, then we can write:
[OH] = Kb
[H+] = Ka
or
pOH = pKb
pH = pKa
By varying the weak acid or base and the salt being
used to make the buffer solution, as well as their
concentration ratio, one can set the pH of the
buffer to almost anything you want.
The more concentrated the buffer components, the
more effective the buffer solution will be at
resisting pH changes. But remember that you can
always overload a buffer by adding too much acid
or base to it.
One of the trickiest things for you to determine is
just what salts will work to make a buffer. The
salts of strong acids and bases (i.e., NaCl, KBr,
Na2SO4, CsNO3) do NOT usually make buffers,
nor do mixtures of strong acids and bases with
their salts!

Acids & Bases 73

Note, however, that reacting equivalent of a


strong base with a weak acid, generates a buffer
solution!

This is the region where we


have a buffer solution present

Acids & Bases 74

Problem: Which of the following solutions is a


traditional buffer prepared from a weak acid/base
and a conjugate salt?
a) 0.1 M NaOH + 0.1 M NaOAc
b) 0.01 M HCl + 0.01 M NaCl
c) 0.5 M NH4Cl + 0.5 M NH3
d) 2 M NaOAc + 2 M HOAc
e) 0.01 M citric acid + 0.01 M sodium citrate
f) 0.3 M H3PO4 + 0.3 M NaH2PO4
g) 2 M HNO3 + 2 M NaNO3
h) 0.05 M H2CO3 + 0.05 M KHCO3
i) 0.5 M HI + 0.5 M CsI
j) 0.2 M benzoic acid + 0.2 M cesium benzoate
k) 1 M NaOAc + 1 M KBr
l) 0.001 M HCl + 0.001 M KOH
m) 0.1 M KOAc + 0.1 M NH3
n) 0.2 M HOAc + 0.2 M KHCO3
b) Which will be the most effective buffer solution
at maintaining a given pH?
c) What are the pHs of the buffer solutions?

REDOX 1

Chapter 21

REDOX &
Electrochemistry

REDOX 2

There are two types of chemical reactions:


1) acid-base reactions (Lewis)
2) electron-transfer reactions
Oxidation/oxidize: when you remove electrons from
a material
Reduction/reduce: when you add electrons to a
material
You cannot have oxidation without reduction; you
cannot generally reduce a molecule without
oxidizing another molecule (and the other way
around).
Oxidizing agent: a chemical that causes another
material to be oxidized (the oxidizing agent is
reduced!)
Reducing agent: a chemical that causes another
material to be reduced (the reducing agent is
oxidized!)

REDOX

REDOX 3

Oxidation States
In order to understand redox reactions, we first
need to be able to figure out what the oxidation state
of an element is. The oxidation state is a method to
indicate how many electrons are "assigned" to a
particular element. For this we use a +/- system:

+n

indicates that an atom has lost electrons and


now has a positive charge

indicates that an atom has gained electrons


and now has a negative charge
indicates that an atom has its elemental
number of electrons assigned to it and,
therefore, has no charge

Oxidation state is a formalism, that is, is may or


may not reflect the actual charge on an atom.
Common reference atoms & their oxidations states:
Alkali metals = +1, alkaline earths = +2
O = 2 (exception = peroxides, H2O2, 1)
Halides = 1 (exception = oxyhalides)

REDOX 4

The key to being able to figure out the oxidation


state of an element in a molecule is to note its
electronegativity:
The higher the electronegativity the more the
element likes to add electrons to its valence shell.
The lower the electronegativity the more likely an
element will lose electrons.
Let's dissect an example reaction:
Reaction: PH 3 + O 2
P2 O5 + H2 O
Electronegativities:

PH 3 + O 2

H = 2.1

P = 2.1

O = 3.5

P2O5 + H2O

-3 +1
0
+5 -2
+1 -2
Atoms that gain electrons usually gain enough to fill
their valence shell (octet rule). Atoms that lose
electrons only lose enough to get them down to the
next lowest filled valence shell (but not always all the
way!).

REDOX 5

When two elements have the same electronegativity


(as with phosphorus and hydrogen) the element
that is furthest to the upper right hand side of the
periodic table is the one that gets the electrons.
1
1

H
Hydrogen

13
4

11
Sodium

19

K
Potassium

37

12
Magnesium

20

55

21

22

23

24

25

26

27

Calcium

38
Strontium

56

Scandium

39

V
Vanadium

Chromium

Manganese

41

42

43

Zirconium

57

72

Niobium

73

Hf Ta

Cesium

Hafnium

Lanthanum

Iron

44

Cobalt

45

Nickel

46

11
29

47

48

Tantalum

Molybdenum

74

Technetium

Ruthenium

75

76

W Re Os
Tungsten

Rhenium

Osmium

Rhodium

77

Ir
Iridium

Palladium

78

Silver

79

Cadmium

80

Pt Au
Platinum

Gold

10

Helium

Ne

Oxygen

Fluorine

Neon

31

14

15

16

17

49
Indium

81
Thallium

18

Si

Cl

Ar

Silicon

Phosphorus

Sulfur

Chlorine

Argon

32

33

34

35

Ge As Se
Galium

Tl
Mercury

Nitrogen

Aluminum

Zinc

He

Cu Zn
Copper

17

Carbon

12 Al
30

15 16

Zr Nb Mo Tc Ru Rh Pd Ag Cd In

Yttrium

Cs Ba La
Barium

28

Cr Mn Fe Co Ni

Ti
Titanium

40

10

Boron

13

Ca Sc

Rb Sr
Rubidium

Group 8

Berylium

Na Mg

14

Li Be
Lithium

18

Periodic Table of the Elements

Germanium

50

Arsenic

51

Selenium

52

Sn Sb Te
Tin

82

Antimony

83

Tellurium

84

36

Kr
Bromine

53

Krypton

54

Xe

Iodine

Xenon

85

86

Pb Bi Po At Rn
Lead

Bismuth

Polonium

Astatine

Radon

It is important to remember that the bond between


phosphorus and hydrogen is, in reality, pretty much
neutral and covalent in character (although the
hydrogen atoms actually do have a small amount of
positive charge on them). Only when there is a
considerable difference in the electronegativities of
two atoms does one see formal charge separations
and polar bonds occurring.

REDOX 6

Problem: What are the oxidation state


assignments for the following compounds:
a) CH4
b) CO2

c) KMnO4

d) FeS

e) H2SO4

f) HCN

g) Na2CrO4

h) N2

i) MnO2

j) H2C2O4

REDOX 7

Balancing Redox Equations


The best way of balancing redox reactions is the half cell
method. Break up the overall rxn into 2 half-cell rxns:
one for the reduction and one for the oxidation. We
then want to multiply each half cell reaction to make the
overall # of electrons the same so they cancel out.

Note that this fairly simple rxn (and many redox rxns)
could have been balanced the normal way. But if it is
a redox rxn and looks hard, use the half cell method.

REDOX 8

Let's do another balancing act:

Note that in the last two examples, one of the products


or reactants (O2, P2O5, and Al2O3) had two redox active
atoms present (O, P, and Al) and that had to be
accounted for when writing out the half cell reaction.
Also note that when balancing half cell rxns, we mainly
worry about the redox active atoms and not the H and O
atoms (assuming they are not changing oxidation states).
These will be balanced at a later point (see next
example).

REDOX 9

Another EXAMPLE:

This is the core redox balanced reaction. Now, you have to


check the oxygen, and then hydrogen atom balance:

REDOX 10

To balance the rxn in basic solution you need to balance


the rxn first as if it was in acidic solution (as shown
above). Then you get rid of the H+ by adding as many
OH to each side as there are protons and reacting the
H+ and OH together to make waters:

REDOX 11

Problem: Balance the following (in water):


a) Cr2O72 + Cl
Cr3+ + Cl2(g) (acidic)

b) MnO4- + CN

MnO2(s) + CNO (basic)

REDOX 12

c) S2O32(aq) + OCl (s)

Cl (aq) + S4O62 (aq)


(acidic)

d) Au(s) + CN(aq) + O2(g)

Au(CN)4(aq) + OH(aq)
(basic)

REDOX 13

Electrochemical/Voltaic/Galvanic Cells
Consider the reaction of Zn(s) and Cu2+(aq):
Zn(s) + Cu2+(aq)
Zn2+(aq) + Cu(s)
This reaction involves the transfer of 2 e- between
the Zn(s) and Cu2+. The Cu2+ oxidizes the Zn, and
is in turn reduced to form metallic Cu, while the Zn
metal dissolves into solution as Zn2+.
This reaction is spontaneous with a G = 212
kJ/mol. Mixing Zn(s) and Cu2+ together in a
beaker causes a spontaneous reaction that gives off
all the energy as heat, warming the solution.
The transferring of the electrons between the Zn
and Cu2+, however, can accomplish much more
useful work if we can figure out a way to get the eto flow through a wire as electricity, where the work
represented by G (free energy) can now be used
as electrical work.

REDOX 14

To harness the intrinsic chemical energy stored in


these oxidizing and reducing agents, we must
separate them to force the electrons to flow though
an external circuit and do some work.
flow of electrons

salt bridge or
semi-permeable membrane

Zn

Zn

2+

Cl Zn

Cl -

Zn

2+ Cl

2+ Cl -

Cl -

Cl Cu
Cl -

Cu

2+

Cl -

2+

Cu

Cu

Cu

2+

Cl - Cl Zn2+
Cl - Cl 2+
2+
2+
Zn Cl Cl - Cu
Cu

Cu

Cu
Cu
Cu
Cu

Cu

ANODE
CATHODE
oxidation
reduction
Anion flow
Red Cat = Reduction occurs at Cathode

REDOX 15

There are different configurations of liquid


galvanic cells that one can setup:
flow of electrons

salt bridge or
semi-permeable membrane

Zn

Cl -

Zn

2+

Zn

2+ Cl

Cl -

Cu

2+

Cu

Cl -

Cl - Cu2+
2+
Cu
Cl
Cl Cl Zn2+
Cl - Cl 2+
2+
2+
Cu
Zn Cl
Cl
Cu
Cl 2+
Zn Cl

ANODE
oxidation

Anion flow

Cu

Cu
Cu
Cu

semipermeable
membrane

Cu
Cu

Cu

CATHODE
reduction

flow of electrons

flow of electrons

ANODE
oxidation

Zn

Zn

Cu
ANODE
oxidation

Anion flow

CATHODE
reduction

salt bridge

CATHODE
reduction

Daniell Cell

REDOX 16

Measuring Cell Potentials (Voltages)

M
salt bridge or
semi-permeable membrane

H2

H+
X
H+

Pt
H+
X

X
H+

+ X

M
X

M+
+
X
H

X
X

X
X
M+ X
M+
+
H

X
X
+
M+
X H
+ X
M
+
+
X H
X M

X
M+

M+

On a platinum electrode, H2(g) and H+(aq) are in


redox equilibrium with one another (1 atm H2, 1M
H+). This is called a hydrogen electrode.
H2

2H+ + 2e-

By setting this electrochemical potential to 0.0 V,


we have a reference electrode to which we can
measure the innate ability of a material in the other
electrode compartment of the electrochemical cell
to either accept (be reduced/cathode) or give up (be
oxidized/anode) electrons from/to H2/H+.

REDOX 17

Standard Reduction Potentials at 25C


Half Cell Rxn
F2(g) + 2e2F-(aq)
O3(g) + 2H+(aq) + 2eO2(g) + H2O
Co3+(aq) + eCo2+(aq)
Cl2(g) + 2e2Cl-(aq)
O2(g) + 4H+(aq) + 4e2H2O
Ag+(aq) + eAg(s)
Cu+(aq) + eCu(s)
Cu2+(aq) + 2eCu(s)
AgCl(s) + eAg(s) + Cl(aq)
Cu2+(aq) + eCu+(aq)
2H+(aq) + 2eH2(g)
Pb2+(aq) + 2ePb(s)
V3+(aq) + eV2+(aq)
Zn2+(aq) + 2eZn(s)
Al3+(aq) + 3eAl(s)
H2(g) + 2e2H-(aq)
Mg2+(aq) + 2eMg(s)
Na+(aq) + eNa(s)
Li+(aq) + eLi(s)

Ered (V)
+2.87
+2.07
+1.81
+1.36
+1.23
+0.80
+0.52
+0.34
+0.22
+0.15
0.00
-0.13
-0.24
-0.76
-1.66
-2.25
-2.36
-2.71
-3.05

It is critically important that you learn how to read,


interpret, understand, and properly use this table.

Voltage (or potential) represents the driving force


for pushing the electrons from one location to
another (for example, from a reducing agent to an
oxidizing agent). The higher the voltage the more

REDOX 18

strongly the electrons will be pushed through the


wire (or solution).
A positive cell potential (voltage) indicates a
spontaneous electrochemical reaction. A negative
cell potential (voltage) indicates a non-spontaneous
reaction (the opposite reaction will, therefore, be
spontaneous!). Note that the sign notation here is
opposite that we learned for G!
Problems: Which of the following substances is the

best reducing agent?


a) Cu
b) Zn
c) Pb

d) Ag

e) H2

Which of the following substances is the best


oxidizing agent?
c) O2
d) Na+ e) Li+
a) Cu+ b) O3

REDOX 19

To calculate the voltage of an electrochemical cell


one can simply add together the two half-cell
reactions that make up the overall galvanic
(electrochemical) cell.
Lets calculate the spontaneous cell potential
(voltage) for our Cu/Zn cell. First look up the two
half cell reactions from the table of Standard
Reduction Potentials:
Cu2+(aq) + 2eZn2+(aq) + 2e-

Cu(s)
Zn(s)

+0.34
-0.76

The more positive potential (relative to the


hydrogen electrode) for the Cu2+ reduction
compared to the Zn2+ reduction means that Cu2+ is
a stronger oxidizing agent (wants to be reduced
more) than Zn2+.

REDOX 20

To add these half-cell rxns together to give us our


overall spontaneous net rxn and cell potential, we
must switch one of the two half-cell rxns around so
that it is written as an oxidation rxn (a compound
giving up electrons on the product side of the rxn).
The half-cell rxn with the smaller positive, or more
negative potential gets flipped around (reversed).
This would be the Zn rxn:
Cu2+(aq) + 2eCu(s)
Zn2+(aq) + 2eZn(s)

+0.34
+0.76

When one flips one of the half-cell rxns around, one


also flips (reverses) the sign of the cell potential!!
The next step is to multiply each of the half cell rxns
by the appropriate # to balance the # of electrons
involved so that they cancel out. For the Cu/Zn
cell, there are 2e- on each side of the two half-cell
rxns, so everything is already balanced:
Cu2+(aq) + 2eCu(s)
Zn2+(aq) + 2e+ Zn(s)

+0.34 V
+0.76 V

Cu2+(aq) + Zn(s)

+1.1 V

Zn2+(aq) + Cu(s)

REDOX 21

So the net reaction has a voltage of +1.1 V, the


positive potential indicates that the rxn is
spontaneous.
Note that the voltage calculated for the cell is for
standard conditions (1 atm, 1 M). Changing the
concentrations away from 1 M will change the cell
potential! Similarly, changing the temperature will
also affect the voltage.
The Nernst Equation allows one to calculate the
voltage (potential) of a electrochemical reaction
when one is not working under standard
conditions!
Also note that for galvanic cells (batteries) we
always want to use a spontaneous reaction that
gives off energy that we can use to perform work.
But on homeworks or tests I may give you an
overall balanced reaction that is non-spontaneous.
In this case you need to write down the half-cell
rxns in the right order to give the overall balanced
rxn and calculate the cell potentials from the way
that these half-cell reactions are written.

REDOX 22

Example: What is the potential for the following

reaction?
2Na+(aq) + 2Cl(aq)

Cl2(g) + 2Na(s)

Reduction half cell: 2[Na+(aq) + 1eOxidation half cell:

2Cl(aq)

This half-cell was the


one that was flipped
around (reversed)

Na(s)]
Cl2(g) + 2e-

2.71 V
1.36 V

sum = 4.07 V
Non-spontaneous

The reverse reaction is very spontaneous with a cell


potential of +4.07 V.

REDOX 23

Example: What is the potential for the

electrochemical cell composed of the following 2


phrased like this, we are looking for a spontaneous
half cell rxns? When
reaction that gives a positive cell potential!
Cr+3(aq) + 3eMnO2 + 4H+ + 2e-

Cr(s)
Mn2+(aq) + 2H2O

-0.74
+1.28

The Cr+3(aq) + 3e- Cr(s) rxn has the more


negative potential, so it gets flipped around:
Cr(s)
Cr+3(aq) + 3eMnO2 + 4H+ + 2eMn2+(aq) + 2H2O

+0.74
+1.28

Now we have to balance the # of electrons in each rxn so we


can add them together. 6 e- is the common factor, so we need
to multiply the Cr rxn by 2 and the MnO2 rxn by 3:
2[Cr(s)

Cr+3(aq) + 3e- ]

3[MnO2 + 4H+ + 2e-

Mn2+(aq) + 2H2O]

+0.74
+1.28

2Cr(s) + 3MnO2 + 12H+


2Cr+3(aq) + 3Mn2+(aq) + 6H2O +2.02V

Note that you do NOT multiply the cell


potentials by the numerical factors (2
or 3) used to balance the # of electrons
in each half-cell rxn!!

DANGER!!
Common
mistake!!

REDOX 24

Example: Zn metal reacts with HCl, but Cu metal

doesnt. Calculate the cell potentials for these rxns


to see if they fit the experimental data.
Zn(s) + 2H+(aq)

Zn2+(aq) + H2(g)

Splitting this rxn into the two half cell rxns, we


have:
Zn(s)
Zn2+(aq) + 2eH2(g)
2H+(aq) + 2eZn(s) + 2H+(aq)

Zn2+(aq) + H2(g)

+0.76
0.00
+0.76V

Note that the Zn half-cell rxn is flipped around


from how it is written in the Standard Reduction
Table, since it is acting as a reducing agent here.
The calculated positive cell potential indicates a
spontaneous rxn.

REDOX 25

For the copper rxn, we have:


Cu(s) + 2H+(aq)

Cu2+(aq) + H2(g)

Cu2+(aq) + 2eH (g)


2H+(aq) + 2e-

Cu(s)

Cu(s) + 2H+(aq)

Cu2+(aq) + H2(g)

0.34
0.00
0.34V

Once again, note that the copper half-cell rxn is


flipped around from how it is written in the
Standard Reduction Table, since it is acting as a
reducing agent here. So here we calculate that the
cell potential is negative, indicating a nonspontaneous rxn or one that should not occur
normally, which fits the experimental data.
This is because copper metal is a more inert
material relative to zinc metal. One could also note
the position of copper above the hydrogen half cell
rxn, which indicates that the reverse rxn of Cu+2
with H2 gas would be spontaneous:
Cu2+(aq) + H2(g)

Cu(s) + 2H+(aq)

+0.34V

Example: What about the reaction of Cu with

nitric acid? Will this be a spontaneous rxn?

REDOX 26

Nitric acid is not like the other strong acids (HCl,


HBr, HI, H2SO4) in that it is a good oxidizing acid
due to the presence of the NO3 anion, which is not
just a simple inert counter-anion for H+. The
combination of NO3 and H+ makes for a rather
strong oxidizing mixture.
Cu2+(aq) + 2e-]
2[NO3(aq) + 4H+(aq) + 3eNO(g) + 2H2O]

3[Cu(s)

3Cu(s) + 8H+(aq) + 2NO3(aq)


3Cu2+(aq) + 2NO(g) + 4H2O

0.34
+0.96
+0.62V

If you get nitric acid on your skin, you will not only
feel the burning of the acid (H+), but your skin will
be oxidized to a yellow-brown color! So
concentrated nitric acid is doubly dangerous!

REDOX 27

Although one can add half-cell rxns to yield overall


redox equations, one can not simply add two halfcell rxns to yield another half-cell rxn. For
example, consider the addition of the following two
half cell rxns to generate a third half cell rxn:
Cu2+(aq) + 1eCu+(aq) + 1eCu2+(aq) + 2e-

Cu+(aq)
Cu(s)
Cu(s)

+0.16 V
+0.52 V
+0.68 V

The correct half-cell potential for this rxn is 0.34 V,


which is exactly half of what we incorrectly
attempted to calculate above. This factor of
comes from the fact that all the half-cell potential
values are normalized to a single e- value even if
multiple e- are used in the half cell rxn.
As you might expect, the electrochemical potential
for a rxn is directly related to the G for a reaction
(only with an opposite sign relationship!):

G = nFE
n = # of electrons being transferred, F = Faradays
constant (96.5 kJ/Vmol), E = standard potential.
Problems: Calculate the cell potentials for the
following reactions. Are they spontaneous or not?

REDOX 28

a) F2(g) + 2Na(s)
b) Cu(s) + Zn2+(aq)
c) 2Al3+(aq) + 3Pb(s)

2Na+(aq) + 2F(aq)
Cu2+(aq) + Zn(s)
2Al(s) + 3Pb2+(aq)

d) 2Co3+(aq) + 2Cl-(aq)

2Co2+(aq) + Cl2(g)

e) 2Mg(s) + O2(aq) + 4H+(aq)

2Mg2+(aq) + 2H2O(aq)

f) 2Al(s) + 6H+(aq)

2Al3+(aq) + 3H2(g)

g) 2Cu(s) + Cl2(g)

2Cu+(aq) + 2Cl(aq)

REDOX 29

Non-Standard Conditions: Nernst Equation


E values are for standard conditions (1 M or 1
atm). If the reactant/product concentrations are
different the cell potential, E, will differ from E.
One can calculate non-standard potentials using the
Nernst equation:

2.303 RT
E= E
log Q
nF
R = gas constant, 8.314 J/molK
T = absolute temperature, K
n = # of moles of electrons transferred
F = faraday, 96,485 C/moleQ = reaction quotient
2.303 RT
= 0.0592 V imol
At room temp, 25 C ,
F
substituting this into the Nernst eq gives :
0.0592
E= E
log Q
n

REDOX 30

Example: Calculate the potential for the Fe3+/Fe2+


electrode when the concentration of Fe2+ is five
times greater than that of Fe3+.
Look up potential in a half cell table:
Fe3+ + eFe2+ E = +0.771 V
The Fe2+ concentration is five times greater than
that of Fe3+, so the Q expression is:

[products] p [Fe2+ ]
Q=
=5
r =
3+
[reactants] [Fe ]
Assume room temp (25C = 298K) and solve using
the Nernst equation:

0.0592
E= E
log Q
n
0.0592
E = 0.771
log(5)
1
E = 0.771 (0.0592)(0.7) = 0.730 V
Problem: The potential for this is lower than E

where both concentrations are 1 M. This should


make sense based on Le Chateliers principle.
Why?

REDOX 31

Problem: A cell is constructed at 25C as follows. One

half-cell consists of Cu2+/Cu, [Cu2+] = 0.4 M. The other


half-cell involves Zn2+/Zn with [Zn2+] = 0.4 M. Apply
the Nernst equation to the overall cell reaction to
determine the cell potential.

Problem: Consider the following half-cell rxn:


Pb(s) + SO42(aq)

PbSO4(s) + 2e-

E = +0.13 V

Can the Nernst equation help us figure out how to


change concentrations to increase the cell potential?
Why?

REDOX 32

Lead-Acid Storage Battery

Anode Rxn:
Pb(s) + SO42(aq)

PbSO4(s) + 2e-

E = +0.13 V

Cathode Rxn:
PbO2(s) + 4H+(aq) + SO42(aq) + 2ePbSO4(s) + 2H2O
E = +1.69 V

Overall Rxn:
Pb(s) + PbO2(s) + 4H2SO4(aq)
2PbSO4(s) + 2H2O
E = +1.82 V

REDOX 33

Dry Cell (regular battery)


Graphite
Cathode
Moist Paste
of NH4Cl,
MnO2 &
carbon
Porous
separator
Zinc anode

Note that the production of


NH3(g) in this regular cell
produces an insulating layer
between the cathode and the
MnO2/NH4+. This causes these
batteries to fade relatively
quickly after steady use.
Letting them sit for a while,
allows the NH3(g) to diffuse
away & allow fresh reagents to
come in contact with the
cathode, thus the battery
becomes rejuvenated.

Anode Rxn:
Zn

Zn2+ + 2e-

E = 0.76 V

Cathode Rxn:
2MnO2 + 8NH4+

2Mn3+ + 4H2O + 8NH3


E = 1.21 V

Overall Rxn:
Zn + 2MnO2 + 8NH4+

2Mn3+ + Zn2+ + 4H2O + 8NH3

Real batteries only produce 1.5 V because the real


chemistry is far more complicated and less efficient.

E = 1.97 V

REDOX 34

Alkaline Cells
Graphite
Cathode
Moist Paste
of KOH,
MnO 2 &
graphite
Porous
separator
Zinc anode

Anode Rxn:
Zn + 2OH
Cathode Rxn:
2MnO2 + 2H2O

The simple replacement of


NH4Cl electrolyte with a
KOH gel eliminates the
production of the problematic
NH3 gas that occurs in the
regular dry cell. This battery
has much improved stability
for delivering electrical
current until all the reagents
are used up.

Zn(OH)2 + 2e2MnO(OH) + 2OH

Overall Rxn:
Zn + 2MnO2 + H2O

2MnO(OH) + Zn(OH)2

E = 1.5 V

REDOX 35

Corrosion of Steel/Iron
Cathode
O2
Fe

Steel

2+

O2

Anode

Cathode: O2 + H2O + 2eFe2+ + 2eAnode: Fe


Sum:

Water

Fe 2+

O2 + Fe + H2O

2OH

Fe(OH)2

Subsequent Redox rxn:


4Fe(OH)2 + O2
2Fe2O3 4H2O

RUST

The presence of NaCl (salt) in the water makes it more


conductive to e- flow, making it easier for the O2 to oxidize the
iron without having to come directly in contact with it.
Instead it can react anywhere at the surface of the water drop.

REDOX 36

Electrolysis Electrolytic Cells


The process by which a chemical reaction is made to
occur by the passage of an electric current through
a solution is call electrolysis.

-+
power source

99.999%
pure Cu
plates out

Cu

2+

Cu

Cu
Cu

2+

Cu

Cu

2+

impure
copper
dissolves

2+

Cu
Cu

Cu

2+

2+

Cu

2+

2+

Cu

2+

Anode sludge
Ag, Au, Pt, Rh

REDOX 37

The amount of material plated out in an electrochemical cell is related to the # of electrons that
pass through the cell:
# moles deposited # moles of electrons
1 Faraday = 96,485 C/mol e1 Amp = 1 C/sec
The overall formula for calculating the number of
moles of material deposited in a electrochemical cell
given a certain current and time is:

(#amps)(time in seconds)
#moles =
(96,485 C / mole e-)(#e)

Remember that if the time is given in


minutes or hours that you have to first
convert it into seconds. Also remember
to convert amps to C/sec for the units to
work out correctly.

REDOX 38

Example: How many grams of Cu(s) will be

deposited from a solution of Cu2+ by a current of


1.5 amps flowing for 2 hours?
Cu2+ + 2eCu(s)
It takes 2 moles of electrons to produce 1 mole of
Cu(s) from one mole of Cu2+. So n = 2.
(1.5C/sec) (7200 sec)
# moles Cu =
(96,500 C/mol e-)(2 e-)

10,800 C
# moles Cu =
= 0.056 mol
193,000 C/mol
# grams of Cu is equal to the # moles times the
molecular (or atomic) weight of Cu:
(5.6 102 mol Cu) (63.5 g/mol Cu) = 3.55 g Cu

REDOX 39

Problem: How many moles of Al metal will be

electrodeposited if 9.6 amps is passed through a


solution of Al+3 for 10,000 seconds?

Problem: How long (in seconds) will it take to

electrodeposit 0.25 moles of Ti metal by passing a


current of 96.48 amps through a solution of Ti+4 ?

REDOX 40

Standard Reduction Potentials - 25C

REDOX 41

REDOX 42

Tables from Chemistry, 8th Ed, Whitten, Davis, Peck & Stanley (Thomson-Brooks/Cole)

CHEM 1422

Honors: General
Chemistry
Section 01

Previous 2 Years of
Blank Homeworks
& Exams
Prof. George G. Stanley
Department of Chemistry

CHEM 1202 - Homework # 1


Background
Due January 22, 2009 (2 PM)

Name: ___________________________
Signature: ________________________
Group: __________________________

Check the box to the right if you want your graded homework to be placed out in the public rack outside
Prof. Stanleys office. Otherwise you will have to pick up your homework from Prof. Stanley in person:

1. (5 pts) Why do or don't you like chemistry as it has been taught to you so far in high school and college?
What did or didnt you like? Don't try to suck up to me by writing only good things (unless they are true).
I'm looking for critical comments and reasoning. Tell me about things you have liked (if any) and disliked.
Short mindless answers will get no credit. This question is an individual, not group, effort. Please type up
your answer neatly, include your name, and E-mail it to me separately (gstanley@lsu.edu) as a Word
or PDF file (or body of the E-mail message).
2. (5 pts) Define electronegativity in your own words (dont just copy from a book). Give a couple of
elemental examples to illustrate your definition (i.e., elements with high or low values and how their
electronegativity affects their ability to gain or lose electrons).

3. (5 pts) Look up, list, and explain the trend in melting points in terms of intermolecular attractive forces for
the following compounds: CsI, LiF, BaS, MgO, and Al2O3. Make use of Coulombs Law (in a qualitative
sense) in your explanation.

CHEM 1422 - HW#1 Background (2009)

4. (5 pts) Consider the reaction: 2C(s) + 2Br2

Br2C=CBr2

If 120 g of carbon reacts with 160 g of Br2 and the reaction goes in 50% yield, how many grams of
Br2C=CBr2 (C2Br4) are produced? Clearly show and explain all your work.

5. (10 pts) Sketch out the Lewis dot structures for the following molecules (use lines for bonds and pairs of
dots for lone pairs). Indicate the formal charges, if any, on the appropriate atoms.
a) H2SO4

b) C2H2

c) AlCl3

d) NO3

e) P(CH3)3

CHEM 1202 - Homework # 2


Thermodynamics
Due Sept 19th, 2006

Name _____________________________
Signature: _________________________
Group Name: ______________________

Check the box to the right if you want your graded homework to be placed out in the public rack outside
Prof. Stanleys office. Otherwise you will have to pick up your homework from Prof. Stanley in person:

1. (12 pts) From the thermodynamic data given at the end of your lecture notes calculate Hrxn, Srxn and
Grxn for the following reactions. For Grxn please use the Grxn = Hrxn TSrxn formula with T =
298K. Indicate whether the reactions are spontaneous or non-spontaneous. Show your work.
a) 12NH3(g) + 21O2(g)

b) 2O3(g)

8HNO3(g) + 4NO(g) + 14H2O(g)

3O2(g)

C6H12O6(s) (glucose) + 6O2(g)


} this is photosynthesis!
c) 6CO2(g) + 6H2O(l)
Hf (C6H12O6) = 1274 KJ/mol Gf (C6H12O6) = 910 KJ/mol
S (C6H12O6) = 212 J/Kmol

2. (8 pts) For the following processes, is the entropy of reaction (Srxn) increasing, decreasing or staying
about the same? Use the qualitative rules about entropy discussed in lecture to determine the answer.
a) Ca(s) + H2SO4(aq)
b) 4Fe(s) + 3O2(g)
c) N2(g) + 3H2(g)

CaSO4(s) + H2(g)
2Fe2O3(s) (rust)
2NH3(g)

d) setting off a fire cracker (illegal in East Baton Rouge Parish)


e) AgCl(s) + Br(aq)
f) Mg(s) + CO(g)
g) Cu+2(aq) + 2OH(aq)
h) Ag+(aq) + Cu(s)

AgBr(s) + Cl(aq)
MgO(s) + C(s)
Cu(OH)2(s)
Ag(s) + Cu+(aq)

3. (5 pts) A typical instant hot pack uses 55 g of CaCl2(s) and 180 g of H2O, which react together to make
hydrated Ca2+(aq). Use the following formula to calculate the temperature (C) of the heat pack after it has
been activated (mixed together). Assume that the initial temperature of the water is 25C (room temperature),
Cp (water) = 75 J/mol K (watch your units!). Hrxn = 60 kJ/mol

T =

nsalt ( Hrxn)
nwater (Cpwater )

nwater = # of moles of water used


T = Tfinal - Tinital
nsalt = # of moles of salt dissolved Cp = heat capacity of solvent
Hrxn = enthalpy of dissolving salt in solution (also called Hsol)

4. (5 pts) (a) Using the Gibbs Free Energy formula to plot the value of Grxn from 500 K to 1200 K for the
reaction: 2HgO(s)
2Hg(l) + O2(g)
(use table at the end of the chapter -- you can assume that Hf and S do not vary much with temperature)
The vertical axis is energy in
kJ/mol -- you have to fill in
the range of units for this
axis based on your
calculations. (HINT: you
only have to calculate 3 to 5
data points to make the plot,
but you can also do this in
Excel and let it calculate
Grxn at many temps).
(b) What is the temperature
range for which this reaction
is spontaneous? Indicate on
plot.
(c) Mark and indicate the
temperature at which Grxn
= 0?

CHEM 1422 - Homework # 2


Thermodynamics
Due January 29, 2009

Name: ____________________________
Signature: _______________________
Group Name: _____________________

Check the box to the right if you want your graded homework to be placed out in the public rack outside
Prof. Stanleys office. Otherwise you will have to pick up your homework from Prof. Stanley in person:

1. (12 pts) From the thermodynamic data given at the end of your lecture notes calculate Hrxn, Srxn and
Grxn for the following reactions. For Grxn please use the Grxn = Hrxn TSrxn formula with T =
298K. Indicate whether the reactions are spontaneous or non-spontaneous. Show your work.
a)

Cl2(g) + H2(g)

b)

Fe(s) + 5CO(g)

c)

SiO2(s) + 6HF(g)

d)

CaCO3(s)

2HCl(g)

Fe(CO)5(l)

H2SiF6(aq) + 2H2O(l)

CaO(s) + CO2(g)

[glass is mainly SiO2]

HW#2 Thermodynamics (2009)

2. (8 pts) Is the entropy increasing, decreasing or staying about the same? Use the qualitative entropy rules
discussed in lecture to determine the answer. Write the answer to the right of each process.
a) Fe2O3(s) + Al(s)
b) raw egg

Fe(s) + Al2O3(s)
hard boiled egg

c) CaCl2(s) + 6H2O(l)
d) C6H12(l) + 9O2(g)
e) H2CO(aq) + H2O(l)

[Ca(H2O)6]Cl2(s)
6CO2(g) + 6H2O(g)
2H2(g) + CO2(g)

f) mowing the lawn


g) AgCl(s) + I(aq)
h) Sr(s) + 2H2O(l)

AgI(s) + Cl(aq)
Sr2+(aq) + 2OH(aq) + H2(g)

3. (2 pts) Circle the compound that has the highest entropy. Give a brief reason explaining your answer.
a) Hg(l)

b) H2O(l)

c) Pb(s)

d) C2H5OH(l)

e) CCl4(l)

4. (4 pts) Why does Al2O3(s) have a lower entropy than Fe2O3(s)? There are two primary qualitative reasons
for this. You may have to use the chemistry library to get more information (i.e., properties) on these two
common compounds to answer the question.

5. (4 pts) a) Small amounts of Fe(CO)5(l) usually form in steel tanks containing pressurized CO(g). You
worked out the thermodynamics of this in question 1b. At what temperature (C) will the formation of
Fe(CO)4 become non-spontaneous? Show your work. b) A similar reaction occurs to make Ni(CO)4(l) with
Grxn = 38 kJ/mol, Hrxn = 230 kJ/mol, and Srxn = 480 J/Kmol. High pressure reactors use a thin
disk of metal as a safety mechanism that will rupture and release gasses if the pressure in the reactor gets too
high. If one was using CO gas, which disk (Fe or Ni) would be more likely to prematurely fail due to the
metal being dissolved away by CO? Briefly explain why.

CHEM 1202 - Homework # 3


Chemical Kinetics
Due Tuesday, Sept 26, 2006 by 1 PM

Name _____________________________
Signed Name ______________________
Group Name: _____________________

1. (5 pts) Which of the following energy diagrams best represents a reaction which will be the fastest and most
spontaneous. Circle your choice and include your reasoning below (brief statement).
a)

b)

c)

d)
R

G
R

Rxn Coordinate

R
P

Rxn Coordinate

Rxn Coordinate

Rxn Coordinate

2. (5 pts) Consider the energy diagram to the right:


Circle the following diagram below that best represents the effect of adding a catalyst to
the above reaction. Include your reasoning below (brief statement).
a)

b)

c)

R
P

d)
Rxn Coordinate

G
R

Rxn Coordinate

Rxn Coordinate

Rxn Coordinate

Rxn Coordinate

3. (5 pts) Consider the following reaction that is quite important for the manufacturing of many chemicals:

3H2

cyclohexane
benzene
G = +200 kJ/mol Activation Energy = +400 kJ/mol

Circle the energy curve shown below (R = reactants, P = products) that best represents the reaction described
above? Include your reasoning below (brief statement).
a)

b)

G
P

Rxn Coordinate

Rxn Coordinate

c)

d)

Rxn Coordinate

P
R
Rxn Coordinate

HW #3 Kinetics (2006)

4. (5 pts) Consider the following reaction and kinetic data. Circle the correct kinetic rate expression for this
reaction. Clearly show and briefly discuss your reasoning.
A + B
Exp #

[A]

[B]

Initial Rate (Msec1)

1.5 M

1.5 M

0.2

1.5 M

3.0 M

0.8

3.0 M

3.0 M

0.8

6.0 M

3.0 M

0.8

a) rate = k[A][B]

b) rate = k[A]

c) rate = k[B]

d) rate = k[B]2

e) not enough data

5. (4 pts) Consider the following reaction and kinetic data. What is the rate constant for the reaction? Clearly
show all your work and reasoning.
H2NCH2CH2CH2CH2NH2 + 2HCl
Exp #

[(N)2]

[HCl]

Initial Rate (Msec1)

0.1

0.1

0.2

0.2

0.1

0.4

0.4

0.2

3.2

0.1

0.2

0.8

[H3NCH2CH2CH2CH2NH3]2+(Cl)2

6. (6 pts) The reaction in question 3 is non-spontaneous at room temperature.

3H2

cyclohexane
benzene
G = +200 kJ/mol Activation Energy = +400 kJ/mol

Discuss the one thing that you can do that will increase the rate of reaction and make the reaction more
spontaneous. Clearly discuss how this change will influence the rate of reaction and why it will affect the
thermodynamics (spontaneity) of the reaction.

CHEM 1422 - Homework # 3


Chemical Kinetics
Due Feb 12, 2009 (2 PM)

Name: ___________________________
Signature: _______________________
Group Name: ____________________

Check the box to the right if you want your graded homework to be placed out in the public rack outside
Prof. Stanleys office. Otherwise you will have to pick up your homework from Prof. Stanley in person:

1. (3 pts) Which of the following energy diagrams best represents the slowest spontaneous reaction? Circle
your choice. Give a brief, but clear, explanation for your answer below the diagrams.
a)

b)

c)

d)
R

G
R
Rxn Coordinate

P
R
Rxn Coordinate

R
P
Rxn Coordinate

P
Rxn Coordinate

2. (5 pts) a) Describe in your own words and terms where the origin of the activation barrier comes from and
what it represents in a chemical reaction. b) Given the same thermodynamic factors, consider the reaction of
two small molecules or two large molecules with one another. Which pair should have the higher activation
energy? Why?

HW#3 Kinetics (2009)

3. (3 pts) Consider the following reaction and information:


O
HO

C
H2

CH3

H2C

G = 100 kJ/mol

CH2 + CO + H2O

Activation Energy = +400 kJ/mol

Circle the energy curve shown below (R = reactants, P = products) that best represents the reaction described
above? Give a brief, but clear, explanation for your answer below the diagrams.
a)

b)

c)

d)

R
G R

P
Rxn Coordinate

P
Rxn Coordinate

Rxn Coordinate

P
R
Rxn Coordinate

4. (5 pts) Consider the following reaction and kinetic data. Circle the correct kinetic rate expression for this
reaction. Show all your work and/or discuss your reasoning.
2A + B
a) rate = k[A][B]

b) rate = k[A]2

c) rate = k[B]

Exp #

[A]

[B]

Initial Rate (Msec1)

0.2 M

0.1 M

0.02

0.4 M

0.1 M

0.04

0.2 M

0.3 M

0.18

0.4 M

0.3 M

0.36

C + D
d) rate = k[B]2

e) rate = k[A][B]2

HW#3 Kinetics (2009)

5. (5 pts) Consider the following reaction and kinetic data. Circle the correct rate constant for this reaction.
Clearly show all your work including the rate law that you determine.
A + 2B
a) 2.2 x 106 M1sec1

b) 22 M1sec1

c) 220 M1sec1

Exp #

[A]

[B]

Initial Rate (Msec1)

0.2 M

0.1 M

0.002

0.2 M

0.2 M

0.002

0.4 M

0.2 M

0.008

0.8 M

0.4 M

0.032

C+D
d) 0.05 M1sec1

e) not enough data

6. (4 pts) Catalysts can be used on non-spontaneous reactions to lower the activation barrier. If a catalyst
lowers the activation barrier too much, however, a serious problem can arise. Consider the diagrams shown
below. What is the problem for the catalyzed rxn with the lower activation energy? Why can a substantial
activation barrier actually help an uphill chemical reaction if one
wants to make as much product as possible?

7. (5 pts) A reaction has a initial rate of reaction of 0.001 Msec-1 at 70C. This increases to 0.100 Msec-1 at
90C. Calculate the activation energy for this reaction?

CHEM 1202 - Homework # 4


Chemical Equilibrium # 1
Due Thursday, Oct 19, 2006

Name ___________________________
Signed Name: ____________________
Group Name: ____________________

Check the box to the right if you want your graded homework to be placed out in the public rack outside
Prof. Stanleys office. Otherwise you will have to pick up your homework from Prof. Stanley in person:

2NO(g)
Keq = 4
1. (3 pts) Given the following information: N2(g) + O2(g)
Calculate the equilibrium constants for the equations shown below (you dont have to show your work):
a) 2NO(g)

N2(g) + O2(g)

b) N2(g) + O2(g)
c) 2N2(g) + 2O2(g)

NO(g)

Keq =
Keq =

4NO(g)

Keq =

2. (2 pts) Circle the equilibrium expression listed below that is the correct one for the following reaction:
SCl2(g) + H2O(g)
a)

[SCl2] [H2O]

Keq = [HCl] [SO]

b)

2HCl(g) + SO (g)

[SCl2] [H2O]

Keq = [HCl]2 [SO]

c)

[HCl] [SO]

Keq = [SCl ] [H O]
2
2

d)

[HCl]2 [SO]
Keq = [SCl ] [H O]
2
2

3. (4 pts) The initial concentrations for the following reaction are [CH3I] = [Cl] = 0 M, and [CH3Cl] = [I] =
2 M. What is the concentration of the reactant [Cl] at equilibrium? Show your work! Soln = solution
Initial:
@Eq:

CH3I(soln) + Cl(soln)

CH3Cl(soln) + I(soln)

Keq = 9

4. (6 pts) The initial concentrations of reactants = 0.2 M and products = 1.6 M. What is the concentration of
methanol (CH3OH) at equilibrium for the following reaction? Clearly show all your work!
Initial:
@Eq:

a) 0.1 M

CH3OH(g) + HI (g)

b) 0.3 M

CH3I(g) + H2O(g)

c) 0.6 M

d) 1.2 M

Keq = 25

e) 1.5 M

f) 3.0 M

Homework # 4 (Equilibrium #1) 2006

5. (2 pts) Consider the potential energy diagram to the right:


Which of the following Keq values and relative reaction rates fits best? Briefly and
clearly give your reasoning in the space below.
a) Keq = 1.5, fast rxn

c) Keq = 1 x 1056, fast rxn

b) Keq = 200, slow rxn

P
R
Rxn Coordinate

e) Keq = 1 x 104, slow rxn

d) Keq = 1 x 106, very fast rxn

6. (4 pts) Consider the following reaction shown below. We start with [Rh4(CO)12] = 0.2 M, [H2] = 1 M, and
[HRh(CO)3] = 0 M. When the reaction reaches equilibrium there is 0.02 M HRh(CO)3. Calculate Keq for
this reaction. Clearly show all your work.
Initial:

Rh4(CO)12 + 2 H2

4 HRh(CO)3

@ Eq:

7. (4 pts) Consider the following reaction:


P4(soln) + 5 NaClO2(soln)

P4O10(soln) + 5 NaCl(soln)

Keq = 1 1052

The initial concentrations of P4 and NaClO2 are both 0.5 M (no products present). Circle the concentration of
P4O12 at equilibrium? Clearly and briefly explain your answer below.
a) 0 M

b) 0.05 M

c) 0.1 M

d) 0.5 M

e) 0.75 M

f) 2.5 M

8. (5 pts) The initial concentration of [CO2] = 2 M, CaO(s) is present in excess, and there is no CaCO3(s).
Calculate the equilibrium concentration of [CO2].
Initial:
@ Eq:

CaO(s) + CO2(g)

CaCO3(s)

Keq = 10

CHEM 1202 - Homework # 5


Chemical Equilibrium # 2
Homework due Oct 26, 2006

Name ___________________________
Signature: _______________________
Group Name: ____________________

Check the box to the right if you want your graded homework to be placed out in the public rack outside
Prof. Stanleys office. Otherwise you will have to pick up your homework from Prof. Stanley in person:

1. (15 pts) Consider the following reactions:


A) 4Na(s) + O2(g)

2Na2O(s)

B) H2S(g) + Fe2+(aq)

FeS(s) + 2H+(aq)

C) 3NO2(g) + H2O(g)

2HNO3(g) + NO(g)

D) NH4NO3(s)

NH4+(aq) + NO3(aq)

E) 2Fe(CO)5(l)

Fe2(CO)9(s) + CO(g)

F) sugar(s)
G) H2O(l) + CO2(g)
H) Cl2(g) + H2(g)

Hrxn = 600 KJ/mol

sugar(soln)

Hrxn = +30 KJ/mol

Hrxn = 0 KJ/mol

H2CO3(aq)
2HCl(g)

exothermic

Based on the information above, which of the equilibria will (there can be more than one correct answer!!):

produce more products when heated? _____________

produce more products when the pressure is raised? _____________

produce more products when the pressure is lowered? _____________

be unaffected by adding or subtracting some product (so long as some remains)? ___________

produce more reactants when heated? _____________

produce more reactants when the pressure is raised? ___________

be generally unaffected by temperature? ________________

be unaffected by pressure? __________________

2. (5 pts) The initial concentrations for the following reaction are [AgCl(s)] = excess present, [NH3] = 2M,
[Cl] = 0.01 M, and [Ag(NH3)2+(aq)] = 0.01 M. What is the concentration of [NH3] at equilibrium?
Initial:
AgCl(s) + 2NH3(aq)

Ag(NH3)2+(aq) + Cl(aq)

Keq = 0.01

@Equilibrium:

3. (5 pts) Prof. Stanleys hydroformylation catalyst produces a 30:1 ratio of linear aldehyde product to
branched aldehyde product. If we assume that this represents an equilibrium ratio (i.e., Keq = 30 favoring the
linear aldehyde product), what is the G energy difference between the linear and branched products? Which
is more stable (lower in energy)? Please give your answer in KJ/mol.

4. (5 pts) What is the equilibrium concentration for [SO42] in the following reaction.
Initial:
Al2(SO4)3 (s)
@Equilib:

2Al+3(aq) + 3SO42(aq)

Ksp = 108 x 1015

CHEM 1422 - Homework # 4


Equilibrium
Due Tuesday, March 3, 2009 (2 PM)

Name: ___________________________
Signature: _______________________
Group Name: ____________________

Check the box to the right if you want your graded homework to be placed out in the public rack outside
Prof. Stanleys office. Otherwise you will have to pick up your homework from Prof. Stanley in person:

1. (3 pts) A reaction has an equilibrium constant of 1 106 and reaches equilibrium very slowly. Circle the
following potential energy diagram that best fits this data. Briefly and clearly discuss your reasoning.
a)

b)

c)

d)
R

Rxn Coordinate

Rxn Coordinate

P
Rxn Coordinate .

Rxn Coordinate

2. (3 pts) Consider the following equilibrium:


Keq = 1 1018
If one starts with 6 M acetylene (C2H2) and lets the reaction reach equilibrium, what will be the equilibrium
concentration of benzene (C6H6)? Circle the answer and clearly discuss your reasoning.
3C2H2(g)

a) 0 M

C6H6(g)

b) 0.6 M

c) 1 M

d) 2 M

e) 6 M

3. (3 pts) The initial concentrations of reactants and products are all 2 M. What is the concentration of
methanol (CH3OH) at equilibrium for the following reaction? Keq = 25 Circle the answer and clearly
show your work!!
CH3OH(g) + HI (g)
a) 0 M

b) 0.33 M

c) 0.66 M

CH3I(g) + H2O(g)
d) 1.00 M

e) 1.33 M

f) 2.66 M

CHEM 1422 HW # 4 Equilbrium

4. (4 pts) Calculate the concentrations for all species at equilibrium for the following reaction. The initial
concentrations are [H2] = [I2] = 0 M, [HI] = 4 M. Clearly show your work.
Initial:

H2(g) + I2(g)

2HI(g)

Keq = 36 (@ 1200 K)

@ eq:

5. (6 pts) Consider the following reactions:


A) Br2(aq) + 2Cl (aq)

Cl2(g) + 2Br(aq)

Hrxn = +68 kJ/mol

B) AgCl(s) + 2NH3(aq)

Ag(NH3)2+(aq)

Hrxn = 13 kJ/mol

C) 2N2O(g)

Hrxn = 30 kJ/mol

2N2(g) + O2(g)
2H2O(l) + SiF62 (aq) + 2H+(aq)

D) SiO2(s) + 6HF(aq)
E) Rh(H)(CO)(PPh3)2(aq) + CO(g)

Rh(H)(CO)2(PPh3)(aq) + PPh3(aq)

F) hemoglobin(aq) + 4O2(g)
G) 2H2O2(aq)
H) S2(aq) + Fe2+(aq)
I ) H2(g) + D2(g)

hemoglobin(O2)4(aq)

2H2O(l) + O2(g)

exothermic

FeS(s)
2HD(g)

Hrxn = 0 kJ/mol

Based on the information above, which of the equilibria will:

produce more products when heated? _____________

produce more products when the pressure is raised? _____________

be unaffected by adding or substracting some product (so long as some remains)? ___________

produce more reactants when heated? _____________

produce more reactants when the pressure is raised? ___________

be unaffected by temperature? _______

be unaffected by pressure? __________________

CHEM 1422 HW # 4 Equilbrium

6. (4 pts) The initial concentrations for the following reaction are [CH3I] = [F] = 1 M, and [CH3F] = [I] = 9
M. What will be the concentrations of each species at equilibrium? Clearly show all your work.
Initial:

CH3I(aq) + F(aq)

CH3F(aq) + I(aq)

Keq = 16

@Eq:

7. (4 pts) Which of the following salts is the least soluble (i.e., will give the lowest Pb+2(aq) concentration)?
Circle your answer. Calculate the concentration of [Pb+2] for the answer and put it and the calculation
details below.
a) PbCO3 (Ksp = 1 1013)

b) Pb3(AsO4)2 (Ksp = 1.1 1036)

c) Pb(CrO4) (Ksp = 1 1014)

d) Pb(OH)2 (Ksp = 4 1016)


f) PbS (Ksp = 1 1024)

e) Pb3(PO4)2 (Ksp = 1.1 1044)

8. (3 pts) What is the equilibrium concentration of Ag+(aq) in the presence of 1 M CrO42 (aq)? Clearly show
all your work.
Initial:

excess

Ag2CrO4(s)
@Equilibrium:

2Ag+(aq) + CrO42(aq)

Ksp = 4 x 1012

CHEM 1202 - Homework # 6


Acids & Bases # 1
Due Nov 14th, 2006 by 2PM

Name ___________________________
Signature: ____________________________
Group Name: _________________________

Check the box to the right if you want your graded homework to be placed out in the public rack outside
Prof. Stanleys office. Otherwise you will have to pick up your homework from Prof. Stanley in person:

1. (5 pts) What are the pHs for the following solutions?


a) 1 M HI =

b) 0.1 M HClO4 =

d) 0.001 M NaOH =

e) 1 1020 M Ba(OH)2 =

c) 10 M LiOH =

2. (5 pts) Why are strong acids strong acids. Specifically discuss the series HF, HCl, HBr, and HI and why HF
is a weak acid and HI is one of the strongest acids known.

3. (5 pts) Calculate the Ka value of stanoic acid (a monoprotic acid) if a 0.01 M solution has a pH = 4. Clearly
show all your work.

Homework # 6 Acids & Bases 1 (2006)

4. (5 pts) What is the pH of a 1 M solution of the base amyl amine. pKb = 8 Clearly show all your work.
Circle the correct answer from those given below. No credit will be given if work is not shown.

a) -2

b) 0

c) 4

d) 7

e) 10

5. (5 pts) Calculate the pH of a 0.1 M solution of an acid that has a pKa of 5.0. Clearly show all your work.
Circle the correct answer from those given below. No credit will be given if work is not shown.

a) 1

b) 1

c) 3

d) 7

e) 11

6. (5 pts) An unknown acid solution has a pH of 5. What important, but simple, piece of information do you
need to tell if the solution is from a strong or weak acid (aside from the Ka value)?

CHEM 1202 - Homework # 7 & 8


Acids & Bases 2
Due Tuesday, Nov 28th, 2006 by Noon

Printed Name: _____________________


Signature: ________________________
Group Name: _____________________

Check the box to the right if you want your graded homework to be placed out in the public rack outside
Prof. Stanleys office. Otherwise you will have to pick up your homework from Prof. Stanley in person:

Table 1. Dissociation Constants for some Acids.


Acid

pKa Value

Acid

pKa Value

Acid

pKa Value

NH4+

10

HBF4

formic

HClO

H2CO3

benzoic

1. (5 pts) Which of the acids listed in Table 1, given a 0.01 M solution in water, will have a pH closest to 2?
b) benzoic
c) H2CO3
d) formic
e) HBF4
a) NH4+
2. (5 pts) Which of the acids listed in Table 1, when reacted with an equivalent amount of NaOH, will form a
solution with the highest pH?
a) NH4+
b) benzoic
c) H2CO3
d) formic
e) HBF4
3. (5 pts)
a)
b)
c)
d)
e)

Order the acids in Table 1 from strongest to weakest. Circle the correct choice.
benzoic > formic > H2CO3 > HBF4 > NH4+ > HClO
HBF4 > formic > benzoic > H2CO3 > HClO > NH4+
NH4+ > benzoic > HBF4 > formic > H2CO3 > HClO
benzoic > HBF4 > NH4+ > formic > H2CO3 > HClO
NH4+ > HClO > H2CO3 > benzoic > formic > HBF4

4. (5 pts) What is the pH of a 0.01 M solution of the weak base benzylamine (C6H5CH2NH2)? pKa = 8.
Circle the answer below and clearly show all your work.

a) 4

b) 5

c) 9

d) 10

e) 13

5. (10 pts) Will FeCl3 generate an acidic, neutral, or basic solution when dissolved in water. Clearly discuss
your reasoning.

Homework # 7&8 Acids 2 (2006)

6. (5 pts) What is the pH of a 1 M solution of KClO? See Table 1 for pKa values. Clearly show all your work.

7. (10 pts) Consider the following list of salts:


A) NH4Cl

B) KI

C) CsF

D) potassium benzoate

E) MoCl4
I) KClO4

F) BaI2
J) NaClO

G) AlBr3

H) LiNO3

Which salts will generate an acidic solution? ________________


Which salts will generate a basic solution? _________________
Which salts will generate a neutral solution? _________________
8. (5 pts) Calculate the pKb of the weak base phenylamine if a 1 M solution has a pH = 10.

9. (10 pts) What is the pH if 800 mL of 0.125 M KOH is added to 200 mL of 0.5 M sucoloic acid (a
monoprotic acid)? pKa = 11 (clearly show all your work)

CHEM 1202 - Homework # 5


Acids & Bases # 1
Due Thursday, March 19 (by 2:00 PM)

Name: ___________________________
Signature: _______________________
Group Name: ____________________

Check the box to the right if you want your graded homework to be placed out in the public rack outside
Prof. Stanleys office. Otherwise you will have to pick up your homework from Prof. Stanley in person:

1. (3 pts) Consider the following weak acids and bases and their pKa values:
O

A)

(pKa = 3.8)

D) N(CH3)3 (pKa = 12)

B) HCN (pKa = 9.3)

C) H2CO3 (pKa = 6.4)

E) H2SO3 (pKa = 1.8)

F) NH3 (pKa = 9)

Which compound is the strongest acid (use letter) ? ___________


Which compound is the strongest base (use letter) ? ___________
Which compound has the strongest conjugate base (use letter) ? ____________
2. (5 pts) What are the pHs for the following solutions?
=

b) 10 M H2SO4 =

d) 0.1 M NaOH =

e) 10 M CsOH =

a) 0.1 M HBr

c) 1 1010 M HNO3 =

3. (4 pts) What is the pKa value of a 0.1 M solution of palmetic acid (HA) that has a pH of 5? Clearly show all
your work and put a box around your answer.

4. (3 pts) Calculate the pH of a 0.01 M solution of acid that has a pKa of 6.0. Clearly show all your work and
put a box around your answer.

CHEM 1422 HW # 5 Acids & Bases #1

5. (5 pts) What is the pH of a 0.1 M solution of the base ethyl amine (CH3CH2NH2). Ka = 1 1011 Clearly
show all your work and put a box around your answer.

6. (5 pts) What is the Kb of a 0.01 M solution of a base that has a pH of 10? Clearly show all your work and
put a box around your answer.

7. (5 pts) Which of the following acids is the strongest based on its structure and atoms present? Clearly
discuss your reasoning.
CH
F

H+

P
F

F
F

O
- or -

P
O

CH3

H+

CHEM 1422 - Homework # 6


Acids & Bases # 2
Due Tuesday, April 14, 2009

Name: ___________________________
Signature: _______________________
Group Name: ____________________

1. (5 pts) 250 mL of 0.2 M acetic acid (HOAc) reacts with 250 mL of 0.2 M NaOH. What is the pH of the
resulting solution? pKa (HOAc) = 5

2. (5 pts) Identify whether the following 1:1 solutions will be acidic, basic, or neutral:
a) NaOAc/HOAc (pKa = 4.7)
b) NH3/NH4NO3 (pKa = 9.3)
c) NaCl/KNO3
d) H3PO4/NaH2PO4 (pKa = 2.1)
e) CH3NH2/CH3NH3Cl (pKa = 10.6)
Table 1. Indicators
Name

pKa

Acid color

Base Color

Name

pKa

Acid color

Base Color

Methyl violet

yellow

violet

bromthymol blue

yellow

blue

methyl yellow

1.7

red

yellow

thymol blue

8.8

yellow

blue

methyl orange

3.5

red

yellow

phenolphthalein

colorless

pink

red

yellow

Alizarin yellow

11

yellow

red

methyl red

3. (3 pts) What is the approximate pH of a colorless solution that turns yellow if a small amount of methyl
yellow, or alizarin yellow is added to it, but turns pink if phenolphthalein is added. See Table 1 for
information about indicators. Circle your answer.
a) 0 to 1

b) 4 to 5

c) 7 to 8

d) 9 to 10

e) 13 to 14

4. (2 pts) Which indicator (see Table 1) would work best to indicate the equivalence point for the titration
12
curve shown to the right (circle answer):
10

a) methyl violet

b) methyl red

c) thymol blue
pH

d) phenolphtalein

e) alizarin yellow

8
6
4
2
0
0

10

20

30

40

mL of HCl added

50

CHEM 1422 HW # 6 Acids/Bases 2

5. (3 pts) What is the pH of a 0.01 M solution of NaHCO3? pKb = 4. Clearly show all your work.

6. (4 pts) Consider the following list of salts:


A) CsNO3

B) TiCl4

C) Na(SH)

D) KF

E) Ca(OH)2

F) Ti(ClO4)4

G) [HN(CH3)3]Cl

H) LiHCO3

I) NaPF6

J) RbBr

Which salts will generate acidic solutions? ________________


Which salts will generate basic solutions? _________________
Which salts will generate neutral solutions? _________________
7. (4 pts) What is the pH if 500 mL of 0.2 M HCl is added to 500 mL of 0.2 M ammonia (NH3, pKb = 5)
(clearly show all your work)

8. (4 pts) What is the pH of the following 0.5 M aqueous solutions containing equal amounts of the two
components shown (find pKa/b or Ka/b values in your textbook appendix or lecture notes):
a) NH4Cl + NH3
b) HNO2 + KNO2
c) H2S + CsHS
d) C5H4N (pyridine) + [C5H4NH]Br

CHEM 1202 - Homework # 9 & 10


Redox & Electrochemistry
Due Friday, Dec 8th, 2006 by Noon

Printed Name: ____________________


Signature: ________________________
Group Name: _____________________

Check the box to the right if you want your graded homework to be placed out in the public rack outside
Prof. Stanleys office. Otherwise you will have to pick up your homework from Prof. Stanley in person:

1. (5 pts) Which of the following substances is the best reducing agent?


a) F

b) Mg

c) Li+

d) Ag+

e) Zn

2. (5 pts) Which of the following substances is the best oxidizing agent?


a) F

b) Mg2+

c) O3

d) Ag+

e) Cu

3. (10 pts) Balance the following reaction in acidic solution.


S2O32(aq) + OCl(aq)

S4O62(aq) + Cl(aq)

4. (10 pts) Balance the following reaction in basic solution:


BiO3(aq) + Cr3+(aq)

Bi3+(aq) + CrO42(aq)

5. (5 pts) Write the oxidation state for the underlined element in the box following each compound.
a) LiAlH4

b) Ba3(AsO4)2

d) CaSO3

e) H2O2

c) Na2NiCl4

Homework 9&10 Redox/Electrochemistry (2006)

6. (15 pts) Calculate the redox potentials for the following reactions. Show the two half cell reactions, written
in the proper direction and their potentials used to calculate your answer.
a) H2(g) + 2Li(s)

2H(soln) + 2Li+(soln)

b) 4H+(aq) + O2(g) + 2Cu(s)

c) F2(g) + 2Cl(aq)

2H2O + 2Cu2+(aq)

2F(aq) + Cl2(g)

E =

E =

E =

d) Cu(s) + 2Ag+(aq)

Cu2+(aq) + 2Ag(s)

E =

e) 3Pb2+(aq) + 2Al(s)

3Pb(s) + 2Al+3(aq)

E =

7. (10 pts) Library/web research topic: Describe in your own words the chemistry (with formulas) involved in
a lithium-ion battery. Is lithium metal used? What is the voltage of this electrochemical reaction? List two
main advantages and two main disadvantages of lithium-ion batteries with BRIEF explanations. DO NOT
COPY DIRECTLY FROM ANY REFERENCE (except for chemical formulas). List your primary reference
used at the end.

CHEM 1422 - Homework # 7


Redox & Electrochemistry
Due Tuesday, April 21 (4 PM)

Name: ___________________________
Signature: _______________________
Group: ___________________________

Check the box to the right if you want your graded homework to be placed out in the public rack outside
Prof. Stanleys office. Otherwise you will have to pick up your homework from Prof. Stanley in person:

1. (1 pt) Which of the following substances is the best reducing agent? Briefly explain your answer.
a) Na+

b) Zn

c) Li+

d) Ag

e) Al

2. (1 pt) Which of the following substances is the best oxidizing agent? Briefly explain your answer.
a) O2

b) Li+

c) Cl2

d) Ag+

e) F

3. (1 pt) Which of the following substances is the best reducing agent? Briefly explain your answer.
a) F2

b) Mg

c) Li+

d) Na

e) Zn2+

4. (1 pt) Which of the following substances is the best oxidizing agent? Briefly explain your answer.
a) H+

b) Al3+

c) Ag+

d) Li

e) O3

5. (3 pts) Balance the following reaction in acidic solution (add water or H+ as needed). Clearly show your
work.
I2(aq) + MnO2(s)
I (aq) + MnO4(aq)

CHEM 1422 HW # 7 Redox & Electrochemistry

6. (3 pts) Balance the following reaction in basic solution (add water or OH as needed). Clearly show your
work.
Cr2O72(aq) + ClO(aq)
Cr3+(aq) + ClO4(aq)

7. (5 pts) Write the oxidation state for the underlined element in the box following each compound.
a) NaH

b) KNO3

d) Ca3(PO3)2

e) Na(NCS)

c) Na2PtCl6

8. (5 pts) Calculate the redox potentials for the following reactions. Show the two half cell reactions used to
calculate the overall potential.
a) 3H2(g) + 2Al3+(aq)

6H(aq) + 2Al(s)

b) 2AgCl(s) + Mg(s)

2Ag(s) + Mg2+(aq) + 2Cl(aq)

c) 2F2(g) + 2H2O(l)

d) Mg2+(aq) + Cu(s)

e) Li(s) + Ag+(aq)

4F(aq) + 4H(aq) + O2(g)

Mg(s) + Cu2+(s)

Li+(aq) + Ag(s)

CHEM 1422 HW # 7 Redox & Electrochemistry

9. (4 pts) A MgCl2 solution containing a Mg electrode is connected by means of a salt bridge to a CuCl2
solution containing a copper electrode. Sketch out a Galvanic Cell showing this and clearly indicate the
movement of anions, cations, electrons, which electrode is dissolving, and which is forming a metallic
deposit. Label the anode and cathode and show the cell potential.

10. (3 pts) How long (in hours) will it take to electrodeposit 1 mole of Al metal by passing a current of 9.65
amps through a solution of Al+3 ? Please clearly show all your work and put a box around your final answer.

11. (3 pts) What is the concentration of [Ag+] in a half-cell if the reduction potential of the Ag+/Ag couple is
observed to be 0.40 V? Clearly show all your work and put a box around your final answer.

CHEM 1422 - Homework # 8


Organic & ChemDraw
Due Friday, May 1 (by 3:00 PM)
Please download ChemDraw from Tigerware (located under Scientific Software, Chemistry software) and
install (along with the included Chem3D program) on your Windows or Macintosh computer. You need to
register with CambridgeSoft using an LSU e-mail address in order to get a license key (serial #) for installing
ChemDraw. I sent instructions on this via e-mail. Study groups can work together, as usual, but each student
needs to E-mail their own typed report with ChemDraw structures to Prof. Stanley as a Word or PDF file.
Prof. Stanley is available to answer questions about ChemDraw.
1. (20 pts) Sketch out nice ChemDraw structures for the following molecules. Each should be shown in
standard organic line notation and with all atoms identified.
Example:

H2C

tetrahydrofuran:

CH2

H2C

CH2

H
Or ganic line
notation

C
H

Tw o possible w ay s of drawing
"complete" str uct ur es

a) 2-hexanone

b) heptanal

c) Z-2-octene

d) E-2-octene

e) dimethylformamide

f) 2-pentyne

g) benzoic acid

h) 6-ethyl-3-methylnonane

i) 1,3-diethylbenzene

j) pyridine

2. (10 pts) Name and redraw the following molecules:

a)

b)

e)
d)

NH

c)

CHEM 1422 - Homework # 9-10


Gaussian & GaussView
Due Thursday, April 30 (by 4:00 PM)
Please download Gaussian 03 and GaussView 4 from Tigerware (located under Scientific Software, Chemistry
software) and install on your Windows computer. Up to 3 people can work together on this assignment.
Macintosh users should pair up with those with PC/Windows computers. Submit one copy of your report and
dont forget to put all the names of those working together on the report. All reports should be typed, formatted
nicely, and include color images (when possible).
Prof. Stanley is available to answer questions about Gaussian & GaussView. Separate instructions for using
GaussView and Gaussian have been posted by Prof. Stanley.
The last part of this assignment is the same as that given to the 1431 Honors Chemistry Lab students.
This double assignment will count for 60 pts.
1. (40 pts) Do DFT molecular orbital calculations on perchloric, sulfuric, and phosphoric acids (optimize, DFT,
B3LYP, 6-311G(d) basis set). Make sure you select a single d function on your basis set.

Questions:
a) Capture and display the optimized molecules with their Mulliken charges.
b) Show the electrostatic surface potential plots using a density value of 0.04. Adjust the color scale for
perchloric acid to best represent the atomic charges, then use the same numerical ranges for displaying the
other two molecules.
c) Do the charges tell you anything about the acidity of the molecule? Note that the calculation is done in
vacuum with no solvent molecules around a proton will not dissociate without water molecules to
interact with. Discuss any correlation of the charges calculated with the acidity of the molecules.
d) List and compare the bond distances for the 3 molecules in table format and discuss how they compare to the
localized structures drawn above. Do the bond distances fit any periodic trends?
e) Show the highest occupied molecular orbital (HOMO) for each molecule along with its energy in eV.
Discuss and explain the trend (if any) in the energies of the HOMOs (lower is more stable) for these three
molecules with periodic properties and what we discussed in class for oxyacids. Discuss what kind of
bonding, non-bonding (lone pair), or antibonding interactions are occurring within the HOMO for each
molecule. Although one might expect hybrid orbitals (e.g., sp3) for some of these, MO calculations often
tend to display orbitals as more or less pure s, p, etc. For example, an oxygen lone pair is more likely to
show up as a p orbital.

2. (20 pts) Shirakura & Suginome published a paper in the Journal of the American Chemical Society (2008,
130, 5410-5411) on nickel catalyzed coupling of silylacetylenes with dienes:

The first step in the catalysis is the oxidative addition (breaking) of the alkyne C-H bond to the nickel atom to
make a nickel-hydride-alkynyl complex:
H
Me3Si

Ni

Ni

SiMe3

In order for a C-H bond to do an oxidative addition to a Ni metal center the orbital associated with the C-H
bond must be reactive enough to want to participate in this reaction. The higher the energy of the orbital the
more reactive it will be (usually).
This reaction apparently does not work for regular alkynes that do not have silyl groups attached.
Use Gaussian (optimization, DFT, B3LYP method, 6-311G basis set) to perform calculations on the following
alkyne molecules:

Questions:
a) Identify the highest energy (C)C-H bonding orbital (# from calculation), list the energy of each in eV, and
show a picture of this molecular orbital for each alkyne.
b) How does the energy of the orbital from question # 2 affect the C-H bond strength? How does the group
attached to the other side of the alkyne affect this energy?
c) List and compare the energies (in eV) of the alkyne -bonding orbitals for each system and show a picture
of one of these molecular orbitals for each alkyne. Although these arent directly involved in the C-H bond
breaking step discussed above, what factor seems to be affecting the energy of these orbitals?

CHEM 1202 Exam # 1A (Sept 28, 2006)

Name (printed):_________________________

Thermo & Kinetics

Name (signed):_________________________

Put a big X this box if you want your graded exam put out in the public racks
outside Prof. Stanleys office after grading. If you dont check it, Prof. Stanley will
keep your exam and you will have to stop by to pick it up from him personally.
Answer Sheet -- Please Hand In!
1. _____

2. _____

3. _____

4. _____

5. _____

6. _____

7. _____

8. _____

9. _____

10. _____

11. _____

12. _____

13. _____

14. _____

15. _____

16.

17.

18.

19.

Bonus:

CHEM 1202 Exam # 1A

Sept 28, 2006

Please Answer All Questions on the Answer Sheet


1. (4 pts) Qualitatively, for which of the following reactions will Srxn increase the most?
a) Mg(s) + 2H+(aq)

Mg2+(aq) + H2(g)

c) 2Al3+(aq) + 3SO42(aq)

Al2(SO4)3 (s)

b) 2FBr(g)

F2(g) + Br2(g)

d) H2C2O4(aq)

H2(g) + 2CO2(g)

2. (4 pts) Which of the following substances has the highest Entropy?


a) CH4(g)

b) C3H8(g)

c) CH3OH(l)

d) PbCl2(s)

e) LiOH(s)

3. (4 pts) Which of the following statements about G is FALSE?


a)
b)
c)
d)
e)

G = H TS
G is at standard conditions (1 atm, 1 M)
spontaneous reactions have + Srxn
G represents the free or available energy available in a reaction to perform work
spontaneous reactions with negative Srxn must be exothermic

4. (4 pts) Calculate Grxn for the following reaction: N2(g) + O2(g) Error! Objects cannot be created from
editing field codes. 2NO(g)
Gf (NO(g)) = +90 kJ/mol
a) 180 kJ/mol

b) 90 kJ/mol

c) +90 kJ/mol

d) +180 kJ/mol

e) +270 kJ/mol

5. (4 pts) Consider the reaction: 2H2S(g) + 3O2(g) Error! Objects cannot be created from editing field
codes.2SO2(g) + 2H2O(l) Hrxn = 1126 kJ/mol and Grxn = 1006 kJ/mol. Which of the
following best describes the reason for the difference between Hrxn and Grxn?
a)
b)
c)
d)
e)

because the surroundings are absorbing the energy difference


because the energy difference is going into the Srxn
because there is an element in its natural state as one of the reactants
because water has a high heat capacity
because some of the energy is given off as light, not heat

6. (4 pts) Calculate Srxn for the following reaction: 2CH4(g) + O2(g) Error! Objects cannot be created from
editing field codes.2CH3OH(g)
S (CH4) = 186 J/molK, S(O2) = 205 J/molK, S(CH3OH) = 238 J/molK
c) +101 J/molK
d) +153 J/molK
a) 101 J/molK
b) 153 J/molK

e) +358 J/molK

7. (4 pts) How much energy is needed to raise the temperature of 1000 gm of water (1 liter) from 25C to 100C?
Specfic heat capacity of water 4 Jgm1C1.
a) 3 kJ

b) 30 kJ

c) 300 kJ

d) 30,000 kJ

The following potential energy diagrams are to be used for problems 8-11.

e) 300,000 kJ

8. (4 pts) Which potential energy diagram best represents the fastest spontaneous reaction?
9. (4 pts) Which potential energy diagram best represents the slowest spontaneous reaction?
10. (4 pts) Which potential energy diagram best represents a non-spontaneous reaction that is proceeding through two
intermediate species?
11. (4 pts) For which spontaneous potential energy diagram will the rate have the greatest temperature dependence?
12. (4 pts) What is the kinetic rate law for the following data:

Exp #

[A]

[B]

Rate (Msec1)

a) rate = k[A]2

b) rate = k[A]2[B]

1.5

1.5

0.2

c) rate = k[A][B]2

d) rate = k[A]

1.5

3.0

0.4

e) rate = k[B]2

f) rate = k[A][B]

3.0

3.0

1.6

3.0

6.0

3.2

13. (4 pts) The rxn: CO + 2H2Error! Objects cannot be created from


editing field codes.CH3OH has the following rate law: rate = k[CO]1[H2]. Calculate the rate constant k if rate
= 4 Msec1 when [CO] = 2 M and [H2] = 4 M.
a) 0.5 Msec1

b) 1 Msec1

c) 2 Msec1

d) 4 Msec1

e) 8 Msec1

14. (4 pts) Which of the following rate laws is consistent with a bimolecular reaction between A & B?
a) rate = k[A][B] b) rate = k[A]2[B] c) rate = k[A][B]2

d) rate = k[A]0.5[B]1.5 e) rate = k[B]2

15. (4 pts) Which of the following statements about the activation energy (or barrier), Ea, is FALSE?
a) The activation energy is related to the probability of the reactant molecules reacting.
b) Catalysts lower the activation energy by providing a better mechanism for the reactants.
c) The activation energy is related to the size/shape of the reacting molecule and strength of the chemical bonds.
d) Raising the temperature lowers the activation energy.
e) A spontaneous reaction with no Ea will be extremely fast and have little temperature dependence.
16. (10 pts) Consider the following reaction: 2NO(g) + N2(g) Error! Objects cannot be created from editing
field codes. 2N2O(g)
At 298 K Grxn = +35 kJ/mol. Hf (NO) = +90 kJ/mol, Hf (N2O) = +82 kJ/mol, S(NO) = 210 J/molK,
S(N2) = 190 J/molK, and S(N2O) = 220 J/molK. (a) At what temperature will G = 0 and below which the
reaction will be spontaneous? Clearly show all your work. (b) Why does one have to go to a lower temperature
to get a spontaneous reaction?
17. (10 pts) S for sucrose (sugar), C12H22O11(s), is 392 J/molK. S for butane, C4H10(g), is 310 J/molK.
(a) Why does butane, which is a gas, have a lower entropy than sucrose, which is a solid? (b) How should you
modify the statement that gases have higher entropies than solids to make it consistent with the above example?
18. (10 pts) Calculate the rate constant k for the reaction 2NO(g) + N2(g) Error! Objects cannot be created from
editing field codes. 2N2O(g) using the data below. Clearly show all your work and include the correct units on k.
Exp #

[NO]

[N2]

Initial Rate (Msec1)

1M

2M

0.1

4M

2M

1.6

4M

4M

1.6

19. (10 pts) (a) Sketch out a potential energy (G) diagram with the following information: Grxn = 40 kJ/mol,
one intermediate with an energy 10 kJ/mol lower than the reactant energy, Ea1 = 10 kJ/mol, Ea2 = 20 kJ/mol.
Label the product with a P, and the transition state energies Ea1 and Ea2. (b) Indicate with an arrow and label

with RDS the rate determining point (step) of this energy diagram. I indicated the energy of the reactant (R) on
the plot, you need to draw out the rest of the diagram.
BONUS (5 pts): Why are perpetual motion machines impossible?

CHEM 1422 Exam # 1 (Feb 19, 2009)

Name (printed):_________________________

Thermo, Kinetics & 1st part of Equilibrium

Name (signed):_________________________

Put a big X this box if you want your graded exam put out in the public racks outside Prof. Stanleys
office after grading. If you dont check it, Prof. Stanley will keep your exam and you will have to stop by
to pick it up from him personally.

1. (5 pts) Qualitatively, for which of the following reactions will Srxn increase the most? Circle the correct answer
and provide clear justification/reasoning for your choice in the space to the left of the answers.
a) Cu2+(aq) + Zn(s)

Cu(s) + Zn2+(aq)

b) AgCl(s) + Br(aq)
c) 3C2H2(g)
d) 2HgO(s)
e) 2SO2(g) + O2(g)

AgBr(s) + Cl(aq)

C6H6(l)
2Hg(l) + O2(g)
2SO3(g)

2. (5 pts) Which of the following substances has the highest absolute Entropy? Circle the correct answer and
provide clear justification/reasoning for your choice in the space to the right of the answers.
a) H2Te(g)
b) H2S(g)
c) NH3(g)
d) H2O(l)
e) LiBH4(s)
3. (5 pts) What is Hrxn for the following reaction? Hf (PCl3) = 320 kJ/mol; P4 = elemental P. Circle the
correct answer and provide clear justification/reasoning for your choice in the space to the right of the answers.
P4(s) + 6Cl2(g)

4PCl3(l)

a) 1280 kJ/mol
b)

320 kJ/mol

c)

0 kJ/mol

d) 320 kJ/mol
e) 1280 kJ/mol
5. (5 pts) Calculate Srxn for: 3O2(g)
2O3(g)
S(O2) = 205 J/mol.K, S(O3) = 240 J/mol.K Circle the correct answer and provide clear justification/reasoning
for your choice in the space to the right of the answers.
a) 620 J/mol.K
b) 310 J/mol.K
c) 135 J/mol.K
d) 310 J/mol.K
e) 620 J/mol.K

CHEM 1422 - Exam # 1 (Thermo, Kinetics, Equilibrium)

The following potential energy


(PE) diagrams are to be used
for problems 6 - 8. They can
be used more than once.

6. (5 pts) Which potential energy diagram best represents a spontaneous reaction that will have the largest
temperature dependence with regards to its rate of reaction? Indicate the letter for the diagram and provide clear
justification/reasoning for your choice.

7. (5 pts) Consider the following reaction mechanism for converting reactants A + B + C into product F. Indicate the
letter for the potential energy diagram that best matches this spontaneous reaction and provide clear
justification/reasoning for your choice.
Step # 1: A + B

D (slow step)

Step # 2: D + C

E (fast; species E observable)

Step # 3: E

8. (5 pts) Which potential energy diagram best represents the reaction that will rapidly form an equilibrium mixture
with roughly equal amounts of reactants and products? Indicate the letter for the PE diagram and provide clear
justification/reasoning for your choice.

9. (5 pts) What is the kinetic rate law for the following data.
Circle the correct answer and provide clear
justification/reasoning for your choice.

Exp #

[A]

[B]

Rate (Msec1)

1.5

1.5

0.2

a) rate = k[A][B]

b) rate = k[A]2[B]

1.5

3.0

0.8

c) rate = k[A][B]2

d) rate = k[A]

3.0

4.5

3.6

e) rate = k[B]2

f) rate = k[A]2

3.0

3.0

1.6

CHEM 1422 - Exam # 1 (Thermo, Kinetics, Equilibrium)

10. (5 pts) Consider the following rxn: 4C3H5N3O9(l)


6N2(g) + 12CO2(g) + 10H2O(g) + O2(g)
Hrxn = 5646 kJ/mol and Ea = 10 kJ. Based on this thermodynamic and kinetic data what can you tell me
about the reaction: energy given off (or absorbed), likely form of the energy (heat, light, nuclear, electrical, etc),
spontaneity, and likely speed of the reaction. Would you want to be around 10 moles of the reactant when it
reacts ?

11. (10 pts) The second law of thermodynamics says that the entropy of the universe always increases for a
spontaneous reaction. But a spontaneous reaction can have a Srxn term that is negative, i.e., the entropy of the
reaction decreases. Explain why this does not violate the second law of thermodynamics.

MgO(s) + C(s)
12. (10 pts) I did the burning Mg demonstration outside for the class: Mg(s) + CO2(g)
Hrxn = 811 kJ/mol. When I used the propane torch to light the Mg(s) sitting on the paper on top of the dry
ice, CO2(s) (temp = 79C; CO2(s)
CO2(g)), the Mg barely started burning. But as soon as I dropped more
dry ice onto the Mg pile it really took off and reacted vigorously. There are two kinetic factors that work in
opposite directions that affect this rxn, but one is clearly more important. What are these two kinetic factors and
how do they affect the rxn rate?

CHEM 1422 - Exam # 1 (Thermo, Kinetics, Equilibrium)

13. (10 pts) Calculate the rate constant for the following reaction. Clearly show all your work.
2C2H4(OH)2(aq) + 5O2(g)

4CO2(g) + 6H2O(g)

Exp #

[C2H4(OH)2]

[O2]

Rate
(Msec1)

0.1

0.01

2.1 104

0.1

0.02

8.4 104

0.1

0.03

1.9 103

0.4

0.03

7.6 103

14. (10 pts) Consider the following equilibrium: N2(g) + 3H2(g)


2NH3(g)
The initial concentrations are: [N2] = 2 M; [H2] = 1 M, [NH3] = 10 M. At equilibrium [N2] = 3 M? Calculate
Keq for this reaction. Clearly show all your work.

15. (10 pts) Consider the reaction: F2(g) + Cl2(g)

2FCl(g)

Keq = 64

If one starts with [F2] = [Cl2] = 0, and [FCl] = 10 M, what will be the concentration of FCl @ equilibrium?
Clearly show all your work!

CHEM 1202 Exam # 2 Make-UP


Equilibrium & Acids/Bases

Name (printed) _____________________


Name (signed) _____________________

Put a big X this box if you want your graded exam put out in the public racks
outside Prof. Stanleys office after grading. If you dont check it, Prof. Stanley will
keep your exam in his office and you will have to stop by to pick it up from him
personally.
Answer Sheet -- Hand In! Please use CAPITAL letters for your multiple choice answers!
1. _____

2. _____

3. _____

4. _____

5. _____

6. _____

7. _____

8. _____

9. _____

10. _____

11. _____

12. _____

13. _____

14. _____

15. _____

16.

17.

Exam # 2A (2006)

18.

19.

Exam # 2A (2006)

CHEM 1202 Exam # 2 - Makeup

November 2006

Please Answer All Questions on the Answer Sheet


1. (4 pts) Calculate Keq for the following reaction: 2NH3(g) + 2H2O(g)
2NO(g) + 5H2(g)
Equilibrium concentrations: [NH3] = 4 M, and [H2O] = 4 M, [NO] = 0.4 M, [H2] = 2 M
a) 2 105

b) 0.02

c) 1

d) 20

e) 50

2. (4 pts) For which of the following reactions will decreasing the overall pressure make more products?
a) N2(g) + 2H2(g)
b) Br2(g) + I2(g)

Hrxn = 2 KJ/mol

2BrI(g)
3H2O(l) + IO3(aq)

c) 3H2O2(aq) + I(aq)

Ag(NH3)2+(aq) + Cl(aq)

d) AgCl(s) + 2NH3(aq)
e) CH3CH3(g)

Hrxn = 95 KJ/mol

H2N=NH2(g)

H2C=CH2(g) + H2(g)

Hrxn = +10 KJ/mol


Hrxn = +120 KJ/mol

3. (4 pts) Heating which of the reactions listed in #2 above by the same amount will cause the largest shift in
equilibrium favoring products?
4. (4 pts) Consider the equilibrium: 2Rh3+(aq) + 3Cl(aq) + 3H2(g)
2Rh(s) + 6HCl(aq).
3
Keq = 1 10 . Which of the following statements about this reaction is FALSE?
a) The reaction favors the reactants.
b) This reaction is spontaneous under standard conditions.
c) Increasing the pressure will make more product.
d) G is negative for this reaction.
e) The addition of Rh(s) will have no effect on the reaction so long as some is present.
5. (4 pts) What is the G value that corresponds to Keq = 2.44 103 at 300 K. R = 8.314 J/molK
a) 150 kJ/mol

b) 75 kJ/mol

c) 25 kJ/mol

6. (4 pts) Calculate Keq for this reaction: 3NO2(g) + H2O(g)

d) 0 kJ/mol

e) 15 kJ/mol

2HNO3(g) + NO(g).

Initially we start with [NO2] = 4 M and [H2O] = 1 M (no products). When the reaction reaches equilibrium there
is 0.5 M NO product.
a) 3.2 104

b) 6.4 102

7. (4 pts) Consider the reaction: 3SO3(g)

c) 0.4
3SO2(g) + O3(g)

d) 40

e) 3.2 106

Keq = 1 x 1025

The initial concentrations of SO2 = O3 = SO3 = 2 M. What is the [SO2] concentration at equilibrium?
a) 0 M

b) 1 M

8. (4 pts) F2(g) + Cl2(g)

c) 2 M
2FCl(g)

d) 4 M

e) 6 M

Keq = 64

If one starts with [F2] = [Cl2] = 0, and [FCl] = 10 M, what will be the concentration of FCl @ equilibrium?
a) 1 M

b) 4 M

c) 8 M

d) 10 M

e) 12 M

Exam # 2A (2006)

9. (4 pts) 2NH3(aq) + 3I2(s)

2NI3(s) + 3H2(g)

Keq = 16

Initial concentrations: [NH3] = 2 M, [I2] = excess, [NI3] = excess, [H2] = 4 M.


What is the concentration of [H2] at equilibrium?
a) 0.5 M

c) 6

b) 4 M

d) 8 M

e) 10 M

10. (4 pts) Consider the potential energy diagram to the right. Which of the following
equilibrium constants best fits this reaction?
a) 1 1010

b) 0.01

d) 1 1069

c) 10

11. (4 pts) Consider the reaction: 2HRe(CO)5(aq)

Re2(CO)10(aq) + H2(g)

Initially, [Re2(CO)10] = [H2] = 1 M (no reactant). What is HRe(CO)5 at equilibrium? Keq = 4


a) 0 M
b) 0.2 M
c) 0.4 M
d) 0.8 M
e) 2 M
12. (4 pts) Which of the following salts will be the least soluble in H2O (give the lowest concentration of [Ag+])?
a) AgCl (Ksp = 1 1010 )

b) AgCN (Ksp = 1 1016 )

c) AgI (Ksp = 1 1016 )

d) AgBr (Ksp = 1 1013 )

e) Ag2S (Ksp = 1 1049)

f) AgF (Ksp = 1 106 )

13. (4 pts) Which of the following is the weakest base?


a) KOH

b) Sr(OH)2

c) NaOH

d) LiOH

e) Be(OH)2

14. (4 pts) If water has a pO2 = 7, what is the O2 concentration represented by this?
a) 1 x 1013 M

b) 1 x 107 M

c) 1 x 103 M

d) 0.3 M

e) 3 M

c) 1

d) 3

e) 10

15. (4 pts) What is the pH of 10 M HNO3?


a) 1

b) 0

2NH3(g)
Keq = 36
16. (10 pts) Consider the reaction: H2(g) + H2N-NH2(g)
Initially, [H2] = [H2N-NH2] = 0.2 M, and [NH3] = 2 M. What is the concentration of [NH3] at equilibrium?
Clearly show all your work and put a box around your final answer.
Ksp = 4 1012
a) Assuming you have excess Mg(OH)2(s) present, what will be the equilibrium concentration of [OH] in the
solution? b) What is the pH of the solution? Show all your work and put boxes around your final answers.

17. (10 pts) Consider the reaction:

Mg(OH)2(s)

Mg2+(aq) + 2OH(aq)

18. (10 pts) a) What is the concentration of Tl+(aq) from TlCl(s) in pure water? Ksp (TlCl) = 1 1014
b) What is [Tl+(aq)] from TlCl(s) in 1 M KCl(aq)? Clearly show all your work and box your final answers.
19. (10 pts) CO2(g) + H2O(l)
H2CO3(aq) Keq 1 103 Ea = only slightly larger than Grxn
H2CO3 (carbonic acid) is an unstable molecule that can not be isolated, but does exist in water when CO2 is
present. Use the Keq and Ea values to sketch out a simple qualitative one-step potential energy diagram (G vs.
rxn coordinate, numerical scale not needed, indicate reactants with R and products with P) and clearly explain
why carbonic acid can not be isolated based on the Keq and Ea values.

CHEM 1422 Exam # 2 (March 26, 2009)

Name (printed):_________________________

Equilibrium & Acids/Bases

Name (signed):_________________________

Put a big X this box if you want your graded exam put out in the public racks outside Prof. Stanleys
office after grading. If you dont check it, Prof. Stanley will keep your exam and you will have to stop by
to pick it up from him personally.

1. (5 pts) For which of the following reactions will increasing the overall pressure make more products? Circle
your answer and briefly discuss your reasoning in the space below.
a) 3NO2(g) + H2O(l)

2HNO3(aq) + NO(g)

b) 3H2S(g) + 2Fe(s)
c) MgO(s) + C(s)

Fe2S3(s) + 3H2(g)

Mg(s) + CO2(g)
d) 2HCl(g)
Cl2(g) + H2(g)
e) AgCl(s) + 2NH3(aq)
Ag(NH3)2+(aq) + Cl(aq)

Re2(CO)10(soln) + H2(g) Initially, [Re2(CO)10] =


2. (5 pts) Consider the reaction: 2HRe(CO)5(soln)
[H2] = 1 M. What is the concentration of HRe(CO)5 at equilibrium? Keq = 4 Circle your answer and show
your work in the space below.
a) 0 M

b) 0.2 M

c) 0.4 M

d) 0.8 M

e) 2 M

3. (5 pts) Which of the following salts will be the least soluble in H2O? Circle your answer and show your work or
reasoning below.
a) AgCl (Ksp = 1 1010)

b) AgF (Ksp = 1 106)

c) AgI (Ksp = 1 1016)

d) AgBr (Ksp = 1 1013)

e) Ag3PO4 (Ksp = 1 1020)

f) AgCN (Ksp = 1 1012)

CHEM 1422 - Exam # 2 (Equilibrium & Acids/Bases)

4. (5 pts) What is the G value (in kJ/mol) that corresponds to a Keq = 0.09 at 500 K. R = 8.314 J/molK
Circle your answer and show your work or reasoning below.
a) 1.0 kJ/mol

b) 10 kJ/mol

c) 10 kJ/mol

d) 100 kJ/mol

e) 10,000 kJ/mol

5. (5 pts) Which of the following salts will produce an acidic solution when dissolved in water? Briefly discuss your
reasoning in the space below.
a) KClO4

b) CsBr

c) Li2CO3

d) FeCl3

e) Ca(OH)2

6. (5 pts) I was at an agonomy talk and the speaker was discussing soil nutrients. The free Fe3+ concentration in soil
was expressed on his overhead as: p[Fe+3] = 17. Which of the following is the Fe+3 concentration represented by
p[Fe+3] = 17. Briefly discuss your answer below.
a) 1 1017 M

b) 1.7 1010 M

c) 1.7 107 M

d) 0.17 M

e) 1.7 M

7. (5 pts) What is the pH of a 1 M solution of the weak base nicotine? Show your work or discuss your reasoning
below.
a) 2

b) 4

c) 8

d) 14

e) Cant calculate from information given

8. (5 pts) What is the pH of a 1 M solution of a compound that has a pKb = 10 ? Briefly explain your answer.
a) 0

b)

c) 10

d) 14

e) Cant calculate from information given

CHEM 1422 - Exam # 2 (Equilibrium & Acids/Bases)

9. (5 pts) What is the pH of a 1 1010 M solution of CsOH? Briefly explain your answer.
a) 4

b)

c) 10

d) 14

e) 18

10. (5 pts) Calculate the pH of a 0.001 M solution of an acid with a pKa = 5.

Show your work.

11. (10 pts) Consider the following reaction: Fe2(CO)8(s) + 2CO(g)


2Fe(CO)5(soln) Keq = 25
There is initially excess Fe2(CO)8(s), [CO] = 1 M, and Fe(CO)5 = 11 M. What will be the concentration of
Fe(CO)5 at equilibrium? Clearly show all your work. Put a box around your final answer.

12. (10 pts) What is the concentration of Ag(aq) in a 1 M K2SO4(aq) solution to which excess Ag2SO4(s) has been
added. Ksp (Ag2SO4) = 4 1010 Clearly show all your work. Put a box around your final answer.

CHEM 1422 - Exam # 2 (Equilibrium & Acids/Bases)

13. (10 pts) What is the pH of a 0.1 M solution of a base whose conjugate acid has a pKa value of 9. Clearly show
all your work. Put a box around your final answer.

14. (10 pts) What is the pH of a 0.01 M solution of KOCl. pKa (HOCl) = 8. Clearly show all your work. Put a box
around your final answer.

15. (10 pts) Consider the following three bases: (CH3)3P

Which is the strongest and why? Clearly discuss your reasoning.

(CF3)2N

CHEM 1202 Exam # 3A (11/30/2006)


Acids/Bases

Name (printed) _____________________


Name (signed) _____________________

Put a big X this box if you want your graded exam put out in the public racks
outside Prof. Stanleys office after grading. If you dont check it, Prof. Stanley will
keep your exam in his office and you will have to stop by to pick it up from him
personally.
Answer Sheet -- Please Hand In! Please use CAPITAL letters for your multiple choice answers!
1. _____

2. _____

3. _____

4. _____

5. _____

6. _____

7. _____

8. _____

9. _____

10. _____

11. _____

12. _____

13. _____

14. _____

15. _____

16.

17.

Exam 3A Acids/Bases (2006)

18.

19.

BONUS QUESTION:

Exam 3A Acids/Bases (2006)

CHEM 1202 Exam # 3A

November 30, 2006

Please Answer All Questions on the Answer Sheet


1. (4 pts) What is the pH of 1 M RbOH?
a) 1

b) 5

c) 9

d) 14

e) 20

c) 7

d) 10

e) 14

c) 5

d) 7

e) 11

d) H2SO4

e) H2S

2. (4 pts) What is the pH of 1 1010 M HNO3?


a) 1

b) 5

3. (4 pts) What is the pH of 1 103 M HClO4?


a) 1

b) 3

4. (4 pts) Which of the following is NOT a strong acid?


a) HCl
b) HNO3
c) HI
5. (4 pts) Which of the following statements is FALSE?

a) Weak acids have moderately strong conjugate bases.


b) Fe3+(aq) is considered a Lewis Acid.
c) HF is a weak acid in large part due to the concentrated negative charge on the small F anion.
O
C

d) H3C
OH is a weak base.
e) Compounds with nitrogen atoms that have a lone pair of electrons are weak bases.
6. (4 pts) Which of the following is NOT a strong base?
a) Fe(OH)3

b) LiOH

c) CsOH

d) Ba(OH)2

e) RbOH

7. (4 pts) What is the G value (in KJ/mol) that corresponds to a pKa = 8 (typical for a strong acid).
R = 8.314 J/molK, T = 300 K
a) 34 KJ/mol

b) 16 KJ/mol

c) 0 KJ/mol

d) 2 KJ/mol

e) 46 KJ/mol

d) PF6

e) NO3

8. (4 pts) Which of the following anions is the most basic?


a) Cl

b)

c)

O
F

C
H3C

9. (4 pts) What is the pH of 10 M HPF6? pKa = 10


a) 10

b) 1

c) 7

d) 12

e) Cant calculate from information given

Exam 3A Acids/Bases (2006)

10. (4 pts) What is the pH of 0.01 M formic acid, HCOOH?


a) 1

b) 1

c) 6

d) 9

e) Cant calculate from information given

11. (4 pts) What is the pH of a 1 104 M solution of the weak base nicotine? Kb = 1 106
a) 0
b) 5
c) 7
d) 9
e) 10
12. (4 pts) Calculate the pH of a 0.1 M solution of carbonic acid, H2CO3. Ka1 = 1 107, Ka2 = 1 1011.
b) 2
c) 4
d) 5
e) 10
a) 1
13. (4 pts) I react 333 mL of 0.3 M NaOH with 666 mL of 0.15 M acetic acid. Which of the following statements is
TRUE? pKa (acetic acid) = 5
a) You will make a very acidic solution (pH ~ 0)
c) You will make a very basic solution (pH ~ 14)
e) You will make a neutral solution

b) You will make a moderately acidic solution (pH ~ 4)


d) You will make a moderately basic solution (pH ~ 9)

14. (4 pts) Which of the following salts will produce a basic solution when dissolved in water?
a) Ga(NO3)3

b) Li2SO4

c) BaO

d) NH4NO3

e) SrBr2

15. (4 pts) Which of the following salts will produce an acidic solution when dissolved in water?
a) Ba(ClO4)2

b) CsI

c) Rb2CO3

d) AlCl3

e) NaBr

16. (10 pts) What is the pH of a 0.1 M solution of citric acid? pKa = 5. Clearly show all your work. Put a box
around your final answer.
17. (10 pts) What is the pH of a 1 M solution of the base methyl amine? pKa (methyl amineH+) = 10. Clearly show
all your work. Put a box around your final answer.
18. (10 pts) Why is triflic acid (F3CSO3H) an even stronger acid than sulfuric acid (H2SO4)? Clearly discuss. There
are two or three important factors.
19. (10 pts) What is the pH of a 1 M solution of sodium carbonate (Na2CO3). pKb (CO32) = 4
Clearly show all your work. Put a box around your final answer.
Bonus Question (10 points): Consider an acetic acid/sodium acetate buffer system. pKa (acetic acid) = 5.
A) Assuming a 1:1 mixture of acetic acid and sodium acetate, what is the pH of this buffer system? Briefly
explain/discuss. If you dont know how to give a precise # for the pH, at least state whether this buffer will be
acidic, basic, or neutral (with reasoning).
B) Clearly explain how this buffer system keeps the same approximate pH when one adds small amounts of
strong acid or base.

CHEM 1422 Exam # 3 (April 23, 2009)

Name (printed):_________________________

Acids/Bases & Redox/Electrochemistry

Name (signed):_________________________

Put a big X this box if you want your graded exam put out in the public racks outside Prof. Stanleys
office after grading. If you dont check it, Prof. Stanley will keep your exam and you will have to stop by
to pick it up from him personally.

1. (5 pts) Consider the following anions. Using fundamental concepts we have discussed in class, which one would
be the weakest conjugate base and give rise to the strongest acid. Circle your choice and provide clear reasoning
for your answer.
a) AsO43

b) F

c) PH2

e) PF6

d) CO32

2. (5 pts) [N(CH3)2] is a stronger base than OH. If 0.1 moles of [N(CH3)2] is added to 1 L of water, what will the
resulting pH be? Circle the answer (yes, the correct answer is there) and clearly explain your reasoning below.
a) 0

b) 0.1

c) 7

d) 13

e) 25

3. (5 pts) Identify the following 1:1 solution mixtures as acidic, basic, or neutral.
a) NH3 + NH4NO3 (pKa = 9.3)
b) NaBr + KClO4
c) H3PO4 + NaH2PO4 (pKa = 2.1)
d) C5H4N (pyridine) + [C5H4NH]Br (pKa = 5.2)
e) H2S + CsHS (pKb = 7.0)
4. (5 pts) Calculate the pH of a 0.1 M solution of carbonic acid, H2CO3. pKa1 = 7, pKa2 = 11.
Circle your choice and clearly show your work (or reasoning) below.
a) 1

b) 2

c) 4

d) 5

e) 10

CHEM 1422 - Exam # 3 (Acids/Bases & Redox/Electrochemistry)

5. (5 pts) Which of the following salts will produce an acidic solution when dissolved in water? Circle your answer
and briefly discuss your reasoning in the space below.
a) KClO4

b) Cs2S

c) Li2CO3

d) CrCl3

e) Mg(OH)2

6. (5 pts) What is the pH of a 1.0 M solution of sodium carbonate (Na2CO3). pKb (CO32) = 4
Clearly show all your work. Put a box around your final answer.

7. (5 pts) Which of the following substances is the best reducing agent? Circle your answer and briefly explain your
reasoning below.
a) Na+

b) Pb

c) Li+

d) F

e) Mg

8. (5 pts) What is the potential of a battery composed of the following: Al/Al3+(aq, 1 M) and Ag/Ag+(aq, 1 M)?
Temperature = 25C. Clearly show all your work and the overall balanced reaction.

9. (5 pts) Given the data presented on the Standard Reduction Potential Table provided, what is the highest voltage
single cell battery that you could make? Write the overall balanced rxn and potential. Briefly discuss why this
ultimate battery isnt practical.

CHEM 1422 - Exam # 3 (Acids/Bases & Redox/Electrochemistry)

10. (5 pts) Qualitatively how would the hydrogen electrode potential change if the [H+] concentration changed from
1.0 M? Why?

11. (10 pts) 100 mL of 0.5 M acetic acid (HOAc) reacts with 400 mL of 0.125 M NaOH. What is the pH of the
resulting solution? pKa (HOAc) = 5 Clearly show all your work. Put a box around your final answer.

12. (10 pts) Consider the titration curve shown to the right.

12
10

a) (2 pts) What type of material (acid or base) is being titrated? Brief reasoning?
pH

8
6
4
2

b) (2 pts) Is the material from part a) strong or weak? Brief reasoning?

0
0

10

20

30

40

mL of HCl added

c) (2 pts) Why is the region between 10 & 20 mL not changing pH?

d) (4 pts) What is the approximate pKa/b of the material in question? Brief reasoning?

50

CHEM 1422 - Exam # 3 (Acids/Bases & Redox/Electrochemistry)

13. (10 pts) Balance the following reaction in basic solution (add, CN, water or OH as needed).
Clearly show all your work.
Au(s) + CN(aq) + O2(g)

[Au(CN)4] (aq) + OH (aq)

14. (10 pts) How many moles of Al can be deposited per hour from a molten mass of Al2O3/Na3AlF6 if a current of
30,000 amps is used?

15. (10 pts) Consider the following electrochemical cell at 25C: H2(g)/2H+(aq) and Ag/Ag+(aq, 1.0 M).
A potential of 0.741 V is measured. What is the pH of the cell? Clearly show all your work.

CHEM 1422

Honors: General
Chemistry
Section 01

Previous 2 Years of
Homeworks & Exams
with Answers
Prof. George G. Stanley
Department of Chemistry

CHEM 1202 - Homework # 1


Background
Due January 22, 2009 (2 PM)

ANSWER KEY

Check the box to the right if you want your graded homework to be placed out in the public rack outside
Prof. Stanleys office. Otherwise you will have to pick up your homework from Prof. Stanley in person:

1. (5 pts) Why do or don't you like chemistry as it has been taught to you so far in high school and college?
What did or didnt you like? Don't try to suck up to me by writing only good things (unless they are true).
I'm looking for critical comments and reasoning. Tell me about things you have liked (if any) and disliked.
Short mindless answers will get no credit. This question is an individual, not group, effort. Please type up
your answer neatly, include your name, and E-mail it to me separately (gstanley@lsu.edu) as a Word
or PDF file (or body of the E-mail message).

2. (5 pts) Define electronegativity in your own words (dont just copy from a book). Give a couple of
elemental examples to illustrate your definition (i.e., elements with high or low values and how their
electronegativity affects their ability to gain or lose electrons).
Electronegativity is the property of an atom to want to hang on to its valence electrons and to possess a filled
octet of electrons. For example, fluorine is the most electronegative element on the Periodic Table and has 7
valence electrons. It is almost impossible for fluorine to lose an electron to form F+. But it has a very strong
tendency to add an electron to form the 8 valence electron F. The high electronegativity means that F is
very stable and unlikely to lose an electron. On the other hand, Cs is one of the least electronegative elements
and has a very high tendency to lose its one valence electron to form Cs+. Elements with low electronegativity
(high tendency to lose electrons to form cationic atoms) are called electropositive elements. Electronegative
elements tend to want to gain electrons and possess negative charges (or partial negative charges in neutral
compounds), while electropositive elements tend to want to lose electrons to form cations.

3. (5 pts) Look up, list, and explain the trend in melting points in terms of intermolecular attractive forces for
the following compounds: CsI, LiF, BaS, MgO, and Al2O3. Make use of Coulombs Law (in a qualitative
sense) in your explanation.
These can all be considered ionic compounds. Their melting points generally
increase as the magnitude of the positive and negative charges on the ions
increase. Coulombs law is:

force

q q
d2

M.P.
CsI

626C

LiF

845C

BaS
1200C
The force of attraction between charged ions is proportional to the magnitude
of the charges times one another divided by the distance between the charges
MgO
2852C
squared. So the higher the charges and the smaller the ions (the closer they
can get to one another) the stronger the force of attaction. This should relate
Al2O3
2015C
directly to physical properties such as melting points, boiling points, hardness,
density, etc. CsI has the lowest melting point since the ions only have
1charges (relative attractive force of 1 x 1 = 1) and Cs+ and I are rather large ions that have a larger
separation distance between them. LiF, on the other hand, has the smallest 1charged ions and the closer
approach leads to a stronger attractive force and higher melting point. The same logic holds for BaS and MgO,
both of which are composed of 2 charged ions (2 x 2 = 4 times the relative attractive force). But Ba2+ and S2
are both larger ions relative to Mg2+ and O2, thus MgO has the higher melting point. Al2O3 is an exception to
these guidelines. It is composed of Al3+ and O2 ions that should have a stronger attraction compared to two
2 charged ions. But there is a covalent component in the Al-O bonding that decreases the simple ionic
charges leading to a reduction in the melting point.

CHEM 1422 - HW#1 Background (2009)

4. (5 pts) Consider the reaction: 2C(s) + 2Br2

Br2C=CBr2

If 120 g of carbon reacts with 160 g of Br2 and the reaction goes in 50% yield, how many grams of
Br2C=CBr2 (C2Br4) are produced? Clearly show and explain all your work.
This is a limiting reagent problem and one always needs to first calculate the # of moles of the reagents and
figure out from the stoichiometry to determine the limiting reagent.
# moles C = (120 g C)/(12 g/mole) = 10 moles C
# moles Br2 = (160 g Br2)/(160 g/mole) = 1 mole Br2
So Br2 is the limiting reagent (carbon is present in large excess). The stoichiometry of the reaction is that 2
moles of Br2 will react with 2 moles of carbon to make 1 mole of product (Br2C=CBr2). Because we only have
1 mole of Br2 present only 0.5 moles of product will be made (half as much as the limiting reagent Br2). Thus
we need to calculate the molecular weight of the product and multiply this times the 0.5 moles of product that
can be produced from 1 mole of Br2 reacting with 1 mole of carbon. MW (Br2C=CBr2) = 344 g/mol.
Grams product (Br2C=CBr2) = (344 g/mol)(0.5 moles) = 172 g
But that assumes a 100% yield. 50% yield means that I need to multiply the 100% yield amount by 0.5 (50%):
50% yield = (172 g product, 100% yield)(0.5) = 86 g product produced.

5. (10 pts) Sketch out the Lewis dot structures for the following molecules (use lines for bonds and pairs of
dots for lone pairs). Indicate the formal charges, if any, on the appropriate atoms.

a) H2SO4

b) C2H2

O
O

d) NO3

c) AlCl3

Cl

Al

Cl

Cl

Al and B are the two common elements


that often only have a 6 e- valence
count (one empty orbital) for their
compounds. These are Lewis Acids.

e) P(CH3)3

H
H C

H C H
P

H
H C H
H

N
O

When an oxygen atom only


has one bond and an octet
of e- it has a formal
negative charge. When N
has 4 bonds it is assigned
a formal positive charge.

CHEM 1202 - Homework # 2


Thermodynamics
Due Sept 19th, 2006

ANSWER KEY

Check the box to the right if you want your graded homework to be placed out in the public rack outside
Prof. Stanleys office. Otherwise you will have to pick up your homework from Prof. Stanley in person:

1. (12 pts) From the thermodynamic data given at the end of your lecture notes calculate Hrxn, Srxn and
Grxn for the following reactions. For Grxn please use the Grxn = Hrxn TSrxn formula with T =
298K. Indicate whether the reactions are spontaneous or non-spontaneous. Show your work.
a) 12NH3(g) + 21O2(g)

8HNO3(g) + 4NO(g) + 14H2O(g)

Hrxn = Hprod Hreact = [(8)(135.1 kJ/mol) + (4)(90.2 kJ/mol) + 14(241.8 kJ/mol)] [(12)(46.1 kJ/mol) + (21)(0 kJ/mol)]
Hrxn = (4105.2 kJ/mol) (553.2 kJ/mol) = 3552 kJ/mol
Srxn = Sprod Sreact = [(8)(266 J/molK) + (4)(211 J/molK) + 14(189 J/molK)] [(12)(192 J/molK) + (21)(205 J/molK)]
Srxn = 5618 J/molK 6609 J/molK = 991 J/molK
Grxn = Hrxn TSrxn = 3552 kJ/mol (298K)(0.991 kJ/molK)
Grxn = 3552 kJ/mol + 295 kJ/mol = 3257 kJ/mol

b) 2O3(g)

spontaneous

3O2(g)

Hrxn = Hprod Hreact = [(3)(0kJ/mol)] [(2)(143kJ/mol)]


Hrxn = (0kJ/mol) (286kJ/mol) = 286 kJ/mol
Srxn = Sprod Sreact = [(3)(205 J/molK)] [(2)(239 J/molK)]
Srxn = 615 J/molK 478 J/molK = 137 J/molK
Grxn = Hrxn TSrxn = 286 kJ/mol (298K)(0.137 kJ/molK)
Grxn = 286 kJ/mol 41 kJ/mol = 327 kJ/mol

spontaneous

C6H12O6(s) (glucose) + 6O2(g)


} this is photosynthesis!
c) 6CO2(g) + 6H2O(l)
Hf (C6H12O6) = 1274 KJ/mol Gf (C6H12O6) = 910 KJ/mol
S (C6H12O6) = 212 J/Kmol
Hrxn = Hprod Hreact = [(1274 kJ/mol) + (3)(0 kJ/mol)] [(6)( 285.8 kJ/mol) + (6)( 393.5 kJ/mol)]
Hrxn = (1274 kJ/mol) (4075.8 kJ/mol) = 2803 kJ/mol
Srxn = Sprod Sreact = [(212 J/molK) + (6)(205 J/molK)] [(6)(69.9 J/molK) + (6)(213.6 J/molK)]
Srxn = 1442.1 J/molK 1701 J/molK = 258.9 J/molK
Grxn = Hrxn TSrxn = 2802.8 kJ/mol (298K)(0.259 kJ/molK)
Grxn = 2803 kJ/mol + 77 kJ/mol = 2880 kJ/mol

non-spontaneous

2. (8 pts) For the following processes, is the entropy of reaction (Srxn) increasing, decreasing or staying
about the same? Use the qualitative rules about entropy discussed in lecture to determine the answer.
a) Ca(s) + H2SO4(aq)
b) 4Fe(s) + 3O2(g)
c) N2(g) + 3H2(g)

INCREASING

CaSO4(s) + H2(g)

DECREASING

2Fe2O3(s) (rust)
2NH3(g)

DECREASING

d) setting off a fire cracker (illegal in East Baton Rouge Parish)


e) AgCl(s) + Br(aq)
f) Mg(s) + CO(g)
g) Cu+2(aq) + 2OH(aq)
h) Ag+(aq) + Cu(s)

AgBr(s) + Cl(aq)

INCREASING

STAYING ABOUT THE SAME

MgO(s) + C(s)

DECREASING

Cu(OH)2(s)

DECREASING

Ag(s) + Cu+(aq)

STAYING ABOUT THE SAME

3. (5 pts) A typical instant hot pack uses 55 g of CaCl2(s) and 180 g of H2O, which react together to make
hydrated Ca2+(aq). Use the following formula to calculate the temperature (C) of the heat pack after it has
been activated (mixed together). Assume that the initial temperature of the water is 25C (room temperature),
Cp (water) = 75 J/mol K (watch your units!). Hrxn = 60 kJ/mol

T =

nsalt ( Hrxn)
nwater (Cpwater )

T = Tfinal - Tinital
nwater = # of moles of water used
nsalt = # of moles of salt dissolved Cp = heat capacity of solvent
Hrxn = enthalpy of dissolving salt in solution (also called Hsol)

# of moles of H2O = (180 g)/(18 g/mol) = 10 moles H2O


# of moles of CaCl2 = (55 g)/(110g/mol) = 0.5 moles CaCl2
Hrxn is given you dont have to calculate!!!
(0.5mol )( 60kJ / mol )
30kJ
T =
=
= 40 K = 40 D C
(1K = 1C, same magnitude, different scale)
(10mol )(0.075kJ / molK ) 0.75kJ / K
Final Temp = Initial Temp + T = 25C + 40C = 65C
4. (5 pts) (a) Using the Gibbs Free Energy formula to plot the value of Grxn from 500 K to 1200 K for the
reaction: 2HgO(s)
2Hg(l) + O2(g)
(use table at the end of the chapter -- you can assume that Hf and S do not vary much with temperature)
The vertical axis is energy in
kJ/mol -- you have to fill in the
range of units for this axis based
Hrxn = 181.6 kJ/mol
on your calculations. (HINT: you
Srxn = 216.4 J/mol K
only have to calculate 3 to 5 data
points to make the plot, but you
can also do this in Excel and let it
calculate Grxn at many temps).
(b) What is the temperature range
for which this reaction is
spontaneous? Indicate on plot.
(c) Mark and indicate the
temperature at which Grxn = 0?

CHEM 1422 - Homework # 2


ANSWER KEY
Thermodynamics
Due January 29, 2009 Check the box to the right if you want your graded homework to be placed out in the public rack outside
Prof. Stanleys office. Otherwise you will have to pick up your homework from Prof. Stanley in person:

1. (12 pts) From the thermodynamic data given at the end of your lecture notes calculate Hrxn, Srxn and
Grxn for the following reactions. For Grxn please use the Grxn = Hrxn TSrxn formula with T =
298K. Indicate whether the reactions are spontaneous or non-spontaneous. Show your work.
a)

Cl2(g) + H2(g)

2HCl(g)

Hrxn =Hprod Hreact = [(2)(92.3 kJ/mol)] [(0 kJ/mol) + (0 kJ/mol)]


Hrxn = (184.6 kJ/mol) (0 kJ/mol) = 184.6 kJ/mol
Srxn = Sprod Sreact = [(2)(186.8 J/molK)] [223.0 J/molK + 130.6 J/molK]
Srxn = 373.6 J/molK353.6 J/molK = 20 J/molK
Grxn = Hrxn TSrxn = 184.6 kJ/mol (298K)(0.02 kJ/molK)
Grxn = 184.6 kJ/mol 6.0 kJ/mol = 190.6 kJ/mol
b)

Fe(s) + 5CO(g)

spontaneous

Fe(CO)5(l)

Hrxn =Hprod Hreact = [(774 kJ/mol)] [(0 kJ/mol) + (5)(110.5 kJ/mol)]


Hrxn = (774 kJ/mol) (552.5 kJ/mol) = 221.5 kJ/mol
Srxn = Sprod Sreact = [(338 J/molK)] [27.3 J/molK + (5)(197.6 J/molK)]
Srxn = 338 J/molK1015 J/molK = 677 J/molK
Grxn = Hrxn TSrxn = 221.5 kJ/mol (298K)(0.677 kJ/molK)
Grxn = 221.5 kJ/mol 202 kJ/mol = 19.5 kJ/mol
c)

SiO2(s) + 6HF(g)

H2SiF6(aq) + 2H2O(l)

spontaneous

[glass is mainly SiO2]

Hrxn =Hprod Hreact = [(2331 kJ/mol) + (2)(286 kJ/mol)] [(911 kJ/mol) + (6)(271 kJ/mol)]
Hrxn = (2903 kJ/mol) (2537 kJ/mol) = 366 kJ/mol
Srxn = Sprod Sreact = [(190 J/molK) + (2)(70 J/molK)] [42 J/molK + (6)(174 J/molK)]
Srxn = 330 J/molK1086 J/molK = 756 J/molK
Grxn = Hrxn TSrxn = 366 kJ/mol (298K)(0.756 kJ/molK)
Grxn = 366 kJ/mol 225 kJ/mol = 141 kJ/mol
d)

CaCO3(s)

spontaneous

CaO(s) + CO2(g)

Hrxn =Hprod Hreact = [(635 kJ/mol) + (393 kJ/mol)] [(1207 kJ/mol)]


Hrxn = (1028 kJ/mol) (1207 kJ/mol) = 179 kJ/mol
Srxn = Sprod Sreact = [(40 J/molK) + (214 J/molK)] [93 J/molK]
Srxn = 254 J/molK93 J/molK = 161 J/molK
Grxn = Hrxn TSrxn = 179 kJ/mol (298K)(0.161 kJ/molK)
Grxn = 179 kJ/mol 48 kJ/mol = 131 kJ/mol

non-spontaneous

I provided the S
value for H2SiF6 in
class (190 J/molK)

CHEM 1422 HW#2 Thermodynamics (2009)

2. (8 pts) Is the entropy increasing, decreasing or staying about the same? Use the qualitative entropy rules
discussed in lecture to determine the answer. Write the answer to the right of each process.
a) Fe2O3(s) + Al(s)

Fe(s) + Al2O3(s)

b) raw egg

hard boiled egg

decreasing (liquid egg going to solid egg)

[Ca(H2O)6]Cl2(s) decreasing (7 particles combining into 1 particle)

c) CaCl2(s) + 6H2O(l)
d) C6H12(l) + 9O2(g)

6CO2(g) + 6H2O(g) increasing (9 gas & 1 liquid making 12 gases)

e) H2CO(aq) + H2O(l)
f) mowing the lawn

about the same

2H2(g) + CO2(g) increasing (2 liquids going to 3 gases)

increasing (cutting up longer grass blades into many short pieces)

g) AgCl(s) + I(aq)
h) Sr(s) + 2H2O(l)

AgI(s) + Cl(aq)

about the same

Sr2+(aq) + 2OH(aq) + H2(g) increasing

3. (2 pts) Circle the compound that has the highest entropy. Give a brief reason explaining your answer.
a) Hg(l)

b) H2O(l)

c) Pb(s)

d) C2H5OH(l)

e) CCl4(l)

If you look up the entropies, S[CCl4(l)] = 216 J/molK, while S[C2H5OH(l)] = 161 J/molK. Although C2H5OH
(ethanol) has more atoms and has a somewhat more complex structure, the heavier Cl atoms in CCl4 end
up contributing more to the entropy. It was not easy, therefore, to qualitatively order these two choices so Ill
give full credit for either one, but you also have to have some qualitative reasoning and not just that you
looked up the entropies and CCl4 had the largest value.

4. (4 pts) Why does Al2O3(s) have a lower entropy than Fe2O3(s)? There are two primary qualitative reasons
for this. You may have to use the chemistry library to get more information (i.e., properties) on these two
common compounds to answer the question.
Al2O3(s), alumina, has a lower entropy (S = 51 J/molK) vs. Fe 2O3(s), rust, (S = 87 J/molK) because it is has stronger
bonding. This is indicated by the higher melting point of alumina (2015C) vs. rust (1565C). Alumina is one of the
harder common materials and the main component in rubies and sandpaper, while rust is a relatively soft, flaky solid.
Hardness and melting point of a material is usually related to the strength of the bonds that connect the solid together.
The Al-O-Al bonding network is very strong relative to iron. We discussed this in class with the difference in entropy
between graphite and diamond. The second main reason is that Al2O3 has a lower MW, but the same relative
complexity as Fe2O3, which usually indicates a lower entropy. Each of these factors counts for about 50% of the
entropy difference.

5. (4 pts) a) Small amounts of Fe(CO)5(l) usually form in steel tanks containing pressurized CO(g). You
worked out the thermodynamics of this in question 1b. At what temperature (C) will the formation of
Fe(CO)5 become non-spontaneous? Show your work. b) A similar reaction occurs to make Ni(CO)4(l) with
Grxn = 38 kJ/mol, Hrxn = 230 kJ/mol, and Srxn = 480 J/Kmol. High pressure reactors use a thin
disk of metal as a safety mechanism that will rupture and release gasses if the pressure in the reactor gets too
high. If one was using CO gas, which disk (Fe or Ni) would be more likely to prematurely fail due to the
metal being dissolved away by CO? Briefly explain why.
a) We can find this temperature by using the Grxn = Hrxn - TSrxn formula, setting Grxn = 0 (the
point at which the reaction shifts from negative G to positive G), and solving for the temperature:

H rxn
221 kJ /mol

327 K

S rxn
0.677 kJ /molK

or

54C

b) Grxn for the nickel reaction is more spontaneous (38 kJ/mol for Ni vs. 19 kJ/mol for Fe) meaning
that it will be more likely to occur and release more free energy in doing so. That means from a
thermodynamic viewpoint that it is more favorable for the Ni disk to be dissolved away by CO gas.
Although that does NOT mean that the Ni reaction will be any faster (reaction rates are governed by
kinetics and not thermodynamics), this does turn out to be the case and this actually happened
repeatedly for one of our high pressure reactors a number of years ago.

CHEM 1202 - Homework # 3


Chemical Kinetics
Due Tuesday, Sept 26, 2006 by 1 PM

ANSWER KEY

1. (5 pts) Which of the following energy diagrams best represents a reaction which will be the fastest and most
spontaneous. Circle your choice and include your reasoning below (brief statement).
a)

b)

c)

d)
R

G
R

Rxn Coordinate

R
P

Rxn Coordinate

Rxn Coordinate

Rxn Coordinate

d) is a spontaneous reaction (downhill from reactants to products) and has the smallest activation energy.

2. (5 pts) Consider the energy diagram to the right:


Circle the following diagram below that best represents the effect of adding a catalyst to
the reaction. Include your reasoning below (brief statement).
a)

b)

c)

d)

R
P

G
R

Rxn Coordinate

Rxn Coordinate

Rxn Coordinate

Rxn Coordinate

Rxn Coordinate

c) has the same reactant/product energies (G), while all the others have changed. G is a thermodynamic parameter
and catalysts never change the thermodynamics of a reaction. They just lower the activation energy for that reaction.

3. (5 pts) Consider the following reaction that is quite important for the manufacturing of many chemicals:

3H2

cyclohexane
benzene
G = +200 kJ/mol Activation Energy = +400 kJ/mol

Circle the energy curve shown below (R = reactants, P = products) that best represents the reaction described
above? Include your reasoning below (brief statement).
a)

b)

G
P

Rxn Coordinate

Rxn Coordinate

c)

d)

Rxn Coordinate

Ea

Rxn Coordinate

d) is the correct answer because it is non-spontaneous (product energy higher than reactant) and the Ea is
about twice as big as G.

HW #3 Kinetics (2006)

4. (5 pts) Consider the following reaction and kinetic data. Circle the correct kinetic rate expression for this
reaction. Clearly show and briefly discuss your reasoning.
A + B
Exp #

[A]

[B]

Initial Rate (Msec1)

1.5 M

1.5 M

0.2

1.5 M

3.0 M

0.8

3.0 M

3.0 M

0.8

6.0 M

3.0 M

0.8

a) rate = k[A][B]

b) rate = k[A]

C
In Exp # 1 & 2 we double the concentration of B (while
holding A at the same concentration), which increases the
rate by a factor of 4. Thus we have square power
relationship: rate [B]2.
In Exp 2 & 3 (or 3 & 4), doubling the concentration of A
has NO EFFECT on the rate. Therefore, A is NOT part of
the kinetic rate expression (it has a order of zero!).

c) rate = k[B]

d) rate = k[B]2

e) not enough data

5. (4 pts) Consider the following reaction and kinetic data. What is the rate constant for the reaction? Clearly
show all your work and reasoning.
First you have to determine the rate law from the exp data!!
H2NCH2CH2CH2CH2NH2 + 2HCl
Exp #

[(N)2]

[HCl]

Initial Rate (Msec1)

0.1

0.1

0.2

0.2

0.1

0.4

0.4

0.2

3.2

0.1

0.2

0.8

[H3NCH2CH2CH2CH2NH3]2+(Cl)2
Order on [(N)2]x is determined from
experiments 2 & 1:
(0.2/0.1)x = (0.4/0.2)
(2)x = 2
x=1
Order on [HCl]y is determined from experiments
4 & 1: (0.2/0.1)y = (0.8/0.2)
(2)y = 4
y=2

RATE LAW: rate = k[(N)2] [HCl]2


To calculate the rate constant, plug in one of the experimental sets of data (#1, for example) and solve for k : k
= rate / [(N)2] [HCl]2 = 0.2 Msec-1 / (0.1 M)(0.1 M)2
k = 0.2 M sec-1 / (0.001 M3) = k = 200 M-2sec-1

6. (6 pts) The reaction in question 3 is non-spontaneous at room temperature.

3H2

cyclohexane
benzene
G = +200 kJ/mol Activation Energy = +400 kJ/mol

Discuss the one thing that you can do that will increase the rate of reaction and make the reaction more
spontaneous. Clearly discuss how this change will influence the rate of reaction and why it will affect the
thermodynamics (spontaneity) of the reaction.
Increase the temperature! Raising the temperature increases the rate of all chemical reactions this is a classic
kinetic effect. It will also make this particular reaction more spontaneous, that is, make G less positive by
magnifying the effect of the entropy term in the G = H TS formula. At room temperature, this reaction has a
positive G, making it non-spontaneous. But the generation of 4 particles from one will definitely increase the
entropy of the reaction, giving us a fairly large positive S entropy term. Srxn is multiplied by the temperature
magnifying its effect as the temperature is increased. So eventually, as we raise the temperature the entropy
term will dominate making G go negative, thus giving us a spontaneous reaction.

CHEM 1422 - Homework # 3


Chemical Kinetics
Due Feb 12, 2009 (2 PM)

ANSWER KEY

Check the box to the right if you want your graded homework to be placed out in the public rack outside
Prof. Stanleys office. Otherwise you will have to pick up your homework from Prof. Stanley in person:

1. (3 pts) Which of the following energy diagrams best represents the slowest spontaneous reaction? Circle
your choice. Give a brief, but clear, explanation for your answer below the diagrams.
a)

b)

c)

d)
R

G
R
Rxn Coordinate

P
R
Rxn Coordinate

R
P
Rxn Coordinate

P
Rxn Coordinate

A spontaneous rxn is one that is thermodynamically downhill (products are lower in energy than reactants, negative G
value). Only c) and d) are spontaneous rxns. The slowest spontaneous rxn will have the largest activation energy, which
narrows the choice to c).

2. (5 pts) a) Describe in your own words and terms where the origin of the activation barrier comes from and
what it represents in a chemical reaction. b) Given the same thermodynamic factors, consider the reaction of
two small molecules or two large molecules with one another. Which pair should have the higher activation
energy? Why?
The activation energy represents the probability that two molecules (for a bimolecular rxn) will react when
they have a collision. The more reactive the two molecules the more likely they will react when they collide
(smaller activation energy, higher probability of reaction). Since most rxns involve the breaking and making
of chemical bonds between two parts of two different molecules, the smaller the molecule the fewer nonreactive bonds present and the higher the probability that when the two molecules collide the reactive
portions of the molecules will come into contact and actually react. For two large molecules, there are many
more bonds present that will not react, thus reducing the odds that the two reactive portions of the molecule
will come together in the right way to react. Thus two larger molecules will have a lower probability of
getting the right portions together to react and thus will have a higher activation energy relative to two small
molecules. In biological systems that are composed of large complicated molecules, Mother Nature
counters this effect to some extent by designing channels that guide small molecules to the reactive portion
of the large enzyme. The use of opposite charges on two large proteins can also help guide the reactive
portions of these molecules together, thus increasing the odds of proper rxn occurring.

HW#3 Kinetics (2009)

3. (3 pts) Consider the following reaction and information:


O
HO

C
H2

CH3

H2C

G = +100 kJ/mol

CH2 + CO + H2O

Activation Energy = +400 kJ/mol

Circle the energy curve shown below (R = reactants, P = products) that best represents the reaction described
above? Give a brief, but clear, explanation for your answer below the diagrams.
a)

b)

c)

d)

R
G R

+400 kJ
R to Ea

P
Rxn Coordinate

P
Rxn Coordinate

Rxn Coordinate

P
+100 kJ
uphill

R
Rxn Coordinate

G = +100 kJ/mol indicates an endoergic or uphill rxn. Only (d) has the product energies higher than the
reactants. The activation energy barrier of 400 kJ should be about 4 times larger than G as measured from
the reactant energy to the top of the activation energy barrier, which is the case for (d).

4. (5 pts) Consider the following reaction and kinetic data. Circle the correct kinetic rate expression for this
reaction. Show all your work and/or discuss your reasoning.
2A + B
a) rate = k[A][B]

b) rate = k[A]2

C + D

c) rate = k[B]

Exp #

[A]

[B]

Initial Rate (Msec1)

0.2 M

0.1 M

0.02

0.4 M

0.1 M

0.04

0.2 M

0.3 M

0.18

0.4 M

0.3 M

0.36

d) rate = k[B]2

e) rate = k[A][B]2

For experiments #1 & #2, the concentration of [A]


doubles, while [B] stays the same. Doubling the
conc. of [A], then, causes the initial rate to double.
Thus there is an exponent of 1 on [A].

For experiments #1 and #3, the conc. of [B] is tripling (factor of 3), while [A] stays the same. When the conc. of
[B] triples, the initial rate increases by a factor of 9. Thus, the exponent on [B] is 2 (i.e., [3]x = 9, therefore x = 2).
So the kinetic rate expression is: rate = k[A] [B]2 Overall, this is a third order rxn.

HW#3 Kinetics (2009)

5. (5 pts) Consider the following reaction and kinetic data. Circle the correct rate constant for this reaction.
Clearly show all your work including the rate law that you determine.
A + 2B
a) 2.2 x 106 M1sec1

b) 22 M1sec1

C+D

c) 220 M1sec1

Exp #

[A]

[B]

Initial Rate (Msec1)

0.2 M

0.1 M

0.002

0.2 M

0.2 M

0.002

0.4 M

0.2 M

0.008

0.8 M

0.4 M

0.032

d) 0.05 M1sec1

e) not enough data

For experiments #1 & #2, the concentration of [A]


stays the same, while [B] doubles. But doubling the
conc. of [B] does not cause any change in the initial
rate of the reaction. Thus there is an exponent of 0
on [B], which means that it is not in the kinetic rate
expression.

For experiments #2 and #3, the conc. of [A] doubles (factor of 2), while [B] stays the same. When the conc. of [A]
doubles, the initial rate increases by a factor of 4. Thus, the exponent on [A] is 2 (i.e., [2]x = 4, therefore x = 2).
So the kinetic rate expression is: rate = k[A]2 Overall, this is a second order rxn.
Now that we have figured out the kinetic rate expression, we can calculate k the rate constant. You can take any
of the four experimental runs and substitute in the various values into the kinetic rate expression and solve for
the rate constant. Ill use experiment # 1:
rate = k[A]2 or

0.002 Msec1 = k [0.2 M]2

or k = (0.002 Msec1)/(0.04 M2) = 0.05 M1sec1

6. (4 pts) Catalysts can be used on non-spontaneous reactions to lower the activation barrier. If a catalyst
lowers the activation barrier too much, however, a serious problem can arise. Consider the diagrams shown
below. What is the problem for the catalyzed rxn with the lower activation energy? Why can a substantial
activation barrier actually help an uphill chemical reaction if one
wants to make as much product as possible?
If you lower the activation energy too much for a non-spontaneous rxn
the products will react backwards quickly to make reactants. The
presence of an activation barrier can help to slow the back-reaction to
the more stable reactants. This is NOT a problem for a spontaneous rxn
where the products are significantly more stable than the reactants and
will not want to back-react.

7. (5 pts) A reaction has a initial rate of reaction of 0.001 Msec-1 at 70C. This increases to 0.100 Msec-1 at
90C. Calculate the activation energy for this reaction?
First you need to convert the temperatures into kelvin: 70 + 273 = 343 K; 90C + 273 = 363 K
The Arrhenius equation uses rate constants, what are given in this problem are initial reaction rate. BUT, reaction
rates are directly proportional to rate constants, so the ratio of either will be the same!

k2
0.1
8.314 J / molK ln

k1
38.29
(8.314) ln(100) (8.314)(4.605)
0.001
Ea =
=
=
=
=
4
4
1
1
1.61 10
1.61 10
1.61 10 4
1
1
T T
343 363
1

2
R ln

Ea = 2.38 105 J / mol = 238 kJ / mol

CHEM 1202 - Homework # 4


Chemical Equilibrium # 1
Due Thursday, Oct 19, 2006

ANSWER KEY

Check the box to the right if you want your graded homework to be placed out in the public rack outside
Prof. Stanleys office. Otherwise you will have to pick up your homework from Prof. Stanley in person:

2NO(g)
Keq = 4
1. (3 pts) Given the following information: N2(g) + O2(g)
Calculate the equilibrium constants for the equations shown below (you dont have to show your work):
a) 2NO(g)

N2(g) + O2(g)

b) N2(g) + O2(g)

Keq = 0.25 (switching the rxn around, 1/Keq)


Keq = 2 (multiplying by , raise Keq to power)

NO(g)

c) 2N2(g) + 2O2(g)

4NO(g)

Keq = 16 (multiplying by 2, raise Keq to 2 power)

2. (2 pts) Circle the equilibrium expression listed below that is the correct one for the following reaction:
SCl2(g) + H2O(g)
a)

[SCl2] [H2O]

Keq = [HCl] [SO]

b)

2HCl(g) + SO (g)

[SCl2] [H2O]

[HCl] [SO]

c)

Keq = [HCl]2 [SO]

Keq = [SCl ] [H O]
2
2

d)

[HCl]2 [SO]
Keq = [SCl ] [H O]
2
2

3. (4 pts) The initial concentrations for the following reaction are [CH3I] = [Cl] = 0 M, and [CH3Cl] = [I] =
2 M. What is the concentration of the reactant [Cl] at equilibrium? Show your work! Soln = solution
Initial:
@Eq:

0M

0M

CH3I(soln) + Cl(soln)

2M

2M

2x

2x

CH3Cl(soln) + I(soln)

Keq = 9

Since the reactant concentrations are 0 M, the rxn has to shift backwards (to the left) to lose product (x) and make more
reactants (+x). So we have (2 x) on the product side and 0 + x or just x on the reactant side. Substituting these into the
equilibrium expression gives:

K eq =

[CH3Cl][I ] (2 x )(2 x ) (2 x )2
=
=
=9
[CH3I][Cl ]
( x )( x )
( x )2

(2 x )2
= 9
( x )2

or,

{ take the square root of each side to simplify:

(2 x )
= 3, or: 2 x = 3 x , 4x = 2, so x = 0.5; [Cl-] = x = 0.5 M
(x)

4. (6 pts) The initial concentrations of reactants = 0.2 M and products = 1.6 M. What is the concentration of
methanol (CH3OH) at equilibrium for the following reaction? Clearly show all your work!
Initial:
@Eq:

0.2 M

0.2 M

1.6 M

CH3OH(g) + HI (g)
0.2 + x

1.6 M

CH3I(g) + H2O(g)

0.2 + x

1.6 x

1.6 x

Keq = 25

Q=

(1.6)(1.6)
= 64
(0.2)(0.2)

Since none of the concentrations are 0, we need to plug the initial concentrations into the equib expression to solve for Q,
(rxn quotient) to see which way the rxn will go to reach equib. Q = 64, which is greater than Keq, so the reaction has to
lose products (x) and make more reactants (+x,go backwards) to reach equib!

Keq =

(1.6 x )
(1.6 x )(1.6 x ) (1.6 x )2
= 5 , multiply through each side
=
= 25 , take the square root of each side:
2
(0.2 + x )
(0.2 + x )(0.2 + x ) (0.2 + x )

by (0.2 + x): 1.6 x = 1 + 5x, or: 6x = 0.6, x = 0.1.


[MeOH] = 0.2 + x = 0.3 M

a) 0.1 M

b) 0.3 M

c) 0.6 M

d) 1.2 M

e) 1.5 M

f) 3.0 M

Homework # 4 (Equilibrium #1) 2006

5. (2 pts) Consider the potential energy diagram to the right:


Which of the following Keq values and relative reaction rates fits best? Briefly and
clearly give your reasoning in the space below.
a) Keq = 1.5, fast rxn

c) Keq = 1 x 1056, fast rxn

b) Keq = 200, slow rxn

P
R
Rxn Coordinate

e) Keq = 1 x 104, slow rxn

d) Keq = 1 x 106, very fast rxn

The products are higher in energy than reactants, so that indicates a +Grxn and Keq < 1. Of the two choices with Keq <
1.0 (c & e), the relatively high activation energy and not large energy difference between R & P indicates a moderately
small Keq and a slow reaction rate which fits e) best.

6. (4 pts) Consider the following reaction shown below. We start with [Rh4(CO)12] = 0.2 M, [H2] = 1 M, and
[HRh(CO)3] = 0 M. When the reaction reaches equilibrium there is 0.02 M HRh(CO)3. Calculate Keq for
this reaction. Clearly show all your work.
Initial:

0.2 M

1M

0M

Rh4(CO)12 + 2 H2
@ Eq:

0.2 x

4 HRh(CO)3

1 2x

0.02 M = 4x

But I told you that when the rxn reaches equilibrium there is 0.02 M HRh(CO)3, so 4x = 0.02 M, or x = 0.005 M
We can now substitute x = 0.005 into the other @Equilbrium values & get their numerical values:
[Rh4(CO)12] = 0.2 x = 0.2 0.005 = 0.195 M
[H2] = 1 2x = 1 0.01 M = 0.99 M

K eq =

Substituting these equilibrium values into the


equilibrium expression allows us to calculate Keq:

[HRh(CO)3]4
(0.02)4
1.6 10 7
=
=
= 8.38 10 7
[Rh4(CO)12][H2]2 (0.195)(0.99)2
0.191

7. (4 pts) Consider the following reaction:


P4(soln) + 5 NaClO2(soln)
0.5 x

P4O10(soln) + 5 NaCl(soln)

0.5 5x

Keq = 1 1052

5x

The initial concentrations of P4 and NaClO2 are both 0.5 M (no products present). Circle the concentration of
P4O10 at equilibrium? Clearly and briefly explain your answer below.
a) 0 M

b) 0.05 M

c) 0.1 M

d) 0.5 M

e) 0.75 M

f) 2.5 M

Because Keq is so huge, we can assume that essentially all the reactants will react to make products. But due to the
equal concentrations of reactants BUT unequal coefficents, we have a limiting reagent problem. The NaClO2 has a
coefficient of 5 and will disappear 5 times faster than the P4 reagent. So set that algebraic equation = 0: 0.5 5x = 0.
Add 5x to each side and divide through by 5 to get x = 0.1. This is the [P4O10] concentration @equilibrium and our
answer.

8. (5 pts) The initial concentration of [CO2] = 2 M, CaO(s) is present in excess, and there is no CaCO3(s).
Calculate the equilibrium concentration of [CO2].
Initial:
@ Eq:

solid

2M

CaO(s) + CO2(g)
solid

2x

CaCO3(s)
solid

Keq = 10

The key here is to realize that solids do not appear in the equilibrium expression, so: K eq =
Substituting in the algebraic terms we get: K eq =

1
[CO2]

1
1
=
= 10 ; multiplying each side by 2 x we get:
[CO2] 2 x

1 = 20 10x, solving for x we get: 10x = 19, or x = 1.9. Subtituting this into the @eq concentration for [CO2] we get:
[CO2] = 2 x = 2 1.9 = 0.1 M

CHEM 1202 - Homework # 5


Chemical Equilibrium # 2
Homework due Oct 26, 2006

ANSWER KEY

Check the box to the right if you want your graded homework to be placed out in the public rack outside
Prof. Stanleys office. Otherwise you will have to pick up your homework from Prof. Stanley in person:

1. (15 pts) Consider the following reactions:


A) 4Na(s) + O2(g)

2Na2O(s)

B) H2S(g) + Fe2+(aq)

FeS(s) + 2H+(aq)

C) 3NO2(g) + H2O(g)

2HNO3(g) + NO(g)

D) NH4NO3(s)

NH4+(aq) + NO3(aq)

E) 2Fe(CO)5(l)

Fe2(CO)9(s) + CO(g)

F) sugar(s)
G) H2O(l) + CO2(g)
H) Cl2(g) + H2(g)

Hrxn = 600 KJ/mol

sugar(soln)

Hrxn = +30 KJ/mol

Hrxn = 0 KJ/mol

H2CO3(aq)
2HCl(g)

exothermic

Based on the information above, which of the equilibria will (there can be more than one correct answer!!):

D
produce more products when heated? _____________

A, B, C, G
produce more products when the pressure is raised? _____________

produce more products when the pressure is lowered? _____________


E

be unaffected by adding or subtracting some product (so long as some remains)? ___________
A

A, H
produce more reactants when heated? _____________

produce more reactants when the pressure is raised? ___________


E

F
be generally unaffected by temperature? ________________

D, F, H
be unaffected by pressure? __________________

Homework # 5 (Equilibrium #2) 2006

2. (5 pts) The initial concentrations for the following reaction are [AgCl(s)] = excess present, [NH3] = 2M,
[Cl] = 0.01 M, and [Ag(NH3)2+(aq)] = 0.01 M. What is the concentration of [NH3] at equilibrium?
Initial:

solid

2M

AgCl(s) + 2NH3(aq)
@Equilibrium:

less solid

2 2x

0.01M

0.01M

Ag(NH3)2+(aq) + Cl(aq)
0.01 + x

Keq = 0.01

0.01 + x

The first thing to figure out was which way the reaction was going to shift to reach equilibrium. Since none of the initial
concentrations was = 0, we have to calculate the Q, the reaction quotient, and compare it to Keq to see which way the
reaction will shift to reach equilibrium. Q = (0.01)2/(2)2 = 2.5 x 105, which is less than Keq, so the reaction has to make
more product to reach equilibrium. That allows us the set the @equilibrium values shown above. Now we can plug these
values into the equilibrium expression and solve for x:

0.01 + x
(0.01 + x )(0.01 + x ) (0.01 + x )2
=
=
0
.
01
taking
the
square
root
of
each
side
gives:
= 0 .1
(2 2 x )2
(2 2 x )2
2 2x
multiplying each side by 2-2x gives: 0.01 + x = 0.2 0.2x, subtracting 0.01 and adding 0.2x to each side gives:
1.2x = 0.19, dividing through by 1.2 gives: x = 0.19/1.2 = 0.16. But x is not our answer, I asked for the concentration of
NH3 at equilibrium, which is [NH3] = 2 2x = 2 0.32 = 1.68 M. So [NH3] = 1.68 M at equilibrium.

3. (5 pts) Prof. Stanleys hydroformylation catalyst produces a 30:1 ratio of linear aldehyde product to
branched aldehyde product. If we assume that this represents an equilibrium ratio (i.e., Keq = 30 favoring the
linear aldehyde product), what is the G energy difference between the linear and branched products? Which
is more stable (lower in energy)? Please give your answer in KJ/mol.

If Keq = 30 favoring the linear aldehyde product, then the linear product is the more stable
product. To calculate the G value we simply need to plug our Keq value of 30 into the G formula
and assume that the temperature = room temperature or 298 K:
G = RT(ln Keq) = (8.31 J/mol K)(298 K)(ln(30)) = (2476 J/mol)(3.4) = 8418 J/mol
G = 8.42 KJ/mol
4. (5 pts) What is the equilibrium concentration for [SO42] in the following reaction.
Initial:
@Equilib:

solid
Al2(SO4)3 (s)
less solid

0M
0M
+3
2Al (aq) + 3SO42(aq)
2x
3x

Ksp = 108 x 1015

Since Al2(SO4)3 (s) is a SOLID, it does not appear in the equilibrium expression. Since I didnt give you any initial
concentrations you can assume that there is excess Al2(SO4)3 (s) present and NO Al+3(aq) and SO42(aq). So we know
which way the equilibrium is going to shift to fill in the zero concentrations and form more products. Now we can set up
our equilibrium expression and substitute in our @equilibrium x values:
Ksp = [Al+3]2 [SO42]3 =

108 x 1015

substituting in our x values we get: (2x)2(3x)3 = (4x2)(27x3) = 108x5

108x5 = 108 x 1015 dividing through by 108 and taking the fifth root of each side gives:
x = 1 x 103. But x is NOT our answer, the [SO42] = 3x = 3 x 103 M

CHEM 1422 - Homework # 4


Equilibrium
Due Tuesday, March 3, 2009 (2 PM)

ANSWER KEY

Check the box to the right if you want your graded homework to be placed out in the public rack outside
Prof. Stanleys office. Otherwise you will have to pick up your homework from Prof. Stanley in person:

1. (3 pts) A reaction has an equilibrium constant of 1 106 and reaches equilibrium very slowly. Circle the
following potential energy diagram that best fits this data. Briefly and clearly discuss your reasoning.
a)

b)

c)

d)
R

Rxn Coordinate

Rxn Coordinate

P
Rxn Coordinate .

Rxn Coordinate

The small K eq value means that this reaction is non-spontaneous (G = positive, reactants are lower in energy than
products). Thus, you can rule out c) and d) since these are both spontaneous reactions that will have K eq > 1. a) and b)
are both non-spontaneous reactions. The height of the activation barrier has nothing to do with the thermodynamic s or
Keq, but does control the speed or rate of reaction. The fact that the rxn reaches equilibrium slowly implies the larger
activation barrier, which fits b). But if your reasoning is OK you could also select a) and get full credit.

2. (3 pts) Consider the following equilibrium:


3C2H2(g)

K eq = 1 1018

C6H6(g)

If one starts with 6 M acetylene (C 2 H2) and lets the reaction reach equilibrium, what will be the equilibrium
concentration of benzene (C 6H6)? Circle the answer and clearly discuss your reasoning.
a) 0 M

b) 0.6 M

c) 1 M

d) 2 M

e) 6 M

Always think qualitatively about the problem and what the K eq value is telling you. The huge K eq value means that this
reaction goes essentially to completion. So we will have all products and no reactants at equilibrium. The stoichiometry
of the reaction, however, is that 3 molecules of acetylene combine to make one molecule of benzene. So if we start with
6 M acetylene, we will end up with 2 M benzene (3:1 reduction ratio).

3. (3 pts) The initial concentrations of reactants and products are all 2 M. What is the concentration of
methanol (CH 3OH) at equilibrium for the following reaction? K eq = 25 Circle the answer and clearly
show your work!!
CH3OH(g) + HI (g)
a) 0 M
Initial:

b) 0.33 M
2M

2M

CH3 OH(g) + HI (g)


@ eq:

2x

2x

CH3 I(g) + H2O(g)

c) 0.66 M
2M

d) 1.00 M
2M

CH3I(g) + H2O(g)
2+x

2+x

e) 1.33 M

f) 2.66 M

If all the concentrations are initially equal and


Keq = 25, then the reaction has to mak e more
products so the ratio of products over reactants
is > 1. (Q < K eq)

(2 x )
[CH3I ][H 2O] (2 x )2
5
Keq

25 now take the square root of both sides:


[CH3OH ][HI ] (2 x )2
(2 x )
rearrange and solve for x: (2 + x) = (5)(2 x); 2 + x = 10 5x; 6x = 8; x = 8/6 or x = 1.33 (but this isnt the answer!!)
The concentration of MeOH at equilibrium, therefore, is: 2 x or 0.67 M

CHEM 1422 HW # 4 Equilbrium

4. (4 pts) Calculate the concentrations for all species at equilibrium for the following reaction. The initial
concentrations are [H2 ] = [I 2] = 0 M, [HI] = 4 M. Clearly show your work.
Initial:

0M

0M

4M

H2 (g) + I 2(g)
@ eq:

2HI(g)

K eq = 36 (@ 1200 K)

4 2x

plug the algebraic expressions into the equilibrium formula:

(4 2x )
[HI ]2
( 4 2x )2
6
Keq

36 now take the square root of both sides:


x
[H 2][I 2]
( x )2
rearrange and solve for x: 4 2x = 6x; 4 = 8x; x = 0.5.
Now plug x into the @eq conditions to solve for the numerical values:

so,

[H2] = [I 2] = 0.5 M and [HI] = 3 M

5. (6 pts) Consider the following reactions:


A) Br 2(aq) + 2Cl (aq)

Cl2 (g) + 2Br (aq)

Hrxn = 68 kJ/mol

B) AgCl(s) + 2NH3 (aq)

Ag(NH 3) 2+ (aq)

Hrxn = 13 kJ/mol

C) 2N 2O(g)

Hrxn = 30 kJ/mol

2N2(g) + O 2(g)
2H2O(l) + SiF6 2 (aq) + 2H+ (aq)

D) SiO 2 (s) + 6HF(aq)


E) Rh(H)(CO)(PPh3) 2(aq) + CO(g)

Rh(H)(CO)2(PPh3)(aq) + PPh3(aq)

F) hemoglobin(aq) + 4O 2(g)
G) 2H 2O 2 (aq)
H) S2(aq) + Fe2+(aq)
I ) H 2(g) + D 2(g)

hemoglobin(O 2 )4 (aq)

2H2O(l) + O2 (g)

exothermic

FeS(s)
Hrxn = 0 kJ/mol

2HD(g)

Based on the information above, which of the equilibria will:

produce more products when heated? _____________


A

E, F
produce more products when the pressure is raised? _____________

be unaffected by adding or substracting some product (so long as some remains)? ___________
H

B, C, G
produce more reactants when heated? _____________

produce more reactants when the pressure is raised? ___________


A, C, G

be unaffected by temperature? _______


I

be unaffected by pressure? __________________


B, D, H, I

If you dont have any Hrxn information (or k now that it is


exo- or endothermic, you can NOT state that a reaction is
unaffected by temperature.

CHEM 1422 HW # 4 Equilbrium

6. (4 pts) The initial concentrations for the following reaction are [CH3 I] = [F ] = 1 M, and [CH3F] = [I ] = 9
M. What will be the concentrations of each species at equilibrium? Clearly show all your work.
Initial:

1M

1M

9M

CH3I(aq) + F(aq)
@Equilibrium:

1+x

9M

CH3F(aq) + I (aq)
9x

1+x

Keq = 16

9 x

(9 x )2
(9 x )
Keq
16 , take the square root of each side to give:
4
2
(1 x )
(1 x )
multiply out to get: 9 x = 4 + 4x, rearrange to get: 5x = 5, or x = 1. Plug x = 1 back into the
equilibrium conditions to get:
[CH3I] = [F ] = 2 M, [CH3F] = [I ] = 8 M

First you have to


calculate Q (rxn
quotient) to figure out
which way the rxn will
go to reach equilib .
Q = 81, which is larger
than Keq, so the
reaction has to go
BACKWARDS to
reach equilib .

7. (4 pts) Which of the following salts is the least soluble (i.e., will give the lowest Pb+2(aq) concentration)?
Circle your answer. Calculate the concentration of [Pb +2] for the answer and put it and the calculation
details below.
a) PbCO 3 (K sp = 1 1013)
c) Pb(CrO 4) (K sp = 1 1014)

b) Pb3(AsO 4 )2 (Ksp = 1.1 1036)


d) Pb(OH) 2 (K sp = 4 1016)

e) Pb3(PO 4 )2 (K sp = 1.1 1044)

f) PbS (K sp = 1 1024)

a) x 2 = 1 1013 ; x = [Pb2+] = 3.2 107 M

b) 108x 5 = 1.1 1036 ; 3x = [Pb2+] = 7.5 108 M

c) x 2 = 1 1014 ; x = [Pb2+] = 1 107 M

d) 4x 3 = 4 1016 ; x = [Pb2+] = 4.6 106 M

e) 108x 5 = 1.1 1044 ; 3x = [Pb2+] = 1.9 109 M

f) x 2 = 1 1024 ; x = [Pb2+] = 1 1012

8. (3 pts) What is the equilibrium concentration of Ag(aq) in the presence of 1 M CrO 42 (aq)? Clearly show
all your work.
Initial:

excess

Ag2CrO 4 (s)
@Equilibrium: less excess

1M

2Ag+ (aq) + CrO 4 2 (aq)


2x

Ksp = 4 1012

1+x

Ksp = [Ag+]2 [CrO42] = (2x)2 (1 + x) = 4 1012 : make the approximation that x << 1 M to simplify the algebra
(2x)2 (1) = 4 1012
4x 2 = 4 1012 , now divide each side by 4 to give: x 2 = 1 1012 ; take the square root of eac h side:
x = 1 106 ; but the concentration of A g+ is 2x, not just x, so:

[Ag+] = 2x = 2 106 M

ANSWER KEY

CHEM 1202 - Homework # 6


Acids & Bases # 1
Due Nov 14th, 2006 by 2PM
1. (5 pts) What are the pHs for the following solutions?
a) 1 M HI = 0

b) 0.1 M HClO4 =

d) 0.001 M NaOH = 11

c) 10 M LiOH = 15

e) 1 1020 M Ba(OH)2 = 7

1 x 1020 M is much, much lower than the 1 x 107 M


OH present in water that it makes no contribution to
the pH. It doesnt really matter whether Ba(OH)2 is a
strong or moderate base in this case.

2. (5 pts) Why are strong acids strong acids. Specifically discuss the series HF, HCl, HBr, and HI and why HF
is a weak acid and HI is one of the strongest acids known.
Strong acids are strong acids because their conjugate bases (the counter-anions) are
extremely weak conjugate bases. The Cl, Br and I anions have decreasing affinities for
binding H+ in aqueous solution (or in the molecular gas phase species for that matter). The
larger size of the anions as we go from Cl to I means that the negative charge on the anion
is spread out over a larger surface area and there is less electrostatic attraction to the H+
cation. Remember that the extremely small H+ cation has a very concentrated positive charge
and will interact the strongest with a small concentrated negative (or multiple negative)
charge. The smallest common anion is F, which does have a fairly strong electrostatic
attraction to the H+ cation, even in water. This makes HF a weak acid compared to HCl, HBr,
and HI.

3. (5 pts) Calculate the Ka value of stanoic acid (a monoprotic acid) if a 0.01 M solution has a pH = 4. Clearly
show all your work.
Initial:

0.01M
HA(aq)

@ Equib:

0.01 x

0M
0M
H+(aq) + A(aq)
x

BUT, before you go any further, Ive given you the pH of the solution in the problem!! So you dont
have to solve for x, you already know it!! So all we have to do is take the anti-log of pH 4 to get the
concentration of the H+ (and A) in solution:
[H+] = antilog(4) = 1 104 M

Ka =

Substituting this into the equlibrium expression gives us:

[H + ][OH ] (1 10 4 )(1 10 4 ) 1 10 8
=

= 1 10 6

4
[HA]
(0.01 1 10 )
0.01

Note that we can approximate 0.01 1 x 104 M as ~ 0.01. So the answer is:
Ka = 1 x 106

4. (5 pts) What is the pH of a 1 M solution of the base amyl amine. Kb = 1 x 108 Clearly show all your
work. Circle the correct answer from those given below. No credit will be given if work is not shown.
Initial:

1M

pure liq

1x

0M

BaseH+(aq) + OH(aq)

Base(aq) + H2O(l)
@ Equib:

0M

pure liq

Since I was nice and gave you Kb for this basic equilibrium, you can go directly to setting up your
equilibrium expression: Kb = ( x )( x ) = 1 10 8
(1 x )

( x )( x )
= 1 10 8
(1)

assume that x << 1,

or x2 = 1 x 108, or x = [OH] = 1 x 104

pOH = log(1 x 104) = 4. BUT THIS IS NOT YOUR ANSWER, since I asked for the pH!!
pH = 14 pOH = 10
a) -2

b) 0

c) 4

d) 7

e) 10

5. (5 pts) Calculate the pH of a 0.1 M solution of an acid that has a pKa of 5.0. Clearly show all your work.
Circle the correct answer from those given below. No credit will be given if work is not shown.
Initial:
@ Equib:

0.1M

0M

HA(aq)

0M
H+(aq) + A(aq)

0.1 x

Convert the pKa of 5.0 into a Ka value: Ka = antilog(pKa) = 1 x 105


solve for the [H+] and the pH.

Ka =

( x )( x )
= 1 10 5
( 0 .1 x )

or x = [H+] = 1 x 103
a) 1

b) 1

assume that x << 0.1,

x2
= 1 10 5 , or x2 = 1 x 106,
(0.1)

So the pH = log(1 x 103) = 3


c) 3

Now, we can set-up and

pH = 3
d) 7

e) 11

6. (5 pts) An unknown acid solution has a pH of 5. What important, but simple, piece of information do you
need to tell if the solution is a strong or weak acid (aside from the Ka value)?
You need to know the molarity (or concentration) of the solution! The pH by itself only tells
you the H+ concentration, NOT whether the acid it came from is a strong or weak acid. For
example, a 1 x 106 M solution of HCl has a pH of only 6, which is just barely acidic, but HCl is
a strong acid. So unless you know both the pH and concentration of the added acid, you can
not directly tell if you are dealing with a strong or weak acid. From the pH and concentration
of added acid you can calculate the Ka value, which will tell you if you have a strong, medium
or weak acid. Of course, if you have an extremely low pH, like 0 to 1, you have to be dealing
with a strong acid since even very concentrated weak acids cant produce that kind of H+
concentration.

ANSWER KEY
Name ___________________________

CHEM 1202 - Homework # 7 & 8


Acids & Bases 2
Due Tuesday, Nov 28th, 2006 by Noon
Table 1. Dissociation Constants for some Acids.
Acid

pKa Value

Acid

pKa Value

Acid

pKa Value

NH4+

10

HBF4

formic

HClO

H2CO3

benzoic

1. (5 pts) Which of the acids listed in Table 1, given a 0.01 M solution in water, will have a pH closest to 2?
a) NH4+
b) benzoic
c) H2CO3
d) formic
e) HBF4
2. (5 pts) Which of the acids listed in Table 1, when reacted with an equivalent amount of NaOH, will form a
solution with the highest pH?
b) benzoic
c) H2CO3
d) formic
e) HBF4
a) NH4+
3. (5 pts)
a)
b)
c)
d)
e)

Order the acids in Table 1 from strongest to weakest. Circle the correct choice.
benzoic > formic > H2CO3 > HBF4 > NH4+ > HClO
HBF4 > formic > benzoic > H2CO3 > HClO > NH4+
NH4+ > benzoic > HBF4 > formic > H2CO3 > HClO
benzoic > HBF4 > NH4+ > formic > H2CO3 > HClO
NH4+ > HClO > H2CO3 > benzoic > formic > HBF4

4. (5 pts) What is the pH of a 0.01 M solution of the weak base benzylamine (C6H5CH2NH2)? pKa = 8.
Circle the answer below and clearly show all your work.
Initial:

0.01M

pure liq

0M

BaseH+(aq) + OH(aq)

Base(aq) + H2O(l)
@ Equib:

0.01 x

0M

pure liq

You first need to convert the pKa into a Ka, then into a Kb for this basic equilibrium:
K
1 10 14
( x )( x )
Kb = w =
= 1 10 6 now you can setup your equilib: Kb =
= 1 10 6

8
(0.01 x )
Ka 1 10
assume that x << 0.01,

( x )( x )
= 1 10 6 or x2 = 1 108, or x = [OH] = 1 104 pOH = 4.
(0.01)

BUT THIS IS NOT YOUR ANSWER, since I asked for the pH!! pH = 14 4 = 10

a) 4

b) 5

c) 9

d) 10

e) 13

5. (10 pts) Will FeCl3 generate an acidic, neutral, or basic solution when dissolved in water. Clearly discuss
your reasoning.
FeCl3 is composed of a Fe3+ cation and three Cl anions. The Cl anions are do nothing or neutral (acidbase property, extremely poor conjugate base) so will not make a basic solution. The Fe3+ cation, on the other
hand, is a Lewis acid and will react with water to release H+: Fe3+(aq) + H2O
[Fe(OH)]2+ + H+(aq).
This is called hydrolysis and most +2 or higher cations can interact with water in a similar fashion to release H+
and make an acidic solution. Note that the [Fe(OH)]2+ is not a typical hydroxide base as the Fe-OH bond is
reasonably strong and will not dissociate OH.

Homework # 7&8 - Acids/Bases 2 (2006)

6. (5 pts) What is the pH of a 1 M solution of KClO? See Table 1 for pKa values. Clearly show all your work.
The first thing to realize is that this is a basic salt that will generate a basic solution! K+ is a do-nothing cation, while
ClO is an active anion that acts as a weak base in solution.
Initial:

1M

pure liq

0M

Base(aq) + H2O(l)
@ Equib:

1x

0M

BaseH (aq) + OH(aq)

pure liq

Next you need to convert the pKa value Ive given you into a Ka, then Kb for the conjugate base in this basic equilibrium:
K
1 10 14
( x )( x )
Kb = w =
= 1 10 6 now you can setup your equilib: Kb =
= 1 10 6 assume that x << 1,
(1 x )
Ka 1 10 8
2
6
3 pOH = 3. pH = 14 pOH = 11 so the pH = 11

( x )( x )
6 or x = 1 10 , or x = [OH ] = 1 10
(1)

= 1 10

7. (10 pts) Consider the following list of salts:


A) NH4Cl

B) KI

C) CsF

D) potassium benzoate

E) MoCl4
I) KClO4

F) BaI2
J) NaClO

G) AlBr3

H) LiNO3

Which salts will generate an acidic solution? ________________


A, E, G
Which salts will generate a basic solution? _________________
C, D, J
Which salts will generate a neutral solution? _________________
B, F, H, I
8. (5 pts) Calculate the pKb of the weak base phenylamine if a 1 M solution has a pH = 10.
Initial:

1M

pure liq

0M

BaseH+ (aq) + OH(aq)

Base(aq) + H2O(l)
@ Equib:

1x

0M

pure liq

Kb =

[baseH+][OH ]
[base]

But I have given you the pH, so you know the OH concentration (and [BaseH+]) at equilibrium. If pH = 10, pOH = 4 and
the [OH] = 1 104 M. This is also small enough relative to 1 x, that we can drop the x here and really make a very
simple expression to solve for Kb:

Kb =

[baseH+][OH ] (1 10 -4)(1 10 -4)


=
= 1 10 8
[base]
1

so

pKb = 8

9. (10 pts) What is the pH if 800 mL of 0.125 M KOH is added to 200 mL of 0.5 M sucoloic acid (a
monoprotic acid)? pKa = 11 (clearly show all your work)
The first thing to realize is that sucoloic acid is a weak acid and that we are titrating it with a strong base. If there are equal amounts of
each, we will be making the salt of a weak acid, which is a weak base! So we may be generating a basic solution depending on the
amount of acid and base reacting. Convert the # of mL and molarity of each into moles:
# moles base = (800 mL OH)(0.125 M OH) = 100 mmoles OH
# moles weak acid = (200 mL weak acid)(0.5 M weak acid) = 100 mmoles weak acid
So we have an equal amount of each. Dont forget that we are adding 800 mL to 200 mL to make 1000 mL total solution volume. This
will generate a 100 mmoles/1000 mL = 0.1 M solution that will act as a weak base:

Initial:
@ Equib:

0.1M
pure liq

Base (aq) + H2O(l)


0.1 x

pure liq

0M

0M
BaseH (aq) + OH(aq)
x

Next you need to convert the pKa value Ive given you into a Ka, then Kb for the conjugate base in this basic equilibrium:
1 10 14
K
( x )( x )
Kb = w =
= 1 10 3 now you can setup your equilib: Kb =
= 1 10 3 assume that x << 1,
(0.1 x )
Ka 1 10 11
( x )( x )
pH = 12
3 or x2 = 1 104, or x = [OH] = 1 102 pOH = 2. pH = 14 pOH = 12 so the
(0.1)

= 1 10

CHEM 1202 - Homework # 6


Acids & Bases # 1
Due Thursday, March 19, 2009

ANSWER KEY
These compounds are the bases you
need to be able to tell acids from bases

1. (3 pts) Consider the following weak acids and bases and their pKa values:
O

A)

(pKa = 3.8)

D) N(CH3)3 (pKa = 12)

B) HCN (pKa = 9.3)

C) H2CO3 (pKa = 6.4)

E) H2SO3 (pKa = 1.8)

F) NH3 (pKa = 9)
The strongest acid has the lowest pKa value

Which compound is the strongest acid (use letter) ? _______


E
Which compound is the strongest base (use letter) ? _______
D
Which compound has the strongest conjugate base (use letter)

The strongest base has the highest pKa value. Note


that the pKa actually refers to the strength of the
conjugate acid derived from the base
The weakest acid has the strongest
? ______
B conjugate base. The weakest acid has the
largest pKa value. You need to be able

to tell an acid from a base.

2. (5 pts) What are the pHs for the following solutions?


a) 0.1 M HBr

d) 0.1 M NaOH =

b) 10 M H2SO4 =

13

e) 10 M CsOH =

15

c) 1 1010 M HNO3 = 7

3. (4 pts) What is the pKa value of a 0.1 M solution of palmetic acid (HA) that has a pH of 5? Clearly show all
your work and put a box around your answer.
Initial:
@ Equib:

0.1 M

0M

HA(aq)

H+(aq) + A(aq)

0.1 x

0M

BUT, before you go any further, Ive given you the pH of the solution in the problem!! So you dont have to solve for x,
you already know it!! So all we have to do is take the anti-log of pH 5 to get the concentration of the H+ (and A):
[H+] = antilog(5) = 1 x 105 M

Ka =

Substituting this into the equlibrium expression gives us:

(1 10 5 ) 2
1 10 10
=
= 1 10 9
0.1 1 10 5
0 .1

so, Ka = 1 x 10 9

pKa = log(Ka) = 9

Note that it is fine to drop the x in the 0.1 x expression because x is much smaller than 0.1. So the answer is:

4. (3 pts) Calculate the pH of a 0.01 M solution of acid that has a pKa of 6.0. Clearly show all your work and
put a box around your answer.
Initial:

@ Equib:

0.01 M

0M

0M

HA(aq)

H+(aq) + A(aq)

0.01 x

First, convert the pKa of 6 into a Ka value: Ka = antilog(pKa) = 1 x 106 Now, we can solve for the [H+] and the pH.
x2
( x )( x )
= 1 10 6 , or x2 = 1 x 108,
= 1 10 6 assume that x << 1,
Ka =
(0.01)
(0.01 x )
or x = [H+] = 1 x 104

So the pH = log(1 x 104) = 4

pH = 4

CHEM 1422 HW # 5 Acids & Bases #1

5. (5 pts) What is the pH of a 0.1 M solution of the base ethyl amine (CH3CH2NH2). Ka = 1 1011 Clearly
show all your work and put a box around your answer.
Initial:

0.1 M

pure liq

0M

BaseH+(aq) + OH(aq)

Base(aq) + H2O(l)
@ Equib:

0.1 x

0M

pure liq

You first need to convert the Ka into a Kb for this basic equilibrium:

K
1 10 14
( x )( x )
Kb = w =
= 1 10 3 -- now you can setup your equilib: Kb =
= 1 10 3

11
( 0 .1 x )
Ka 1 10
( x )( x )
assume that x << 1,
= 1 10 3 or x2 = 1 x 104, or x = [OH] = 1 x 102 pOH = 2. BUT THIS IS NOT YOUR
(0.1)
ANSWER, since I asked for the pH!! pH = 14 2 = 12

pH = 12

6. (5 pts) What is the Kb of a 0.01 M solution of a base that has a pH of 10? Clearly show all your work and
put a box around your answer.
Initial:
@ Equib:

0.01 M
pure liq

Base (aq) + H2O(l)


1x
pure liq

@ Equib:

104

0.01 1 x

Plug into the Kb expression and solve: K b =

0M

0M
BaseH (aq) + OH(aq)
x
1x

104

A pH of 10 means that the pOH is 4.


We do NOT use the pH directly in this
problem because we are dealing with
a basic equilibrium!! Thus, we know
that the [OH] = 1 x 104 M.

1 x 104

(1 10 4 )2
= 1 10 6
0.01 1 10 4

Kb = 1 x 106

You can drop this since it is small compared to 0.01 M

7. (5 pts) Which of the following acids is the strongest based on its structure and atoms present? Clearly
discuss your reasoning.
CH
H+

F
F
HPF6 is an extremely strong acid, while HPO2(OCH3)2
- or P
P O CH3
is a rather weak acid. The two main reasons is that the
F
F
O
negative charge for the [PF6]- anion is spread out evenly
O
F
H+
over the 6 flourine atoms making for a very dilute charge.
In contrast, the negative charge in the [PO2(OCH3)2]- anion
is localized (concentrated) on one (or at most two) oxygen atoms. More localized negative charges will more
strongly attract the H+ cation. This is why F- (small concentrated charge) by itself is a moderately good
conjugate base for the weak acid HF. But the [PF6]- anion is not a simple F- anion, its charge is spread out
over 6 flourine atoms.
The other significant factor is that fluorine has a valence of 1, that is, it only wants to make one chemical
bond to another atom. In the [PF6]- anion each fluorine is already coordinated to the central phosphorus atom
and wont want to make another covalent bond to a H+. The [PO2(OCH3)2]- anion, on the other hand, has one
oxygen atom with only a single bond to the central phosphorus atom (and a negative charge). Oxygen atoms
like to have two chemical bonds in general (H2O !!) so it will have a fairly strong tendency to want to make a
chemical bond to the H+. Remember that OH- loves to coordinate to H+. The PO3 group attached to the O- is
electron-withdrawing, so the P-O- group will not be as electron-rich as OH- and will not want to bond to the H+
nearly as strongly.

CHEM 1422 - Homework # 6


Acids & Bases # 2
Due Tuesday, April 14, 2009

ANSWER KEY

1. (5 pts) 250 mL of 0.2 M acetic acid (HOAc) reacts with 250 mL of 0.2 M NaOH. What is the pH of the
resulting solution? pKa (HOAc) = 5
The first thing to realize is that one is reacting a weak acid (acetic acid) with a strong base (NaOH). This will generate the
salt of a weak acid (sodium acetate, NaOAc), which is a weak base. There are equivalent amounts of acid and base
present (same volume and same concentration) and since one is doubling the solution volume (250 to 500 mL) the
concentration is being halved to 0.1 M. So what I am asking you to calculate is the pH of a 0.1 M solution of sodium
acetate (the salt of a weak acid). This will be a basic equilibrium for which you need to use a Kb value.
Initial: 0.1 M
-0
0
pKb = pKw pKa = 14 5 = 9
OAc (aq) + H2O(l)
HOAc(aq) + OH (aq)
Kb = antilog(9) = 1 109
@eq: 0.1 x
-x
x
Plug the @equilibrium values into the equilbrium expression and solve for x:
( x )( x )
( x )( x )
2
10
1 10 9 or x = 1 10 ,
Kb
1 10 9 assume that x << 0.1 since Kb is so small,
(
0
.
1
)
(0.1 x )
or x = [OH] = 1 105 so the pOH = 5. BUT THIS IS NOT YOUR ANSWER, since I asked for the pH!!
pH = 14 pOH = 9 so the

pH = 9

2. (5 pts) Identify whether the following 1:1 solutions will be acidic, basic, or neutral:
a)
b)
c)
d)
e)

ACIDIC (buffer solution, pH = 4.7)


BASIC (buffer solution, pH = 9.3)
Neutral (not a buffer!! neutral salts!!)
ACIDIC (buffer solution, pH = 2.1)
BASIC (buffer solution, pH = 10.6)

NaOAc/HOAc (pKa = 4.7)


NH3/NH4NO3 (pKa = 9.3)
NaCl/KNO3
H3PO4/NaH2PO4 (pKa = 2.1)
CH3NH2/CH3NH3Cl (pKa = 10.6)

Table 1. Indicators
Name

Acid color

Methyl violet

pKa
1

yellow

violet

methyl yellow

1.7

red

yellow

methyl orange

3.5

red

yellow

red

yellow

methyl red

Base Color

Name

Acid color

Base Color

bromthymol blue

pKa
7

yellow

blue

thymol blue

8.8

yellow

blue

phenolphthalein

colorless

pink

Alizarin yellow

11

yellow

red

3. (3 pts) What is the approximate pH of a colorless solution that turns yellow if a small amount of methyl
yellow, or alizarin yellow is added to it, but turns pink if phenolphthalein is added. See Table 1 for
information about indicators. Circle your answer.
a) 0 to 1

b) 4 to 5

c) 7 to 8

d) 9 to 10

e) 13 to 14
12

4. (2 pts) Which indicator (see Table 1) would work best to indicate the
equivalence point for the titration curve shown to the right (circle answer):
a) methyl violet

b) methyl red

d) phenolphtalein

e) alizarin yellow

c) thymol blue

10

pH

8
6
4

Equivalence point around pH 4,


need to use an indicator with a
color change around 4.

2
0
0

10

20

30

40

mL of HCl added

50

CHEM 1422 HW # 6 Acids/Bases 2

5. (3 pts) What is the pH of a 0.01 M solution of NaHCO3? pKb = 4. Clearly show all your work.

The first thing to realize is that this is a basic salt that will generate a basic solution! Na is a do-nothing cation, while
HCO3 (bicarbonate) is a basic anion that acts as a weak base in solution, even though it has a proton attached to it.
With very strong bases like hydroxide, the HCO3 (bicarbonate) can act as a proton donor, but typically not by itself.
Initial:

0.01 M

pure liq

0M

0.01 x

Since I gave you a pKb, which is appropriate


for a base equilibrium, you only have to
convert it to a Kb value:
Kb = antilog(4) = 1 104

H2CO3 (aq) + OH(aq)

HCO3(aq) + H2O(l)
@ Eq:

0M

pure liq

Your equilib setup should look like this: Kb

( x )( x )
1 10 4 assume that x << 0.01,
(0.01 x )

( x )( x )
1 10 4 or x2 = 1 106, or x = [OH] = 1 103 pOH = 3. BUT THIS IS
(0.01)
NOT YOUR ANSWER, since I asked for the pH!! pH = 14 pOH = 11 so the

pH = 11

Note that the x approximation


almost doesnt work here. But
it is an order of magnitude
smaller than 0.01, so it is just
barely reasonable to drop it.

6. (4 pts) Consider the following list of salts:


A) CsNO3

B) TiCl4

C) Na(SH)

D) KF

E) Ca(OH)2

F) Ti(ClO4)4

G) [HN(CH3)3]Cl

H) LiHCO3

I) NaPF6

J) RbBr

B, F, G
Which salts will generate acidic solutions? ________________
C, D, E, H
Which salts will generate basic solutions? _________________
A, I, J
Which salts will generate neutral solutions? _________________
7. (4 pts) What is the pH if 500 mL of 0.2 M HCl is added to 500 mL of 0.2 M ammonia (NH3, pKb = 5)
(clearly show all your work)
The first thing to realize is that one is reacting a weak base (ammonia) with a strong acid (HCl). This will generate the
salt of a weak base (ammonium chloride, NH4Cl), which is a weak acid. There are equivalent amounts of acid and base
present (same volume and same concentration) and since one is doubling the solution volume (500 to 1000 mL) the
concentration is being halved to 0.1 M. So what I am asking you to calculate is the pH of a 0.1 M solution of ammonium
chloride (the salt of a weak base). This will be an acidic equilibrium for which you need to use a Ka value.
Initial: 0.1 M
0
0
pKa = pKw pKb = 14 5 = 9
+
+
NH4 (aq)
H (aq) + NH3 (aq)
Ka = antilog(9) = 1 109
@eq: 0.1 x
x
x
Plug the @equilibrium values into the equilbrium expression and solve for x:
( x )( x )
( x )( x )
2
10
1 10 9 or x = 1 10 ,
Kb
1 10 9 assume that x << 0.1 since Ka is so small,
(0.1)
(0.1 x )
or x = [H+] = 1 105 so the pH = 5.
pH = 5

8. (4 pts) What is the pH of the following 0.5 M aqueous solutions containing equal amounts of the two
components shown (find pKa/b or Ka/b values in your textbook appendix or lecture notes):
a) NH4Cl + NH3

pH = 9.3

b) HNO2 + KNO2

pH = 3.3

c) H2S + CsHS

pH = 7.0

d) C5H4N (pyridine) + [C5H4NH]Br

pH = 5.2 (pyridine is a weak enough base that its conjugate acid


takes over and makes the solution acidic)

1:1 mixtures of weak acids or bases and their salts


form buffers whose pH = pKa of the weak acid or the
acidic salt of a weak base. For weak bases buffers
the pH = 14 pKb. See the Henderson-Hasselbach
equations near the end of the Acid-Base chapter!!

CHEM 1202 - Homework # 9 & 10


Redox & Electrochemistry
Due Friday, Dec 8th, 2006 by Noon

ANSWER KEY

Check the box to the right if you want your graded homework to be placed out in the public rack outside
Prof. Stanleys office. Otherwise you will have to pick up your homework from Prof. Stanley in person:

1. (5 pts) Which of the following substances is the best reducing agent?


a) F

c) Li+

b) Mg

d) Ag+

e) Zn

2. (5 pts) Which of the following substances is the best oxidizing agent?


a) F

b) Mg2+

d) Ag+

c) O3

e) Cu
Two of the sulfur atoms have +2
oxidation state while the other
two have +3, only the two that
are oxidized should be used in
figuring the # of e- transferred.

3. (10 pts) Balance the following reaction in acidic solution.

+2, +3 2
1
S4O62(aq) + Cl(aq)
Use the half-cell method. Write out the oxidation and reduction half cells and balance based on # of electrons:
1st determine ox states:

+2 2
2 +1
2 S2O32(aq) + OCl(aq)

Oxidation half cell: 2S2O32(aq)


Reduction half cell: OCl(aq) + 2e-

S4O62(aq) + 2eCl(aq)

We need two S2O32- to produce one S4O62-. At


least four redox active sulfur atoms on each side
are needed to balance the rxn

Since we have the same # of electrons on each side of the reaction, we can simply add them together to get our core
redox-balanced rxn (otherwise we would have to multiply each rxn by an integer # to get the same # of e- on each side):
2S2O32(aq) + OCl(aq)

S4O62(aq) + Cl(aq)

Now we need to balance the oxygen atoms on each side by adding H2Os to the side missing oxygen atoms and the
appropriate # of H+ to the opposite side of the rxn. The product side is missing 1 oxygen atom so we need to add one
water and then 2 H+ to the reactant side. This gives us our final balanced equation:
2H+(aq) + 2S2O32(aq) + OCl(aq)

S4O62(aq) + Cl(aq) + H2O

4. (10 pts) Balance the following reaction in basic solution:


+5 2
+3
+3
+6 2

3+
3+
BiO3 (aq) + Cr (aq)
Bi (aq) + CrO42(aq)
Write out the oxidation and reduction half cells and balance based on # of electrons:
1st determine ox states:

Oxidation half cell: Cr3+(aq)


Reduction half cell: BiO3(aq) + 2e-

CrO42(aq) + 3eBi3+(aq)

Since we do NOT have the same # of electrons on each side of the reaction, we to multiply each rxn by an integer # to get
the same # of e- on each side. 6e- is the common factor, so we need to multiply the Cr3+ half cell by 2 and the BiO3 half
cell by 3 and add them together to get the core redox-balanced rxn:
3BiO3(aq) + 2Cr3+(aq)

3Bi3+(aq) + 2CrO42 (aq)

We now need to balance the oxygen atoms and H+, then convert to basic solution by adding the same # of OH anions
to each side of the rxn as we have H+, reacting them to make 2 H2O molecules, then canceling out the redundant H2Os:
ACID solution:

3Bi3+(aq) + 2CrO42 (aq) + H2O

2H+ (aq) + 3BiO3(aq) + 2Cr3+(aq)

BASIC Solution:

3Bi3+(aq) + 2CrO42 (aq) + 2OH (aq)

H2O (aq) + 3BiO3(aq) + 2Cr3+(aq)

5. (5 pts) Write the oxidation state for the underlined element in the box following each compound.
a) LiAlH4
d) CaSO3

1
+4

b) Ba3(AsO4)2

+5

e) H2O2

c) Na2NiCl4

+2

Homework 9&10 Redox/Electrochemistry (2006)

6. (15 pts) Calculate the redox potentials for the following reactions. Show the two half cell reactions, written
in the proper direction and their potentials used to calculate your answer.
2H(soln) + 2Li+(soln)

a) H2(g) + 2Li(s)
H2 + 2e2Li

2H

= 2.25 V
= +3.05 V

2Li+ + 2e-

b) 4H+(aq) + O2(g) + 2Cu(s)


4H+ + O2 + 4e2Cu
c) F2(g) + 2Cl(aq)
F2 + 2e2Cl

E = + 0.80 V

2H2O + 2Cu2+(aq)

E = + 0.89 V

2H2O = +1.23 V
= 0.34 V

2Cu2+ + 4e-

2F(aq) + Cl2(g)
2F
= +2.87 V
Cl2 + 2e = 1.36 V

E = + 1.51 V

d) Cu(s) + 2Ag+(aq)
Cu2+(aq) + 2Ag(s)
2Ag
= +0.80 V
2Ag+ + 2eCu
Cu2+ + 2e = 0.34 V

E = + 0.46 V

e) 3Pb2+(aq) + 2Al(s)
3Pb(s) + 2Al+3(aq)
2+
3Pb + 6e3Pb
= 0.13 V
3+
2Al
2Al + 6e = +1.66 V

E = + 1.53 V

7. (10 pts) Library/web research topic: Describe in your own words the chemistry (with formulas) involved in
a lithium-ion battery. Is lithium metal used? What is the voltage of this electrochemical reaction? List two
main advantages and two main disadvantages of lithium-ion batteries with BRIEF explanations. DO NOT
COPY DIRECTLY FROM ANY REFERENCE (except for chemical formulas). List your primary reference
used at the end.
Lithium ion rechargeable batteries have anodes composed of carbon/graphite (represented by C6 in the formula below)
while the cathode is composed of Li+[CoO2] (Co3+ oxidation state) or LiMn2O4 (mixed Mn3+/Mn4+). When the battery is
charged some of the carbon anode is reduced to [C6]x (where x = # of electrons transferred) and some of the the Co3+ in
the cathode is oxidized to a +4 oxidation state. Enough Li+ cations migrate to the anode to balance the negative charge
build-up. This forms a mixture of [Li+]x[C6]x at the anode (electron source) and oxidized CoO2 at the cathode (electron
acceptor). Li metal is NOT formed unless something goes wrong during the charging process! When the battery
discharges (produces electricity) the electrons flow from the reduced anode to the oxidized cathode through the external
circuit (wire) reducing the CoO2 to [CoO2]. The Li+ cations migrate back from the carbon anode to the cathode to reform
LiCoO2. There is an electrolyte typically composed of either a polar organic gel and a lithium salt or a nano-porous
polymer (Li-polymer-ion). This makes the Li+ cation migration possible between the anode and cathode.
Reaction: [Li+]x[C6]x + xCoO2

C6 + xLiCoO2

The voltage produced is about 3.6 V.


Advantages: light weight & high power density (voltage & amps; very good for small electronic devices); no memory
effect (doesnt matter how often you charge or discharge, charging after light use does not reduce the battery capacity);
low self-discharge (built-in computer chip for monitoring battery stats does slowly drain battery).
Disadvantages: limited lifetime (aging will gradually deactivate regardless of level of use, typically 3 years); heat
sensitive (faster deactivation at higher temps); not good for high power drain devices; built-in computer chip and
protection circuit to monitor charging and discharging, temp, etc. (adds to cost); internal short-circuit can cause fire due to
high power density (recent fires and recalls for notebook computer batteries).
Refs:

http://electronics.howstuffworks.com/lithium-ion-battery1.htm
http://www.buchmann.ca/Article5-Page1.asp
http://en.wikipedia.org/wiki/Lithium_ion_battery

CHEM 1422 - Homework # 7


Redox & Electrochemistry
Due Tuesday, April 21 (4 PM)

ANSWER KEY
IMPORTANT: for questions 1-4, it is very important to think qualitatively and
understand what oxidizing agents (they get reduced) and reducing agents
(they get oxidized) do. You should understand from the periodic table trends
that Li+, for example, cant act as a reducing agent. Nor can F act as an
oxidizing agent.

1. (1 pt) Which of the following substances is the best reducing agent? Briefly explain your answer.
a) Na+

c) Li+

b) Zn

d) Ag

e) Al

It is IMPOSSIBLE for Na+ and Li+ to act as reducting agents that would produce Na2+ and Li2+. So you can limit
your consideration to Zn, Ag, and Al. Of these three, Al has the most positive oxidation potential (reverse of
reduction potential) = +1.66 V

2. (1 pt) Which of the following substances is the best oxidizing agent? Briefly explain your answer.
b) Li+

a) O2

e) F

d) Ag+

c) Cl2

It is IMPOSSIBLE for F to act as an oxidizing agent that would produce F2. Li+ is very happy where it is, and has
a very negative (non-spontaneous) reduction potential, so that can be ruled out as well. That leaves O2, Cl2, and Ag+.
Cl2 has the most positive reduction potential (+1.36 V), so that is your best answer.

3. (1 pt) Which of the following substances is the best reducing agent? Briefly explain your answer.
a) F2

c) Li+

b) Mg

e) Zn2+

d) Na

It is IMPOSSIBLE for F2, Li+, and Zn2+ to act as reducting agents that would produce F+, Li2+, and Zn3+. So you
can limit your consideration to Mg and Na. Of these three, Na has the most positive oxidation potential (reverse of
reduction potential) = +2.71 V

4. (1 pt) Which of the following substances is the best oxidizing agent? Briefly explain your answer.
a) H+

b) Al3+

c) Ag+

d) Li

e) O3

It is IMPOSSIBLE for Li to act as an oxidizing agent that would produce Li. Of the others, O3 has the most positive
reduction potential (+2.07 V), so that is your best answer.

5. (3 pts) Balance the following rxn in acidic solution (add water or H+ as needed). Clearly show your work.
Oxidation states:

1
I (aq) +

+7 2

MnO4(aq)

The two half cell rxns are:


Oxidation:
2I
I2 + 2e
Reduction: MnO4 + 3eMnO2
Note that you have to have at least 2I- in order to make one I2.
It is important to perform this internal balancing within the half
cell first.
The common factor is 6e, so we need to multiply the first half
reaction by 3 and the second by 2, then add the reactions
together and cancel out the 6e on each side to give the overall
core redox balanced reaction:
6I + 2MnO4

3I2 + 2MnO2

+4 2

I2(aq) + MnO2(s)
Now, check that the # of oxygen atoms on each side
matches!
There are 8 oxygens on the reactant side, and 4 oxygens on
the product side. Add 4 water molecules to the product side
that is missing oxygens, then add 8H+ to balance the
hydrogens on the reactant side to produce our final balanced
equation. Note that the spectator cations are not included.
8H+ + 6I + 2MnO4

3I2 + 2MnO2 + 4H2O

CHEM 1422 HW # 7 Redox & Electrochemistry

6. (3 pts) Balance the following rxn in basic solution (add water or OH as needed). Clearly show your work.
Oxidation states:

+6 2

+1 2

ClO
Cr2O72 + 6e-

ClO4 + 6e2Cr3+

Note that we need to add 2Cr3+ because the Cr2O72 has two
redox active Cr6+ atoms in it. Each Cr6+ reacts with 3e- for a
total of 6e-. Thus, each half cell uses 6e- and we are balanced:
Cr2O72 + ClO

Cr3+(aq) + ClO4(aq)
8H+ + Cr2O72 + ClO

Write out the REDOX half cells and balance:


Ox half rxn:
Red half rxn:

+7 2

+3

Cr2O72(aq) + ClO(aq)

2Cr3+ + ClO4

2Cr3+ + ClO4 + 4H2O


Now, convert to basic solution by adding as many OH to each
side as there are H+(you need to add 8OH). React the added
OH with the H+ on the product side to make H2O's:
8H2O + Cr2O72 + ClO

2Cr3+ + ClO4 + 4H2O + 8OH

Finally, cancel out excess waters (4 on each side):

Now, check that the # of oxygen atoms on each side matches!


There are 8 oxygens on the reactant side, and 4 oxygens on the
product side. Add 4 water molecules to the product side that
missing 4 oxygens, then add 8H+ to the reactant side to balance
the hydrogens:

4H2O + Cr2O72 + ClO

2Cr3+ + ClO4 + 8OH

7. (5 pts) Write the oxidation state for the underlined element in the box following each compound.
a) NaH

b) KNO3

+5

d) Ca3(PO3)2

+3

e) Na(NCS)

c) Na2PtCl6

+4

8. (5 pts) Calculate the redox potentials for the following reactions. Show the two half cell reactions used to
calculate the overall potential.
6H(aq) + 2Al(s)

a) 3H2(g) + 2Al3+(aq)
2H+ 2eAl

H2
Al+3 + 3e

E = 0.00 V
E = 1.66 V

Ag + Cl
Mg2++ 2e-

2F
O2 + 4H++ 4e-

d) Mg2+(aq) + Cu(s)
Mg2++ 2eCu

2.58 V

Ag
Li++ 1e-

E = 2.87 V
E = 1.23 V

1.64 V

E = 2.36 V
E = 0.34 V

2.70 V

E = 0.80 V
E = 3.05 V

3.85 V

Mg(s) + Cu2+(s)

Mg
Cu2++ 2e-

e) Li(s) + Ag+(aq)
Ag+ + 1eLi

E = 0.22 V
E = 2.36 V

4F(aq) + 4H(aq) + O2(g)

c) 2F2(g) + 2H2O(l)
F2 + 2e2H2O

Nonspontaneous

2Ag(s) + Mg2+(aq) + 2Cl(aq)

b) 2AgCl(s) + Mg(s)
AgCl + 1eMg

1.66 V

Li+(aq) + Ag(s)

Nonspontaneous

CHEM 1422 HW # 7 Redox & Electrochemistry

9. (4 pts) A MgCl2 solution containing a Mg electrode is connected by means of a salt bridge to a CuCl2
solution containing a copper electrode. Sketch out a Galvanic Cell showing this and clearly indicate the
movement of anions, cations, electrons, which electrode is dissolving, and which is forming a metallic
deposit. Label the anode and cathode and show the cell potential.
The spontaneous rxn is: Mg(s) + Cu2+

Mg2+ + Cu(s)

Mg(s)
Mg2+(aq) + 2e- E = 2.36 V
Cu2+(aq) + 2eCu(s) E = 0.34 V
Cell potential = 2.70 V

10. (3 pts) How long (in hours) will it take to electrodeposit 1 mole of Al metal by passing a current of 9.65
amps through a solution of Al+3 ? Please clearly show all your work and put a box around your final answer.

(# moles)(96, 485 C/mole e-)(# e ) (1)(96, 485 C/mole e-)(3e-)

(# amps)
(9.65 C/sec)
# sec 29, 995 30, 000
30, 000 sec
# hrs
8.33 hr
3600 sec/hr
# sec

11. (3 pts) What is the concentration of [Ag+] in a half-cell if the reduction potential of the Ag+/Ag couple is
observed to be 0.40 V? Clearly show all your work and put a box around your final answer.
Ag+(aq) + e-

Ag(s)

+0.80 V

1
[ Ag]

Rearrange the Nernst equation and solve for Q, from which we can calculate [Ag+]:

E E

0.052
n(E E) (1)(0.4)
log(Q) ; or: log(Q)

6.757
n
0.0592 0.0592

Take the antilog (10x) of each side to get the value of Q: Q = 106.767 = 5.71 106

[Ag+]

1
1

1.75 10 7 M
Q 5.71106

CHEM 1422 - Homework # 8


Organic & ChemDraw
Due Friday, May 1 (by 3:00 PM)

ANSWER KEY

Please download ChemDraw from Tigerware (located under Scientific Software, Chemistry software) and
install (along with the included Chem3D program) on your Windows or Macintosh computer. You need to
register with CambridgeSoft using an LSU e-mail address in order to get a license key (serial #) for installing
ChemDraw. I sent instructions on this via e-mail. Study groups can work together, as usual, but each student
needs to E-mail their own typed report with ChemDraw structures to Prof. Stanley as a Word or PDF file.
Prof. Stanley is available to answer questions about ChemDraw.
1. (20 pts) Sketch out nice ChemDraw structures for the following molecules. Each should be shown in
standard organic line notation and with all atoms identified.
Example:

H2C

tetrahydrofuran:

CH2

H2C

CH2

H
Or ganic line
notation

C
H

Tw o possible w ay s of drawing
"complete" str uct ur es

a) 2-hexanone

b) heptanal

c) Z-2-octene

d) E-2-octene

e) dimethylformamide

f) 2-pentyne

g) benzoic acid

h) 6-ethyl-3-methylnonane

i) 1,3-diethylbenzene

j) pyridine

2. (10 pts) Name and redraw the following molecules:


a) vinylcyclohexane

b) 5,6-dimethyloctan-3-imine

d) (1s,4s)-bicyclo[2.2.1]hepta-2,5diene

e) but-2-yn-1-amine

c) naphthalene

CHEM 1422 - Homework # 9-10


Gaussian & GaussView
Due Thursday, April 30 (by 4:00 PM)

ANSWER KEY

Please download Gaussian 03 and GaussView 4 from Tigerware (located under Scientific Software, Chemistry
software) and install on your Windows computer. Up to 3 people can work together on this assignment.
Macintosh users should pair up with those with PC/Windows computers. Submit one copy of your report and
dont forget to put all the names of those working together on the report. All reports should be typed, formatted
nicely, and include color images (when possible).
Prof. Stanley is available to answer questions about Gaussian & GaussView. Separate instructions for using
GaussView and Gaussian have been posted by Prof. Stanley.
This double assignment will count for 60 pts.
1. (40 pts) Do DFT molecular orbital calculations on perchloric, sulfuric, and phosphoric acids (optimize, DFT,
B3LYP, 6-311G(d) basis set). Make sure you select a single d function on your basis set.

Questions:
a) Capture and display the optimized molecules with their Mulliken charges.
Perchloric Acid

Sulfuric Acid

Phosphoric Acid

b) Show the electrostatic surface potential plots using a density value of 0.04. Adjust the color scale for
perchloric acid to best represent the atomic charges, then use the same numerical ranges for displaying the
other two molecules.
Perchloric Acid

Sulfuric Acid

Phosphoric Acid

charge range = -0.02 to +0.4

charge range = -0.02 to +0.4

charge range = -0.02 to +0.4

CHEM 1422 Honors General Chemistry Gaussian Homework

c) Do the charges tell you anything about the acidity of the molecule? Note that the calculation is done in
vacuum with no solvent molecules around a proton will not dissociate without water molecules to
interact with. Discuss any correlation of the charges calculated with the acidity of the molecules.
There appears to be some correlation between the acidity and the atomic charges, but not a very strong
one. Consider the charge data in table format showing the comparisons:
Charges
Atom

HClO4

H2SO4

H3PO4

0.45

0.47

0.45

O(-H)

-0.54

-0.64 (avg)

-0.69 (avg)

O(=X)

-0.35 (avg)

-0.44 (avg)

-0.56

1.15

1.24

1.28

No correlation with the H positive charge, which is where one might expect the largest effect.
The positive charge on the central X atom does increase as one might expect from electronegativity effects:
P is the least electronegative and should lose the most electron-density to the electronegative oxygen
atoms. Directly related to this is the fact that the negative charge on the O-H oxygen atoms increases as
one goes from HClO4 to H2SO4 to H3PO4. The higher the negative charge on the O-H oxygen atom, the
stronger the electrostatic attraction to the H atom, and the harder it will be for the H+ to dissociate. The
lower negative charge on the HClO4 oxygen (O=X) atoms means that they should be able to pick up and
spread out the negative charge when the H+ dissociates. This will also help make HClO4 the strongest
acid.
But you cant carry this analysis too far as H2SO4 looks fairly close to H3PO4, but they are, in fact, quite far
apart with respect to their acidities.

d) List and compare the bond distances for the 3 molecules in table format and discuss how they compare to the
localized structures drawn above. Do the bond distances fit any periodic trends?
Distances ()
HClO4

H2SO4

H3PO4

O-H

0.974

0.969 (avg)

0.965 (avg)

X-O(-H)

1.720

1.628 (avg)

1.608 (avg)

1.456 (avg)

1.440 (avg)

1.467

O(=X)

The X=O and X-OH bond distances correlate extremely well with the double and single bonds drawn at the
beginning of this assignment. Double bonds between two atoms are almost always shorter than single
bonds. The X=O bond distances are shorter than the single bonds by amounts ranging from 0.141 for
H3PO4, to 0.264 for HClO4, which is typical for double vs. single bonds.
There isnt any recognizable trend in bond distances. The double bond distances decrease as one goes
from P=O to S=O, which does fit with the periodic trend of decreasing atom sizes as one goes from P to S.
But the Cl=O bond distance increases, even through Cl is even smaller than S. This indicates that the O=Cl
double bond is getting weaker, which would increase the bond distance. This would be consistent with
periodic trends as Cl does not typically like expanded valence and high oxidation state situations
The X-OH bond distances, also, do not fit the pattern of decreasing covalent radii for P-S-Cl as their bond
distances increase as one goes from P to Cl. No clear reason for this that you should know of. But, X-OH
bond strengths decrease as one goes from P to Cl, so the increasing bond distances do seem to clearly
support the trend of weaker -bonding between the X and OH atoms.

CHEM 1422 Honors General Chemistry Gaussian Homework

e) Show the highest occupied molecular orbital (HOMO) for each molecule along with its energy in eV.
Discuss and explain the trend (if any) in the energies of the HOMOs (lower is more stable) for these three
molecules with periodic properties and what we discussed in class for oxyacids. Discuss what kind of
bonding, non-bonding (lone pair), or antibonding interactions are occurring within the HOMO for each
molecule. Although one might expect hybrid orbitals (e.g., sp3) for some of these, MO calculations often
tend to display orbitals as more or less pure s, p, etc. For example, an oxygen lone pair is more likely to
show up as a p orbital.
Perchloric Acid

Sulfuric Acid

Phosphoric Acid

9.58 eV

8.98 eV

8.37 eV

All three HOMOs are mainly oxygen pure p lone pair type orbitals. They are non-bonding (no bonding or
antibonding with respect to the X-O or O-H bonds). They have weak antibonding through-space
interactions with adjacent oxygen atoms. This weak repulsion changes the shape from that of a pure
isolated p orbital to the that shown where the opposite color orbital lobes repel one another. There may be
a very small amount of X=O -bonding present for H2SO4 and H3PO4, as indicated by the same color
orbital connection between the doubly bound oxygen atoms and the central atom. The larger orbital size on
the X=O atoms indicates that the p-orbitals (lone pairs) on these atoms are higher in energy relative to the
lone pairs on the OH oxygen atoms, at least for this specific orbital.
The trend in the orbital energies is exactly what one expects based on the increasing electronegativity of the
central atom. HClO4 has the lowest energy because Cl is the most electronegative central atom. S is next,
followed by P. There is a fairly uniform shift of approximately 0.6 eV to higher energy as one goes from Cl
to S to P in the HOMO energy, which follows the periodic electronegativity trend.

CHEM 1422 Honors General Chemistry Gaussian Homework

2. (20 pts) Shirakura & Suginome published a paper in the Journal of the American Chemical Society (2008,
130, 5410-5411) on nickel catalyzed coupling of silylacetylenes with dienes:

The first step in the catalysis is the oxidative addition (breaking) of the alkyne C-H bond to the nickel atom to
make a nickel-hydride-alkynyl complex:

In order for a C-H bond to do an oxidative addition to a Ni metal center the orbital associated with the C-H
bond must be reactive enough to want to participate in this reaction. The higher the energy of the orbital the
more reactive it will be (usually).
This reaction apparently does not work for regular alkynes that do not have silyl groups attached.
Use Gaussian (optimization, DFT, B3LYP method, 6-311G basis set) to perform calculations on the following
alkyne molecules:

Although I didnt ask for it here are the various angles and bond distances for these three optimized molecules from
Gaussian:
Molecule

Experimental

()C-H

CC

H3X-C

X-H (avg)

CC-H

X-CC

H-X-C (avg)

1.061

1.203

---

---

180.00

180.00

---

1.064

1.213

1.461

1.097

180.00

180.00

111.36

1.066

1.219

1.854

1.500

180.00

180.00

109.85

0.950**

1.194

1.83

179.9

176.7

106.9
to
109.4

** X-H bond distances (X = almost any element) from X-Ray structures are almost always too short due to the small
amount of electron density around a H atom. X-Ray diffraction relies on diffraction of the X-Rays off the electrons
around an atom. Most X-H bonds have most of the electron density around the X atom, which is also generally more
electronegative and pulls the single electron around the H atom towards the X atom. This results in X-H bond
distances that are too short. Neutron diffraction gives accurate X-H bond distances since the neutrons diffract off the
atomic nuclei, not the electrons around each atom.

CHEM 1422 Honors General Chemistry Gaussian Homework

Questions:
a) Identify the highest energy (C)C-H bonding orbital (# from calculation), list the energy of each in eV, and
show a picture of this molecular orbital for each alkyne.
Molecule

MO #

E (eV)

#5

13.79

#7

12.34

# 11

11.01

Highest Energy (C)C-H MO Orbital

b) How does the energy of the orbital from question # 2 affect the C-H bond strength? How does the group
attached to the other side of the alkyne affect this energy?
The higher the energy of an orbital (less negative) the less stable that orbital is and the less it contributes to
the bonding between atoms. Although the three orbitals shown are NOT the only C-H bonding orbitals, the
trend in this three is a steady decrease in the MO energy. This is definitely related to the group change
from H to CH3 to SiH3. Exactly how, is a more complicated answer, which I dont expect you to know and
are discussed below.
There are several factors that can play a role here. Typically as you add more bonded atoms to a
molecule, there is less of any one bonding interaction present in any one MO. The bonding present in the
CH3 and SiH3 groups spreads out the orbital density away from the ()C-H bond and shifts it towards the
CH3 and SiH3 groups. There is also something called the trans effect where a strong bond on one side of
an atom can weaken another bond on the opposite side. The CC triple bond is a good transmitter of this
type of effect, so the stronger donating CH3 and SiH3 groups can potentially weaken the C-H bond a small
amount. The other orbital that plays an important role in the C-H bond breaking is the lowest energy empty
C-H antibonding orbital. Generally speaking, the higher the energy of the bonding orbital, the lower the
energy of the antibonding analog. The lower the energy of the C-H antibonding orbital, the easier it is for
the Ni atom to donate some of its electrons into that antibonding orbital and help break the C-H bond.
The last effect concerns the energy of the CC -bonding orbitals. The donating CH3 and SiH3 groups
make the triple bond more electron-rich & higher in energy (see next question) and that, in turn, makes it a
better donor to the Ni metal center. The alkyne has to first coordinate to the Ni atom via the CC bonding MO before the C-H bond can be broken.

CHEM 1422 Honors General Chemistry Gaussian Homework

c) List and compare the energies (in eV) of the alkyne -bonding orbitals for each system and show a picture
of one of these molecular orbitals for each alkyne. Although these arent directly involved in the C-H bond
breaking step discussed above, what factor seems to be affecting the energy of these orbitals?
Molecule

MO #

E (eV)

#7

8.13

# 11

7.13

# 15

7.69

Highest Energy (C)C-H MO Orbital

The two of the C-H orbitals on the CH3 and SiH3 groups can weakly interact with the CC -bonding MO in
an antibonding fashion to push it up in energy relative to the H-CC-H baseline case. The spherically
symmetric H s orbital cannot interact with the CC -bonding MO when it is collinear with the CC bond
axis. BUT, when two of the H-atoms on the CH3 or SiH3 groups are oriented up and down from the CC
bond axis their s-orbitals can now weakly interact with the CC -bonding orbitals in both a bonding and
antibonding fashion. In the orbitals above, we are seeing the weak antibonding interaction that affects and
pushes up the energy of the CC -bonding MO. If you look through the lower energy orbitals you will find
one that looks similar, but has a -type bonding interaction between the X-H bonds (X = C or Si) and the
CC -system (shown below for H3C-CC-H & H3Si-CC-H).

11.7 eV

10.0 eV

The CH3 C-H orbitals are close enough to the CC -bonding MO to have a stronger bonding/antibonding
interaction compared to the SiH3 group, which is further away and has longer X-H bonds: C-C(C) = 1.46
, Si-C(C) = 1.85 , H2C-H = 1.1 , H2Si-H = 1.5 . The further away orbitals are from one another the
weaker their interactions (either bonding or antibonding). This is why the HOMO in H3Si-CC-H is lower in
energy (more stable) relative to the same orbital for H3C-CC-H.

You might also like